◆ わからない問題はここに書いてね 12 ◆

このエントリーをはてなブックマークに追加
1132人目のさくらたん

    , ― ノ)
 γ∞γ~  \   / ̄ ̄ ̄ ̄ ̄ ̄ ̄ ̄ ̄ ̄ ̄ ̄ ̄ ̄ ̄ ̄ ̄
 人w/ 从从) )  わからない問題はここに書いてね ♥
  ヽ | | l  l |〃 過去スレ,業務連絡・その他,関連スレ,
  `wハ~ ーノ)   数学記号の書き方例は >>2-10 の中にあるよ♪
   / \`「   \_________________

【前スレ】
◆ わからない問題はここに書いてね 11 ◆
http://cheese.2ch.net/test/read.cgi?bbs=math&key=997329928
2132人目のさくらたん:01/09/05 20:32 ID:osuYd0VA
【過去スレ】
◆ わからない問題はここに書いてね ◆
http://cheese.2ch.net/test/read.cgi?bbs=math&key=967755172
◆ わからない問題はここに書いてね 2 ◆
http://cheese.2ch.net/test/read.cgi?bbs=math&key=970795775
◆ わからない問題はここに書いてね 3 ◆
http://cheese.2ch.net/test/read.cgi?bbs=math&key=974911042
◆ わからない問題はここに書いてね 4 ◆
http://cheese.2ch.net/test/read.cgi?bbs=math&key=978209589
◆ わからない問題はここに書いてね 5 ◆
http://cheese.2ch.net/test/read.cgi?bbs=math&key=981372834
◆ わからない問題はここに書いてね 6 ◆
http://cheese.2ch.net/test/read.cgi?bbs=math&key=985594205
◆ わからない問題はここに書いてね 7 ◆
http://cheese.2ch.net/test/read.cgi?bbs=math&key=988952592
◆ わからない問題はここに書いてね 8 ◆
http://cheese.2ch.net/test/read.cgi?bbs=math&key=991223596
◆ わからない問題はここに書いてね 9 ◆
http://cheese.2ch.net/test/read.cgi?bbs=math&key=993571403
◆ わからない問題はここに書いてね 10 ◆
http://cheese.2ch.net/test/read.cgi?bbs=math&key=995448453
◆ わからない問題はここに書いてね 11 ◆
http://cheese.2ch.net/test/read.cgi?bbs=math&key=997329928
3132人目のさくらたん:01/09/05 20:32 ID:osuYd0VA
【業務連絡・その他】
■900を超えたら新スレに移行準備.
■旧スレ側 → 終了宣言,新スレへの誘導.
■新スレ側 → 開始宣言と目次,旧スレのリンク,掲示板での数学記号の書き方例,業務連絡・その他,旧スレ側の残り問題の移動.
■数学板の要望スレで数学板の注意書き(リンク先)の変更依頼.

■単独の質問スレは,このスレか「くだらんスレ」に誘導して下さい.
■誤って過去スレに書き込まれた質問は,最新スレに誘導して下さい.


【数学板削除依頼スレ】
http://teri.2ch.net/test/read.cgi?bbs=saku&key=986384122 (レス削除)
http://teri.2ch.net/test/read.cgi?bbs=saku&key=987829968 (スレッド削除)

【リンク先更新総合スレッド】
http://teri.2ch.net/test/read.cgi?bbs=accuse&key=992178408
4132人目のさくらたん:01/09/05 20:33 ID:osuYd0VA
【掲示板での数学記号の書き方例】
■数の表記
 ●スカラー:a,b,c,...,z, A,B,C,...,Z, α,β,γ,...,ω, Α,Β,Γ,...,Ω, ... (← ギリシャ文字はその読み方で変換可.)
 ●ベクトル:V=[V[1],V[2],...], |V>, V↑, vector(V) (← 混同しない場合はスカラーと同じ記号でいい.通常は縦ベクトルとして扱う.)
 ●テンソル(上下付き1成分表示):T^[i,j,k...]_[p,q,r,...], T[i,j,k,...;p,q,r,...]
 ●行列(1成分表示):M[i,j], I[i,j]=δ_[i,j]
 ●行列(全成分表示):M=[[M[1,1],M[2,1],...],[M[1,2],M[2,2],...],...], I=[[1,0,0,...],[0,1,0,...],...] (← 列(または行ごと)に表示する.)

■演算・符号の表記
 ●足し算:a+b
 ●引き算:a-b
 ●掛け算:a*b, ab (← 通常は"*"を使い,"x"は使わない.)
 ●割り算・分数:a/b, a/(b+c), a/(bc) (← 通常は"/"を使い,"÷"は使わない.)
 ●複号:a±b=a士b, a干b (← "±"は「きごう」で変換可.他に漢字の"士""干"なども利用できる.)
 ●内積・外積・3重積:a・b=(a,b), axb=a∧b=[a,b], a・(bxc)=(axb)・c=det([a,b,c]), ax(bxc)

■関数・数列の表記
 ●関数:f(x), f[x]
 ●数列:a(n), a[n], a_n
 ●平方根:√(a+b)=(a+b)^(1/2) (← "√"は「るーと」で変換可.)
 ●指数・指数関数:a^b, x^(n+1), exp(x+y)=e^(x+y) (← "^"を使う."exp"はeの指数.)
 ●対数・対数関数:log_{a}(b), log(x/2)=log_{10}(x/2), ln(x/2)=log_{e}(x/2) (← 底を省略する場合,"log"は常用対数,"ln"は自然対数.)
 ●三角比・三角関数:sin(a), cos(x+y), tan(x/2)
 ●行列式・トレース:|A|=det(A), tr(A)
 ●絶対値:|x|
 ●ガウス記号:[x] (← 関数の変数表示などと混同しないように注意.)
 ●共役複素数:z~
 ●転置行列・随伴行列:M† (← "†"は「きごう」で変換可.)
 ●階乗:n!=n*(n-1)*(n-2)*...*2*1, n!!=n*(n-2)*(n-4)*...
 ●順列・組合せ:P[n,k]=nPk, C[n.k]=nCk, Π[n,k]=nΠk, H[n,k]=nHk (← "Π"は「ぱい」で変換可.)

■微積分・極限の表記
 ●微分・偏微分:dy/dx=y', ∂y/∂x=y,x, D^(n)f(x) (← "∂"は「きごう」で変換可.)
 ●ベクトル微分:∇f=grad(f), ∇・A=div(A),∇xA=rot(A), (∇^2)f=Δf (← "∇"は「きごう」,"Δ"は「でるた」で変換可.)
 ●積分:∫[0,1]f(x)dx=F(x)|_[x=0,1], ∫[y=0,x]f(x,y)dy, ∬_[D]f(x,y)dxdy, 点[C]f(r)dl (← "∫"は「いんてぐらる」,"∬"は「きごう」で変換可.)
 ●数列和・数列積:Σ_[k=1,n]a(k), Π_[k=1,n]a(k) (← "Σ"は「しぐま」,"Π"は「ぱい」で変換可.)
 ●極限:lim_[x→∞]f(x) (← "∞"は「むげんだい」で変換可.)

■その他
 ●図形:"△"は「さんかく」,"∠"は「かく」,"⊥"は「すいちょく」,"≡"は「ごうどう」,"∽"は「きごう」で変換可.
 ●論理・集合:"⇔⇒∀∃∧∨¬∈∋⊆⊇⊂⊃∪∩"は「きごう」で変換可.
 ●等号・不等号:"≠≒≦≧≪≫"は「きごう」で変換可.

※ ここで挙げた表記法は1例であり,標準的な表記法からそうでないものまで含まれているので,後者の場合使う時にあらかじめことわっておいたほうがいい.
※ 関数等の変数表示や式の括弧は,括弧()だけでなく[]{}を適当に組み合わせると見やすい場合がある.
※ 上記のほとんどの数学記号や上記以外の数学記号は大体「きごう」で順次変換できる.
5132人目のさくらたん:01/09/05 20:37 ID:osuYd0VA

━━━━━━━━━━━━━━━━━━━━━━━━━━━━━━

          移転完了しました (o^-')b
        ◆ わからない問題はここに書いてね 12 ◆

━━━━━━━━━━━━━━━━━━━━━━━━━━━━━━

★__________________________.
|.            │
│ はにゃ〜ん.   │
| γ∞γ~  \    │
│人w/ 从从) )   │
│ ヽ | |┬ イ |〃  │
│ `wハ~ . ノ)    │
│  / \`「 .     │
| 数学板さくらスレ. │
|_________________________│





(● ´ ー ` ●)ノ さくらスレ旗掲揚
6132人目の素数さん:01/09/05 20:45 ID:DLFo8/Tc
立命の星、それが田中洸人
http://www.geocities.co.jp/Technopolis-Mars/3422
7132人目の素数さん:01/09/05 20:59 ID:LVxWuD9M
球の体積ってなんでしたっけ?
3分の4πですか?
3分の4π三乗ですか?
誰か教えて下さい
8132人目の素数さん:01/09/05 21:22 ID:8kSoYhrM
z' = ( a + iω ) z - ( | z |^2 ) * z
a : 定数

z(t) = r(t) exp(i(t)) として、z(t) を求めてください。
また、t→∞のとき、r(t)はどのような値に近づきますか?
9>7:01/09/05 21:34 ID:1FVjIshY
>球の体積ってなんでしたっけ?
>3分の4πですか?
>3分の4π三乗ですか?

大きい球と小さい球では体積が違うことは明らか
だったら半径に依存するはず
ということはわかるだろ。
3分の4πですか?
だの
3分の4π三乗ですか?
だの
半径に依存しないわけがなかろ
10ぐんぐん:01/09/05 21:39 ID:3R6bvKU6
V=4πr^2
S=4πr^3/3
体積のほうはあってるか微妙。
微積を使えば簡単に証明できるはず。
表面積の方はちょっとだけ難しいかも。。
11>:01/09/05 21:40 ID:1FVjIshY
で結論いうと
3分の4π * (半径の三乗)

導出は”くだらない問題”スレにある
12132人目の素数さん:01/09/05 22:15 ID:IJ9rA1oI
半径 1 の n 次元球の体積を A_n とすると、
A_n/A_{n-1}=∫[-1,1](1-x^2)^((n-1)/2)dx
=∫[0,1](1-t)^((n-1)/2)t^(-1/2)dt
=B((n+1)/2,1/2)=Γ((n+1)/2)Γ(1/2)/Γ((n+2)/2)
A_1=2 だから、A_n=π/Γ(n/2+1)
半径 r の n 次元球の体積を V_n 、表面積を S_{n-1} とすると
V_n=πr^n/Γ(n/2+1), S_{n-1}=dV_n/dr=nπr^(n-1)/Γ(n/2+1)
1312:01/09/05 22:20 ID:IJ9rA1oI
訂正、スマソ
A_n=π^(n/2)/Γ(n/2+1)
V_n=π^(n/2)r^n/Γ(n/2+1)
S_{n-1}=nπ^(n/2)r^(n-1)/Γ(n/2+1)
1410:01/09/05 23:06 ID:3R6bvKU6
すんません。VとS逆
15132人目の素数さん:01/09/05 23:18 ID:TV7n.9ac
(1)a=1,2,3,4,5,6に対して ax≡1, mod 7 となるxをみつけろ
(2)pを素数とする.a=1,2,…p-1 に対して ax=1, mod p となるx
が存在することを示せ
だれかお願いします。
16132人目の素数さん:01/09/05 23:23 ID:RhrlmwkE
こんばんは。質問させていただきたいと思います。
a(1)=1,a(n+2)=3a(n+1)-2a(n),lim_[n→∞]a(n^2)/a(2n)=3
を満たす数列a(n)の第2項を求めよ。
という問題なのですが…。ご解答をお願い致します。
17ごろう:01/09/05 23:25 ID:.wZ5wL..
x→0のときxlogx→0を証明せよ。
おねがいします
18>15:01/09/05 23:34 ID:TV7n.9ac
マジむずいっすね。
19132人目の素数さん:01/09/06 00:05 ID:SqNhYyjE
>>15
(1)
(a,x,ax−1)=(1,1,0),(2,4,7),(3,5,14),(4,2,7),(5,3,14),(6,6,35)

(2)wakaran
20132人目の素数さん:01/09/06 00:20 ID:KmsCHSqs
>>17
xlogx=logx/(1/x)とかに変形して、
ロピタルの定理を使うと、-xになるので、
x→0のときxlogx→0って感じ?
21高校2年:01/09/06 00:35 ID:biGDmU/A
a(1)=2, b(1)=1
a(n+1)={a(n)+b(n)}/2, b(n+1)=2*a(n)*b(n)/{a(n)+b(n)} (n=1,2,3,・・・)
で与えられる漸化式について、

nが増加するにしたがって、a(n)は単調減少、b(n)は単調増加し、a(n)>b(n)であることを示す

これをやっていただけませんか?
22132人目の素数さん:01/09/06 00:41 ID:EY.FvgT6
>>15
◆ わからない問題はここに書いてね 11 ◆ の918 に回答しときました
23132人目の素数さん:01/09/06 02:07 ID:cmNjX.ow
>>16
a(n+2)=3a(n+1)−2a(n)を満たすなら
lim_{n−>∞}(a(n^2)/a(2n))は
1か∞で3にはならない。
24132人目の素数さん:01/09/06 02:12 ID:cmNjX.ow
>>21
a(n)>b(n)>0ならば
a(n)>a(n+1)>b(n+1)>b(n)
を示せばいい。
25132人目の素数さん:01/09/06 02:45 ID:t6xe4MPE
>>16
lim[n→∞]a(n)^2/a(2n)じゃろ?
26132人目の素数さん:01/09/06 04:24 ID:cmNjX.ow
R×R−>Rの関数gが任意の実数a,b,cに対して
g(a,b)+g(a,c)=g(a,b+c)
g(g(a,b),c)=g(a,bc)
g(a,c)+g(b,c)=g(a+b,c)
g(a,1)=a
となるときgを求めよ。
これ分かる人いますか。
27132人目の素数さん:01/09/06 04:44 ID:z3el98Ek
>>15
x,y が整数全体を動く時、ax+py は a,p の最大公約数
の倍数全体を動く。p が素数で a=1,2,...,p-1 のときは
a と p は互いに素だから ax+py=1 となる x,y がある。
このとき ax≡1 (mod p)
>>16
a(n)=2^(n-1)(a(2)-1)-a(2)+2 だから、23さんの指摘どおり。
25 さんに従えば、a(2)=7
28132人目の素数さん:01/09/06 06:08 ID:lLcVOnSA
バカな質問ですいません、

f(x)=e^x 、f:R→R とか
f(x)=3x-4 、f:R→R って単射になるんですか?
理由も全くわからないのですが、、、
29132人目の素数さん:01/09/06 06:31 ID:3a2WzsjA
>>28

グラフを書きゃ自明
30132人目の素数さん:01/09/06 07:28 ID:PGUi6pTI
スレ違いならすいません。綿刑型計画法・図解法・シンプレックス法って何ですか?
よろしかったら、意味と解き方お願いします。
31132人目の素数さん:01/09/06 07:55 ID:rzmMK30o
>>28

「単射」ってなんだか知ってるの?
32132人目の素数さん:01/09/06 08:06 ID:orEPthFA
>>26
その条件
⇔以下で定義される写像:R→Hom_Z(R,R)=Hom_Q(R,R)

 r→(x→g(x,r))

 が環準同型

逆に環準同型φ:R→Hom_Z(R,R)をとるとき

 g(x,r)=φ(r)(x)

でさだめるとこれは与えられた条件をみたす。
R→Hom_Z(R,R)は山のようにあるのでその条件だけではgは一意には
さだまらないような気がする。
33>28:01/09/06 08:39 ID:PKwnvsDw
x != y のとき f(x) != f(y) が 単射の定義
対偶をとって "f(x)=f(y) のとき x=y" でもよい

そうすると28の例は f(x)=f(y) なら x=y を示せばよい
これは容易。。。
34132人目の素数さん:01/09/06 08:53 ID:ArLcDmNo
>x != y

一般的でなさげ
x≠yの意味アルyo
35>:01/09/06 09:09 ID:PKwnvsDw
スマソ。記号入力がめんどくさかったっす。
36132人目の素数さん:01/09/06 11:50 ID:TCR2NoU6
証明できることを証明できるんですか。
37_man:01/09/06 12:23 ID:dqCksqtM
どうしても解らなかったんで、誰か考えてください。

ペプシコーラのボトルキャッププレゼントをイメージして下さい。
ボトルキャップは50種類があって、できれば全部集めたいとする。
キャップは商品1本に1個付いてきて、(ここでは)買って開封するまでどの種類かはわからず、
また、欲しい種類を1発で引き当てる確率は50分の1(全種類分の1)とする。
できれば、全50種のうち(9割5分の)45種は集めたいが、その期待値が50%を越えるためには、
商品を何個買わなければならないか・・・。

[ 質問 ] 均等な比率のn種のアイテムから構成される無数のサンプルがある。
ここからx個を無作為に取り出したとき、とりだしたアイテムがm種(m<n)以上になる確率が
(仮に)1/2以上となるようなxはなにか。
※ 言い回しが変な場合は、うまく汲み取って下さい。

これって中高生レベルの「順列組合せ」&「数学的帰納法」あたりで
簡単に判りそう、と思ったところ、nが増えると「例外のパターン」みたいのが
増えていき、シンプルにくくれず手におえなくなっていきました(オレの頭では)。

誰か、お願い!
※ できればポワソン分布みたいな「数をまるめる」考えでなく、お願いしたいのだが・・・。
38おじさん:01/09/06 14:34 ID:z3el98Ek
>>37
((1/n)^x)Σ[n=m,k]C(n,k)Σ[i=0,k-1]((-1)^i)C(k,i)(k-i)^x≧1/2
ですか?私の手には負えない・・・
3938:01/09/06 14:42 ID:z3el98Ek
訂正
((1/n)^x)Σ[k=m,n]C(n,k)Σ[i=0,k-1]((-1)^i)C(k,i)(k-i)^x≧1/2
n 種類のうち n-k 種類は 0 個、残りは少なくとも一つある。
というやり方ね。残り・・・に関しては、i 種類を 0 個に
固定した場合を考えて包除原理を使う。
40おじさん:01/09/06 16:25 ID:z3el98Ek
>>37
UBASIC(大きな整数が得意)プログラム
10 N=50:M=45:Pinv=2
20 X=M
30 S=0:for K=M to N
40 T=0:for I=0 to K-1
50 T=T+(-1)^I*combi(K,I)*(K-I)^X
60 next I
70 S=S+combi(N,K)*T
80 next K
90 U=N^X
100 print X,S/U
110 X=X+1:if Pinv*S<U goto 30
120 end
x≧109 となりました。あってるかな?
41宇宙の心理:01/09/06 16:31 ID:mOxOP14I
lim(x→0) x/sinx =1 の証明ってなんとうりくらいありましたっけ?
くだらないしつもんですいません。
42おじさん:01/09/06 16:41 ID:z3el98Ek
>>41
sin の定義にもよるでしょうね。
高校だと扇形と三角形の面積を使って
1/2sin2x<x<1/2tan2x であることと、
lim(x→0)cosx=1 から導くのかな?
43_man:01/09/06 16:51 ID:dqCksqtM
>>39
少し時間があるときに、式の内容を理解できるように、考えてみます。

>>40
2倍強の数(50に対する109)を取り出しても、44種類目や45種類目は
なかなかそろわないような気がしたんですが、オレの直感が変なのかな?
ペプシコーラのときはケースでばんばん買った馬鹿(知人です)がいたけど、
全然そろわなかったし・・・。
※ もちろん50種の分布が均一ではなかったのかもしれないが。

どうでしょうか?
44おじさん:01/09/06 18:32 ID:z3el98Ek
>>42 迂遠でした。(1/2)sinx<x/2<(1/2)tanx
から、cosx<sinx/x<1
>>43 そんな気もするので、擬似乱数で実験しました。
(UBASIC) 前の結果は正しそうです。
10 randomize:dim A(100)
20 N=50:M=45:X=109
30 C=0:S=0
40 C=C+1:K=0
50 for I=0 to N-1:A(I)=0:next I
60 for I=1 to X
70 R=int(rnd*N)
80 if A(R)=1 goto 100
90 A(R)=1:K=K+1
100 next I
110 if K>=M then S=S+1
120 print C,S/C
130 goto 40
試行回数とm種類以上になった比率を出力します。
#景品の類いは、購買意欲を煽るために
レアなものを混ぜるらしいです。
45132人目の素数さん:01/09/06 19:24 ID:3hCzgTrY
3次元の領域を角度でどうやって表現すんの?誰か教えてください。
46松尾ゆうこ:01/09/06 21:22
AIC基準で、最大対数尤度の対数の中は有次元量でいいのですか?
確率変数の単位の尺度(例えばmm、km)によってモデルの評価基準は、
おかしなことにならないのでしょうか?例えば用いる単位によっては最大対数尤度が、
相当大きくなって、母数のパラメータ数があまり利いてこないのでは?
教えてください。

授業では可微分多様体を実線形空間の局所座標で習いましたが
一般の体の線形空間でも全く同じように出来るのでしょうか?
また、Cの線形空間による局所座標によって定義された可微分多様体は
Rの局所座標系によって定義された可微分多様体になるのでしょうか?
48ごろ−:01/09/06 21:28
不定積分の問題です。

∫1/(x^3+1)dx

お願いします。
>>48
x^3+1=(x+1)(x+w)(x+w^2)
1/(x^3+1)=A/(x+1)+B/(x+w)+C/(x+w^2)
A,B,C:定数
ってすればいいんじゃない?
計算適当だけど。
50132人目の素数さん:01/09/06 22:03
>>48
答えにarctanが混じると思うんだけど。

 ∫1/(x^3+1)dx
 =∫1/(x+1)dx-∫(x-1)/(x^2-x+1)dx
 =∫1/(x+1)dx-(1/2)∫(2x-1)/(x^2-x+1)dx-(1/2)∫1/(x^2-x+1)dx

第1項、第2項は高校の範囲まででとけるし第3項は

 ∫1/(x^2-x+1)dx=∫1/(((x-(1/2))^2+(3/4))dx

だからx-(1/2)=(√3/2)tanuと置換すればとける。
(けど答えにarctanがまじるけどいいのかな?)
51132人目の素数さん:01/09/06 22:06
>>49
かぶった。スマ。
52おじさん:01/09/06 22:12
>>48
1/(x^3+1)
={1/(x+1)-(x-2)/(x^2-x+1)}/3
={2/(x+1)+(2x-1)/(x^2-x+1)+3/((x-1/2)^2+(√3/2)^2)}/6
とすればよい
53解説もおねがいします:01/09/06 22:13
aを実数とする 等式 sinθ-acosθ+2a-2=0

をみたすθが0<θ<πの範囲に少なくとも1つ存在するようなaの値を求めよ。
54132人目の素数さん:01/09/06 22:16
タダ一つの固有値を持つ4次正方行列の、ジョルダン標準形のすべての型を教えて下さい。
55解説もおねがいします:01/09/06 22:18
θは0<θ<π/2の範囲の角とする
(1)sin3θ=sin2θをみたすθを求めよ
(2)m、nを0以上の整数とする。θについての方程式
sin3θ=msin2θ+nsinθ
が解を持つ時の(m、n)と、その時の解θを求めよ
56ごろー(48):01/09/06 22:22
>>50
他の問題の答えで、arctanが出てきているのでOKです。
みなさんありがとうございました。
57132人目の素数さん:01/09/06 22:23
次のx、yに関する連立方程式が、実数解をもつための実数a,bについての条件を求めよ。
cosx+2cosy=a
sinx+2siny=b



教えてください。
5850:01/09/06 22:26
>>56
>>50は計算まちがってる。スマ。>>52のほうが正しい。
59132人目の素数さん:01/09/06 22:27
aは0と異なる実数とし、f(x)=ax(1-x)とおく。
f(x)≠xかつf(f(x))=xをみたす実数xが存在するようなaの値を求めよ



お願いします。
60132人目の素数さん:01/09/06 22:30
>>54
J_1(α)○J_1(α)○J_1(α)○J_1(α)
J_2(α)○J_1(α)○J_1(α)
J_2(α)○J_2(α)
J_3(α)○J_1(α)
J_4(α)
(ただしA○Bは行列の直和)でいいんじゃないの?
61132人目の素数さん:01/09/06 22:35
>56,58
私もミスを・・・正しくは
={2/(x+1)-(2x-1)/(x^2-x+1)+3/((x-1/2)^2+(√3/2)^2)}/6
>>53
t=tan(θ/2) とおくと 0<t<∞ で、
sinθ=2t/(1+t^2), cosθ=(1-t^2)/(1+t^2)。
これを使えば、tの2次方程式ができるから
正の実数解をもつようにする。
62132人目の素数さん:01/09/06 22:57
>>57
今夜は受験数学多いね。
これ要するに(a,b)=(cosx+2cosy,sinx+2siny)の軌跡をもとめる問題だよね。
yを固定してxだけ動かすと中心(2cosy,2siny),半径1の円。
これでyを動かすとその軌跡は半径3の円の内部と周から半径1の円の
内部をぬいた部分を動くから結局もとめる軌跡は1≦√(a^2+b^2)≦3。
63おじさん:01/09/06 22:58
>>55
(1)sin3θ-sin2θ=2cos(5θ/2)sin(θ/2)
0<θ<π/2 により、5θ/2=π/2
(2)s=sinθ,c=cosθとおくと、s≠0, 0<c<1
s(4c^2-1)=2msc+ns となるから、s≠0 で割って、
4c^2-2mc-(n+1)=0 あとは2次方程式の問題。
64132人目の素数さん:01/09/06 23:03
双子素数(3,5)見たいな感じの2だけ離れた素数の組を
小さい順に探す最適な方法
誰か知ってますか?
65132人目の素数さん:01/09/06 23:05
>>59
f(x)=xの解はax(1-x)-x=0...(*)をといてx=0,1-(1/a)。
f(f(x))=xの解はax{ax(1-x)}{1-ax(1-x)}-x=0...(*)の解。
(**)はx=0,1-(1/a)を解にもつのでx(x-1+(1/a))[二次式]=0と
因数分解できる(ハズ)。その二次式がx=0,x=1-(1/a)以外の
実数解を持つ条件をもとめればよい。
66 :01/09/06 23:36
lim(x→a)f(x)=b ならばlim(x→a)(-f(x))=-b となることを
ε-δ論方をつかって証明していただけないでしょうか、、お願い致します、。。
67132人目の素数さん:01/09/06 23:43
>>66
ε>0 に対し、ある δ>0 があって、|x-a|<δ ⇒ |f(x)-b|<ε。
したがって、この δ に対し、|x-a|<δ ⇒|-f(x)-(-b)|=|f(x)-b|<ε。
6867:01/09/06 23:48
0<|x-a|<δ だ、ごめんね。
6966:01/09/07 00:20
>67-68さん
ありがとうございました!
もうチンプンカンプンで、、、こりゃ単位落とすな。
70はなう:01/09/07 00:34
>>64
面白そう。
ぱっと見他がだめである証明までできんが

nが偶数の時、n番目の双子素数の1個目は
3n(n-2)/2 + 5
で、
nが奇数(かつ≠1)の時、n番目の双子素数の1個目は
{3(n-1)^2}/2 + 5
だね。示し方考え中。
71はなう:01/09/07 01:01
>>70
証明しようとして混乱してきた。だれか、やってー。
とりあえず答えはあってます。
72132人目の素数さん:01/09/07 01:08
>>70ってnがどんなに大きくても成り立つんですか?
73132人目の素数さん:01/09/07 01:12
>nが偶数の時、n番目の双子素数の1個目は
>3n(n-2)/2 + 5

n=10、を入れると、125で、双子以前に、素数にならないが、、、
74132人目の素数さん:01/09/07 01:14
>>70 素数ならば双子って言いたいの?
75はなう:01/09/07 01:19
>>72
>>73
スマソ。その通り。間違ってました。ウツだ・・やりなおそう。
単純な式ではあらわせんのかのぉ。
7674:01/09/07 01:19
でもないか
n=14 で 1番目257(素数),2番目259=7*37
77はなう:01/09/07 01:22
>>74
違います。双子素数の小さい方は6の倍数ずつ変化していることに法則性を見いだそうとしてシッパイしたのです
78 :01/09/07 01:23
66です。双子素数で盛りあがってるのに水を差すようですいません。

{am-an→0} (m,n→∞)のコーシー列ですが
{an}がコーシー列である定義をε-δ論方で書いていただけないでしょうか、
また、収束する数列はコーシー列だということをε-δ論方で証明を
お願いしたいのですが、、
79132人目の素数さん:01/09/07 01:26
>単純な式ではあらわせんのかのぉ

素数は無限にあるけど、双子素数は無限にあるかどうかわからない
って、以前きいたけど。もしそうなら、そう簡単には出来ないと思う。

そもそも、とびとびでもいいから、素数だけを小さい方から拾い上げる
nの式自体、存在しないような気がする。あるのかな?

nにどんな自然数を入れても素数になる式はありますが。
80御願いします:01/09/07 01:32
a{1,2,2} b=[1,1,0}を正規直交化せよ
また、正規直交基底を求めよとゆう問題ですtt
グラムシュミットでとくのでしょうが、うまくいかないです。

グラムシュミットで
r=1の時何故、部分集合<a(1)>の正規直交基底が 1/lal倍のaになるのか
わかりません。大きさは1なのですが、直交してなくないですか?
81132人目の素数さん:01/09/07 01:35
>>78
anがコーシー列⇔∀e>0∃N∀m,n>N|am-an|<e
an→aとする。e>0をとる。an→nよりNが存在してn>N⇒|an-a|<e/2
が成立するようにとれる。このとき任意のm,n>Nに対し
|am-an|≦|am-a|+an-a|=e/2+e/2=e。∴anはコーシー列。
82はなう:01/09/07 01:36
>>79
なるほど。浅はかでした。今まだ思案中
83132人目の素数さん:01/09/07 01:40
∀ε>0∃N∀n、m>N |am-an|<ε  アルファベットは自然数、ギリシャ文字は実数としとくね

極限値をaとする。
∀ε/2>0∃N∀n、m>N  |an−a|<ε/2 |am-a|<ε/2

|an−a|<ε/2 |am-a|<ε/2 ならば、

|am-an|=|am-a+an-a|<=|am-a|+|an-a|<ε/2+ε/2=ε

<= は、「以上」ってこと。等号つきの不等号の出し方がわからん
84_man:01/09/07 01:41
74ですが、こんなのはどう?
「p,p+2が双子素数なら、24n+1の平方根mをとってmが整数となる場合m∈{p}(ただしn=正の整数)」
8579:01/09/07 01:43
となみに
>nにどんな自然数を入れても素数になる式はありますが

の例は

6+(−1)^n
86_man:01/09/07 01:43
84で、「 >>79 」と書き忘れた
87132人目の素数さん:01/09/07 01:51
>26
g:R×Q→Rに制限すれば
>g(a,b)+g(a,c)=g(a,b+c)
>g(a,c)+g(b,c)=g(a+b,c)
>g(a,1)=a

より、
g(a,0)=g(0,b)=0で
a≠0のときg(a,b)=0⇔b=0
正の整数nに対して
n g(a,b)=g(a,nb)
負の整数-nに対しても
g(a,-nb)+n g(a,b)=0
であるので一般に整数nに対して
n g(a,b)=g(a,nb)

整数m,nによってb=m/nとすれば
n g(a,b)=g(a,m)=m g(a,1)=m a
なので、bが有理数のときは
g(a,b)=ab
でなければならない。
問題はbが無理数の時なんだけど

>g(g(a,b),c)=g(a,bc)

を使ってないんだよね。cが有理数ならば
今までの結果から
g(a,bc)= g(g(a,b),c)=c g(a,b)
とすることはできてbを1などの特殊な値に近似していくことが
できるので、g(a,b)=abを示すために
gについて連続性とか必要なんじゃないかな?
88はなう:01/09/07 01:51
>>84
うーむ、n=1820でm=209(=11*19)となるようです。
8979:01/09/07 01:53
>84で、「 >>79 」と書き忘れた

と俺に振られても困る。実は整数論は苦手なんです。
誰か、いませんか?
90132人目の素数さん:01/09/07 01:58
ディレクレ関数f(x)
f(x)=1 (xは有理数) f(x)=0 (xは無理数)
は任意の有界区間[a,b]においてリーマン微分可能かどうかという
問題なのですが、やはり[a,b]における定積分になるのでしょうか?
あまりよく理解していないものですいません。
91はなう:01/09/07 01:58
そもそも、素数の個数自体曖昧なのに、双子素数を論じようとすることが無謀なのかな・・

議論ふっかけておいてすいませんが仕事なのでネマス。どういう話になっていくのか、明日の朝が楽しみだワ

ちなみに、250までの双子素数
(3,5)(5,7)(11,13)(17,19)(29,31)(41,43)(59,61)(77,79)
(101,103)(107,109)(131,133)(137,139)(149,151)(161,163)
(179,181)(191,193)(197,199)(227,229)(239,241)
9278:01/09/07 02:01
78です。
81さん、お答えいただきありがとうございました。
夏休み遊びすぎてでドキュソパー野郎になってしまいました。もともとパーだったのですが。
9384:01/09/07 02:02
>>88
84を変更〜〜 双子やめ〜〜

「24n+1の平方根mをとってmが整数となる場合、5以上の全ての素数p∈{m}(ただしn=正の整数)」
94132人目の素数さん:01/09/07 02:03
>>89
すくなくとも整数係数の多項式P(n)でnにどんな自然数をいれても
素数になるなんて都合のいいものはない。たとえば
P(n)=pが素数とすると
P(n+p)≡P(n+2p)≡P(n+3p)≡...P(n)=p≡0 (mod p)
でしかもP(n+kp)=±pとなるkはたかだか有限個しかないので
いづれは±pでないpの倍数がでてきてしまう。それは素数たりえない。
有理数が係数でそれが整数値をとるnとしてもほぼ同じ証明で
やっぱりムリ。
9578:01/09/07 02:07
83さんもお答え下さっていたのですね。見落としていてすいませんでした。
それと同時にありがとうございました。チンプンカンプンなんで
どれがどれの解答やらも、、^^;
96132人目の素数さん:01/09/07 02:15
>すくなくとも整数係数の多項式P(n)でnにどんな自然数をいれても
>素数になるなんて都合のいいものはない。

揚げ足取り的突っ込み

P(n)=5 という、恒等関数、とか。
それとも0次式は多項式って言わないのかな?

塾で教えていて、単項式と多項式を区別してあるのがうっとうしかった。
曲線ってのは直線含むし、楕円は円を含むよね。
大学以上では単項式や定数関数含めて、多項式でいいと思うが。
じゃないと、「多項式環」っていっても、多項式だけだと
和について閉じてないってことになっちゃう
97132人目の素数さん:01/09/07 02:19
>>96
しまった。一本とられた。
98132人目の素数さん:01/09/07 02:26
>>93 の命題は正しい。
この命題は
「5以上の全ての素数pに対して、 p^2を24で割ると1余る」
と同値。

(証明)
pを6で割ると1または5余る。

(6n+1)^2 = 36n^2 + 12n + 1 = 12(3n+1)n + 1
(3n+1)とnの一方は偶数なので、(6n+1)^2を24で割ると1余る。

(6n+5)^2 = 36n^2 + 60n + 25 = 12(3n+5)n + 24 + 1
(3n+5)とnの一方は偶数なので、(6n+5)^2を24で割ると1余る。

(証明終わり)
>>91
(71,73)が抜けている。
あと77,133,161は7の倍数。
10093:01/09/07 03:06
>>79
n^2-2999n+2248541 は、1460≦n≦1539 に対して素数である(ミオ)
あ。板をよごしたか?
101132人目の素数さん:01/09/07 03:08
{a(i)}が正規直交⇔|a(i)|=1&(a(i),a(j))=0 (∀i≠j)
r=1ならi≠jとなるi,jはないから直交もへったくれもない。

>a{1,2,2} b=[1,1,0}を正規直交化せよ
>また、正規直交基底を求めよとゆう問題ですtt

あとでどうせ基底にせんといかんのでc=[1,1,0]をとっておいてこれから
正規直交基底をつくる。
a'=a/|a|=(1/3,2/3,2/3)
b''=b-(b,a')a'=(1,1,0)-(1/3,2/3,2/3)=(2/3,1/3,-2/3)
b'=b''/|b''|=(2/3,1/3,-2/3)
c''=c-(c,a')a'-(c,b')b'=(1,0,0)-(1/3)(1/3,2/3,2/3)-(2/3)(2/3,1/3,-2/3)=(4/9,-4/9,2/9)
c'=c''/|c''|=(2/3,-2/3,1/3)
から{(1/3,2/3,2/3),(2/3,1/3,-2/3),(2/3,-2/3,1/3)}が求める正規直交基底。
102132人目の素数さん:01/09/07 03:10
>>101
あ、これ>>80へのレスです。スマ。
103132人目の素数さん:01/09/07 03:11
図形の問題を一題おねがいします

四面体OABCがあり辺の長さと頂角が
OA=OB=OC=1
∠AOC=∠BOC
∠AOC+∠BOC+∠AOB=180°
をみたしているとき四面体OABCの体積Vの最大値を求めよ

お願いします。
莫大な計算したら自分では1/6という数がでてきました。
>>103
直感で正四面体のとき体積最大。

1/6になる例として
∠AOC=∠BOC=∠AOB=90°のときがあるけど
角度の和が270°になって不適。
∠AOC+∠BOC+∠AOB=180° だと
体積1/6にはなり得ないと思われ。

∠AOC=2θ
ABの中点をM
△OMCでOからMCに下ろした垂線の足をH、とおけば
AB=2cos(2θ),AM=cos(θ),AC=BC=2sinθなどから
底面△ABC,高さOHが計算できて体積の式が立つ。

四面体の成立条件はたぶん45°<2θ<90°
体積V(θ)を出して微分して・・・面倒そうなのでパス。
105132人目の素数さん:01/09/07 06:56
∫sinx/sinx+cosx dxと∫cosx/sinx+cosx dx
なのですが、どう解いていいものやら、、
お願いいたします。
106132人目の素数さん:01/09/07 07:48
>>105
x-π/4=uと置換する。
∫sinx/(sinx+cosx)dx
=∫sinx/√2cos(x-π/4)dx
=(1/√2)∫sin(u+π/4)/cosudu
=(1/√2)∫{(1/√2)sinu+(1/√2)cosu}/cosudu
=(1/2)∫(1+tanu)du
=(1/2){u-log|cosu|}+C
=(1/2)x-log|cos(x-π/4)|+C
107132人目の素数さん:01/09/07 07:50
>>106
あ、最後の行まちがった。
=(1/2)x-(1/2)log|cos(x-π/4)|+C
108132人目の素数さん:01/09/07 07:53
A=∫cosx/(sinx+cosx) dx
B=∫sinx/(sinx+cosx) dx

A+B, A-B は簡単だよ
109132人目の素数さん:01/09/07 08:07
>>108
そうか。A-Bも簡単だったのね・・・
110132人目の素数さん:01/09/07 12:16
ジョルダン行列ってなんのためにあるんでしょうか?
111132人目の素数さん:01/09/07 12:45
A,B,CをA1,A2,A3とし、四面体OABCの体積をVとすれば、
V=1/6 det(OA1,OA2,OA3)
故に、V^2=1/36 det((OAi ・OAj) 1≦i,j≦3)
(行列式の積で一方を転置しておくと出る)
∠AOC=∠BOC=θとすれば,
V^2=-1/18(1+cos 2θ)cos 2θ
が行列式の容易な計算で出る。
(ヒント.1列から2列を引け)
よって、cos 2θ=-1/2のとき、Vは最大値1/(6√2)
112111:01/09/07 12:47
>>104
です。
113132人目の素数さん:01/09/07 14:35
>>47だけど、誰も教えてくれにゃいにょ?

実直交行列O(n)={X∈GL(n,R)|XX^t=1}がR^{n(n+1)/2}次元多様体って
授業で習ったけど、ユニタリー行列U(n)の場合はどうなるのかなあと思ったんです。
114132人目の素数さん:01/09/07 14:39
第一期に建設費用が5000億かかり、第二期以降に毎期100億円の純便益
が永久に生じるプロジェクトがある。社会的割引率が何%であれば
社会的余剰が0になるか?

式は0=−5000+100/r+1+100/(r+1)^2+
・・・・・・・・・100/(r+1)^r−1です。

誰かお願いします。
115132人目の素数さん:01/09/07 15:01
「それぞれ一つずつ1からmまでの数字が書かれたカードm枚をよく混ぜ、無作為にn枚抜き取る。
 抜き取ったカードの中で、一番小さな数字の期待値を求めよ」

という問題です。よろしくお願いします。
116132人目の素数さん:01/09/07 15:43
>>47
一般の体だと、返答に困る。位相が入ってないかも知れないし。
Z/pZも体だけど、微分なんて考えられない。
Qも、完備でないから扱いにくい。
だからRで、ってことだろうけど。で、CとR^2は、位相同型だから

>また、Cの線形空間による局所座標によって定義された可微分多様体は
>Rの局所座標系によって定義された可微分多様体になるのでしょうか?

は、多分直感的にはそうだと思うけど。ただ複素多様体って、恐ろしく
難しくて、そういいきって良いかどうか自信がない。

n次元複素多様体なら2n次元実多様体は言えると思うけど、
逆は言えなそう。実数2変数の2つの実関数が微分可能でも、
1複素数変数の1つの複素関数と見なしたとき複素関数として
微分可能とはかぎらないし。

複素n次行列全体はC^(n^2)のベクトル空間と同型。

ユニタリであるためには、n^2個の式が成り立たなくてはならないが、
n^2−n^2=0で、そんなはずないから、n^2個の中に独立でな
いものが混じってるんだろうな。で、たぶん実数C^同様、
C^{n(n+1)/2}でいいんじゃないかな。
で、それは多分、R^n(n+1)と同相

保証は出来ないが。


>114  高校教科書 数列の和 等比数列  を参照
117132人目の素数さん:01/09/07 15:54
>115
Σ(k=1〜n−m+1) k*m-kCn/mCn
118114:01/09/07 16:00
>>116
分母の最後が(r+1)のn−1乗の間違いです。

ていうか高校の数列では解くことはできません。
関数電卓かExcelで誰か答え出していただけませんでしょうか。
当方、使い方がわからないもんで・・・・鬱死。
119132人目の素数さん:01/09/07 16:08
>>116
有難う御座います。
実数の場合はn次対称行列VがR^{n(n+1)/2}に同相であり、
実n次正定値対称行列LがVの開集合であることを用いて、
陰関数定理を用いた公式を使うと授業で習いました。
120132人目の素数さん:01/09/07 16:16
[1]X,Yは独立な確率変数で、どちらも標準正規分布に従うものとする。
すなわち確率X,Yはともに確率密度関数

   f(x) = exp(-x^2/2)/√(2π)

をもつものとする。
V = X+2Y, W = 3X+Y とするとき、
(V,W)の同時密度関数、
Vの確率密度関数および平均と分散を教えて下さい。

それと


[2]Cを中心をOとする半径1の定円周とし、AをC上の定点とする。

(1)ランダムな点PをC上一様に分布するようにとる。
Sを三角形OPAの面積とする。Sの分布関数と確率密度関数および平均を求めよ。
(2)ランダムな点QをCの内部に一様に分布するようにとる。
Sを三角形OQAの面積とする。Sの分布関数と確率密度関数および平均を求めよ。

できれば途中の式もお願いします。
121115:01/09/07 16:19
>>117
もっと簡単になりませんか?
Σを使わずに表せるようなんですけれども・・・
122132人目の素数さん:01/09/07 16:33
a^n/n!の極限の求め方の過程を教えて下さい。
123123:01/09/07 21:03
V={{Xn}n=1 to ∞|Xnは実数の要素でlim(n→∞)Xnが存在する}
{Xn}+{Yn}:={Xn+Yn} a{Xn}:={aXn}(aは実数)
で定義される実数上のヴェクトル空間Vがどうして有限次元でないのでしょうか
124132人目の素数さん:01/09/07 21:07
だれか>>103に答えてやれ.104以外で
125132人目の素数さん:01/09/07 21:33
>>124 同意。
ただし、受験数学→趣味によって無視してよし。応えても良し。
ただし^2、といえどもヒント有り???(ドクター論公表(提出)への)

※ すまん。どう考えてもドクターには及ぶ命題(ぷっ!)ではなさそう
126114:01/09/07 21:48
0=-5000 + 100/1+r + 100/(1+r)^2 +........+100/(1+r)^n-1

のr求めてください。宜しくお願いします。
127132人目の素数さん:01/09/07 22:03
>>126
たぶんnの関数として表すのは無理なんじゃないの?
128114:01/09/07 22:09
>>127
答えが0.01or0.02or0.03or0.04or0.05のうちのどれかなんです。
129132人目の素数さん:01/09/07 22:14
>>114
114だけじゃないんだが、問いかけがまずいな。
どういう人がどういう気持ちのとき「応えてくれる」ということ、に対する想像力が弱い。

1.114について受け手の興味(それぞれの興味)があればOK
2.・・・・(くどいのでやめる
130132人目の素数さん:01/09/07 22:24
>>128
なんだ、そういう事か。問題文と照らし合わせてようやく理解できた。
上のほうで誰か言っていたけど、そんなのは高校数学の範囲内。
しかも十分に手計算で可能。

まぁでも答えるわけにはいかないので、ヒント。
お前の使ってるnって何? 問題文をよーく読め。
131114:01/09/07 22:30
>>130
まったくわかりません・・・N(年数)→∞ってこと?
マジでわからない・・・・・もっとヒントください(藁
132132人目の素数さん:01/09/07 22:33
(´д`)
133132人目の素数さん:01/09/07 22:45
>>131
そういう事。
「純便益が永久に生じる」と書いてある以上、勝手にN年後までに限定してはいけない。

とりあえず手元に数学の教科書ある?
それで等比数列について載ってるページを読んでみてくださいな。
あと、等比数列の和かな?

もしその内容が理解できていないのであれば、理解できるように
教科書を読み直せ。話が進まん。

自分は理解してるぞ!って場合は、その事を式に当てはめてください。
134114:01/09/07 23:02
>>133
無理だ。わからん(藁。チャートを引っ張ってみてきたが無理。
つーかN年後じゃ駄目なのか?

 ∞
 狽P00/(r+1)^@を求めるのか?
@=1

つーか無理だ答え教えてくれ。頼む(藁。
135132人目の素数さん:01/09/07 23:12
>>134
なんでだ?そこまでわかってんのに?
>無理だ。わからん(藁。チャートを引っ張ってみてきたが無理。

>まったくわかりません・・・N(年数)→∞ってこと?

> ∞
> 狽P00/(r+1)^@を求めるのか?
>@=1

引っ張ってみてきたチャートには
 N
 狽P00/(r+1)^@の公式もN(年数)→∞の求め方ものってるだろ?
@=1
136114:01/09/07 23:24
>>135
駄目だ全然理解できない、漏れには・・・。
すでに大学3年の漏れにこの問題はキツすぎるよ(藁。

頼むから答えだけ教えてくれ。おねがいしやす。
137132人目の素数さん:01/09/07 23:31
>>47
ユニタリー群 U(n) は 実n^2次元リー群です。
たとえば、U(1) は S^1 (円周)と同相ですから、
複素多様体ではありません。
138132人目の素数さん:01/09/07 23:39
>>137
>U(1) は S^1 (円周)と同相ですから、

成る程、そうですね。
有難う御座います!!
139132人目の素数さん:01/09/07 23:42
 f(xy) = xf(y) + yf(x)
を満たす関数で、「恒等的に零」以外のものはあるんでしょうか?
140114:01/09/07 23:48
誰か答え教えてくれ・・・・
141132人目の素数さん:01/09/07 23:50
0.02
142114:01/09/07 23:56
マジっすか・・・・単位落としたな(藁
>>139
R−>Rの関数ならaxlog(x)がある。(aは定数)
axlog|x|だった。(f(0)=0)
145132人目の素数さん:01/09/08 00:24
>>139
微分可能として、yで微分して xf'(xy)=xf'(y)+f(x)
y=1 として xf'(x)=xf'(1)+f(x)。f'(1)=a とおいて
f'(x)/x-f(x)/x^2=a/x すなわち (f(x)/x)'=a/x で
f(x)=axlog|x|+C ここで f(1)=0 だから C=0
f(x)=axlog|x| (x≠0), f(0)=0 とすればOK。
146132人目の素数さん:01/09/08 00:45
>>110

Jordan標準形ってのは、対角化の進化系だ。
147132人目の素数さん:01/09/08 00:50
>>103
>>124

別に>>104の方針でも
AM=cos2θ=t,∠AOC+∠AOM=90°
などを利用していけば√が外れて簡単になるよ

(△ABC)^2=t^2(2-2t-t^2)
(OH)^2=(1-2t^2)/(2-2t-t^2)
9V^2=(△ABC)^2*(OH)^2=t^2(1-2t^2)=-2(t^2-(1/4))^2+(1/8)≧1/8

t=-1/2,θ=30°,∠AOC=60°,つまりO-ABCが正四面体のとき
体積の最大値=1/√72=1/(6√2)

>>111と同じ答えになって一安心
148139です:01/09/08 01:05
>>143-145
なるほど。
ありがとうございます!
>>116
>Z/pZも体だけど、微分なんて考えられない。

標数pの体でも微分体って考えられなかったっけ?
150おじさん@素人:01/09/08 01:37
>>120
[2](2) S≦x となる点を円内に書いてみると、OA を底辺と見て、
高さが 2x 以内の点だから、(この面積/円の面積) が分布関数。
F(x)={2arcsin(2x)+4x√(1-4x^2)}/π で、
F'(x)=(8/π)√(1-4x^2)、平均 4/(3π)
151150:01/09/08 01:50
平均は 2/(3π) ですね、失礼。お詫びに・・・
[2](1) S≦x となる点を円周上に書いてみると OA を底辺と見て、
高さが 2x 以内の点だから、(この長さ/円周) が分布関数。
F(x)=(2/π)arcsin(2x) で、F'(x)=4/(π√(1-4x^2))、平均 1/π
152116:01/09/08 02:50
>標数pの体でも微分体って考えられなかったっけ?

もちろん、正式なり有理式なりの形式的微分は考えられるけど、
多様体論で言うところの微分は無理じゃないかという意味。
それとも出来るのかな?専門じゃないから分からない。
>>149
Z/pZも微分できるんですか?
154132人目の素数さん:01/09/08 03:04
>149
微分の意味が違う?
Fをp元体とすると、Fを微分体と見るってことは
加法群Fの自己準同型でライプニッツルールを満たすものを
与えるってこと?
でもFの各元がF内でp乗元(a^p=a)であることから、
この意味の微分は0写像だけになるんでは?
もとの問題の方は、むしろ可換環 Map(F,F) の適当な
部分環に微分環の構造を入れよ、ってことかな?
でもこのときも f^p=f だから0写像しかないんでは?
>>152
>もちろん、正式なり有理式なりの形式的微分は考えられるけど、
>多様体論で言うところの微分は無理じゃないかという意味。

う〜ん、だんだん難しくなってきました。
少なくとも任意の体に関して位相多様体を定義できると考えても良いのでしょうか?
156116:01/09/08 03:18
>ユニタリー群 U(n) は 実n^2次元リー群です。
>たとえば、U(1) は S^1 (円周)と同相ですから、
>複素多様体ではありません。

いい加減なこと書いてすまん。
よくよく考えたらF(z)=zの共役複素数 って、正則関数にならないから、そうだよね。

Aの転置行列・A=単位行列 ということなら、複素多様体だろうけど(多分)
157116=152:01/09/08 03:31
>>155

>もちろん、正式なり有理式なりの形式的微分は考えられるけど
もちろん、正式じゃなくて整式の誤植

これは全然難しくなくて、
体、可換環でもいいや、その多項式環を考える
で、
f(x)=A0X^0+A1x^1+A2x^2+A3x^3+A4x^4+・・+Anx^n
の「微分」を
A1x^0+2*A2x^1+3*A3x^2+4*A4x^3+・・+n*Anx^(n-1)
というように形式的に定義すると言うこと。極限概念は使わない。
代数学でやります。


他の人の議論は私には分からない
>>157
有難う御座います。
私は代数はあまり得意ではないのですが、Z/pZを値域とする連続関数fを
定義できるように開集合を取る事が出来るのでしょうか?
>152
もともと>47の質問が、
R^nと同相な局所座標を持ってきて可微分多様体を構成してるのを
Rを一般の体にして多様体と同じような物を構成ってことだったから
その時点で多様体…かなぁとも思って、一般の体だったら微分自体を
変えなければならんねと単純に思っただけで…
僕も聞いたこと無いけど変な物が出来上がりそうだなぁと思ってさ

駄レスsage
160116=152 :01/09/08 03:44
>私は代数はあまり得意ではないのですが、Z/pZを値域とする連続関数fを
>定義できるように開集合を取る事が出来るのでしょうか?

私には分かりません。距離なり位相なり入れないと、連続関数を定義できないが、どうなんでしょう?
>>156
複素数の座標で定義した多様体でも陰関数定理が成り立てば
U(n)は複素多様体になりそうな感じです。

>>157
申し訳ありません、開集合はどんな集合でも適当に取れますね。
Z/pZからZ/pZへのC^∞関数を考える事も出来ますよね。
>158
距離も一致してる時以外は1みたいな距離入れて
連続関数を定義しよう
位相も離散
163まこ☆:01/09/08 03:55
いきなり、、すいません
この問題解ける方、いらっしゃいます??
どうしても答えが知りたいんです。

【問題】

「重りとバネばかり」
パチンコ玉が六個あります。そのうち五個は同じ重さですが、一個だけ目方が少し
違うことがわかっています。(重いか軽いかは不明)
ばねばかりを三回つかって、その問題の一個の玉を当ててください。
ただし、標準の玉は何グラム、一個の玉は何グラムまで答えてください。

お願いします。
>>163
問題文を正確に写しましょうね
165まこ☆:01/09/08 04:07
>>164

今、何度も読み返しましたが、正確です。
大学の講義中に配られた用紙を写しました。
講師は、『6個は難しいから、4個から考えなさい!』との事。
考えかた判んないんだから、4個も6個も同じだぁ〜A^^;
>>163-165
三回量った結果をA、B、Cグラムとして
標準の玉と一個の玉の重さをそれぞれA、B、Cを用いて表せ
ってことでしょう?
167童貞女子高生:01/09/08 05:07
だれかこの不貞積分のとき方おしえて。
∫1/(1+sinx)dx

お願いします
168まこ☆:01/09/08 05:19
>>166

はい! そうです。

一個だけ目方が少し違う玉はどの玉か?
標準の玉と一個の玉のそれぞれの重さは?

三回で、この2点を答えるんです。
169おじさん@素人:01/09/08 05:43
>>163
普通、特別な玉の重さをそれぞれ p,q とする。
玉を a,b,c,d,e,f で表す。
1回目 a,b,c,d の重さを x とする。
2回目 c,d,e の重さを y とする。
x:y=4:3 のとき、f が特別。
3回目 f の重さを z とすると、p=x/4, q=z
そうでない時、3回目 b,c の重さを z とする。
(多分この場合分けがポイント)
y:z=3:2 のとき、a が特別。p=z/2, q=x-y
x:z=2:1 のとき、e が特別。p=z/2, q=y-z
x+z=2y のとき、c が特別。p=y-z, q=2z-y
3(x-z)=2y のとき、b が特別。p=(x-z)/2, q=x-y
z=2(x-y) のとき、d が特別。p=x-y, q=y-z
#どれが特別な玉かに応じた x,y,z の表を作った。
170132人目の素数さん:01/09/08 05:56
>>29
パラドックスの解決法の話では?
171170:01/09/08 05:57
誤爆ですごめん。
172おじさん@素人 :01/09/08 05:57
>>167
∫1/(1+sinx)dx
=∫(1-sinx)/{(1+sinx)(1-sinx)}dx
=∫(1-sinx)/(cosx)^2dx=tanx-1/cosx
173縞栗鼠(シマリス)の親方:01/09/08 06:17
大検、大学受験の学習相談やってます。
全教科対応です。
お気軽にご質問ください。

「縞栗鼠(シマリス)の親方」まで
http://www.tkcity.com/renbbs/1/user/daiken.html
174132人目の素数さん:01/09/08 06:34
>>169
3回目に何を量るか場合分けをする判断基準がよくわからないので解説願います。
「そうでない時」がどの否定かわかりません。
4回目を量っているように読めてしまいました。
175132人目の素数さん:01/09/08 07:01

モナドってなんでせうか。
群論の何たららしいのですが。
176132人目の素数さん:01/09/08 08:19
2ちゃんでIQ高い板ってここだよね??
177132人目の素数さん:01/09/08 08:38
大学受験板・就職板・資格全般板あたりのほうがIQ高そうなイメージあるけどね
>>174
x:y=4:3のときとそうでないとき。
>>178
ありがとうございます。

1、2回目で得たx、yに対して
x:y=4:3のとき → 3回目にf=zを量る → p=x/4, q=z
x:y≠4:3のとき → 3回目にb+c=zを量る → 略

・・・ということだったんですね。すっきりしました。
180132人目の素数さん:01/09/08 10:01
>>175
いま数学辞典でしらべたらカテゴリー論のことばみたい。
定義ながい。
181132人目の素数さん:01/09/08 13:21
すいません。
証明問題を一題教えてください

空間にn個の点A(1).A(2).....A(n)が次の条件をみたすように配置されている
(条件)
どの2点を結ぶ線分上にも他の点がなくてこれらすべての線分には向きがつけられている

このとき適当な一点A(k)(k=1.2.3.....nのいずれか)をとれば
A(k)を起点として指定された向きに進んで
A(k)以外のn-1個の点に直接にまたは一点を経て間接に到達できる事を示せ

漸化式の問題とおもって漸化式を立てる方針でいったのですが失敗してドツボに嵌りました
どなたかよろしくお願いします
182まさ_r:01/09/08 14:51
僕もひとつ質問しても良いですか?
「今日が日曜日なら、明日は水曜日である」というのが真か偽か?という問題なんですけど…
一見うそに決まってんじゃんて感じなんですけど述語論理を使って証明しなくちゃいけなくて…
どなたかわかる方がいたら教えてください。
183おじさん@素人:01/09/08 15:07
>>181
向きに従って矢印を書く。2点の間にはどちらかを向いた矢印がある。
出て行く数が最大 m になるものの一つを A(k) とする。
もし、うまく行けない点 A(i) があるとすると、
その点からは少なくとも A(k) から直接行ける点 m 個と、A(k) に
向かって、合計 m+1 個の矢印があるから、m が最大である事に反する。
(グラフ理論の言葉でいうとトーナメントの隣接行列の2乗を考える)
184132人目の素数さん:01/09/08 15:12
>>181
これ既出なんだけど過去ログさがしてもみつかんなかった。
nに関する帰納法、n=2であきらか。n未満で成立するとして
A(1).A(2).....A(n−1)にかんして帰納法を適用してそれに関して
題意を満たす点A(k)をとる。A(k)→A(n)もしくはA(k)→A(l)→A(n)
なら終わり。よってA(n)→A(k)かつ任意のA(k)→A(l)なるlについて
A(n)→A(l)と仮定する。このときはA(n)が題意をみたす点。
185132人目の素数さん:01/09/08 15:18
>>183
ありゃ、かぶった。スマ。しかもこれ帰納法つかう既出の解法よりイイ!
186132人目の素数さん:01/09/08 17:21
>>115
これまともに計算すると死んでしまう問題だね。
たとえばm=4,n=2だと全事象が
 12 1
 13 1
 14 1
 23 2
 24 2
 34 3
でそれぞれの事象が1/6なので期待値は(1+1+1+2+2+3)/6
分母のC[m,n]はいいとして分母だけどこれは3個の●と2個の○をならべた表
とそのいちばん左の●をぬいた表をつくる。
○○●●● ○○●● 34
○●○●● ○○●● 34
○●●○● ○●○● 24
○●●●○ ○●●○ 23
●○○●● ○○●● 34
●○●○● ○●○● 24
●○●●○ ○●●○ 23
●●○○● ●○○● 14
●●○●○ ●○●○ 13
●●●○○ ●●○○ 12
いちばん右の数字は●の位置。みてわかるとうり12、13、14が1回づつ
23、24が2回づつ、34が3回でてきているのでこれがさきほど
もとめたかった分子。つまり分子はC[m+1,n+1]になる。よって
もとめる期待値はC[m+1,n+1]/C[m,n]。
もちろん記述でこんな解答つくると×。ちゃんとした解答は自作してみるべし。
187132人目の素数さん:01/09/08 17:40
1000000000を越えると具体的にどうなってしまうの?
http://kaba.2ch.net/test/read.cgi?bbs=news&key=999927113&ls=50
188132人目の素数さん:01/09/08 20:29
整数問題だとは思うのですが
うまくいきません。。
どなたかお願いします

正の定数a.bに対して
不等式4m<n^2<4m+(a/√m)+(b/m)を考え次の問いに答えよ

(1)m>0かつm.nともに整数であってこの不等式を満たすようなm.nの組は
有限個であることを示せ
(2)a=8.b=9.m≧9であるときは
上の不等式を満たす整数m.nの組はn^2=4m+1を満たすことを示せ
(3)(2)の場合のm.nの組みのうちnが最も大きいものを求めよ
189120:01/09/08 21:05
>>150
ありがとうございます。


[1]X,Yは独立な確率変数で、どちらも標準正規分布に従うものとする。
すなわち確率X,Yはともに確率密度関数

   f(x) = exp(-x^2/2)/√(2π)

をもつものとする。
V = X+2Y, W = 3X+Y とするとき、
(V,W)の同時密度関数、
Vの確率密度関数および平均と分散を教えて下さい。

こちらを誰かお願いします。
190132人目の素数さん:01/09/08 21:20
次の問題の証明について宜しければ誰か教えて下さい。

n,m,kは0以上の整数とする。
C[n,m] は 2を法とする二進係数。すなわち0か1。
このとき C[n+k,2m-1]*C[n,m] = 0 (mod2)
191132人目の素数さん:01/09/08 21:29
二進係数ってなに?
192132人目の素数さん:01/09/08 21:47
>>182

日曜日には偽、月曜〜土曜は真

PならばQ  と  「Pでない」または「Q」、  は全く同じ
「PならばQ」は Pが偽で有れば、Qの内容によらず真。
193132人目の素数さん:01/09/08 22:41
次の問題をご教授いただけないでしょうか。

三角形ABCの内心をIとして
AI(↓)=x・AB(↓)+y・AC(↓)と表す(ただし↓はベクトル)
(1)
BC=a CA=b AB=cとして
x=(b/a+b+c) y=(c/a+b+c)であることを示せ
(2)
三角形ABCの形状を変化させるのに
点(x.y)が動く範囲を求めよ。
194190:01/09/08 22:55
>191

すいません。間違えました。二進係数ではなく、二項係数です。
195132人目の素数さん:01/09/08 23:07
さくらスレ11の↓の問題だけど

2つの半直線OX.OYをとり
OY上の点PからOXにおろした垂線の足をQとする。
Pを通りOXに平行な直線上に点Sを、線分OSと線分PQが交わるようにとり
OSとPQの交点をRとする
線分OPの長さが1、線分RSの長さが2を保ちながら
∠XOYを鋭角の範囲で変化させたときの線分QRの長さの最大値を求めよ


∠XOY=αと置く。
Oを原点、OXをx軸の正の方向に重ねた座標平面を取ると、

>P(cosα,sinα) S(cosα+2,sinα)より、
>OSを結んだ直線はy=(sinα/(cosα+2))x
>Rのx座標はcosαなので、R(cosα,(sinαcosα)/(cosα+2))
>よってRS=(sinαcosα)/(cosα+2)なので、
>あとはこの最大値を調べる。

なんか違わない?
RSの最大値もとめてどうしたいのかが俺にはみえてこないんだけど。。
あとSの座標が(cosα+2 .sinα)というのも出てこない、、
俺ぼけちゃったかな(汗
196132人目の素数さん:01/09/08 23:13
スレあげるのも迷惑だから、ここでお聞きします。

質問:
メーテルと文系と今井弘一
は誰が一番お馬鹿なの?
解に至る過程も述べよ。(10点)
197はなう:01/09/08 23:41
ちょっと前のを掘るか。

>>188

(1)は、a/√mもb/mもm→∞になるにつれ1よりは小さくなる。で、もちろんn^2は整数だから、<で挟まれている以上、あり得なくなるって事。

(2)問題の条件の時
(a/√m)+(b/m)<4である、
つまり、n^2≡1,2,3(mod4)であるが
nが奇数の時
n≡1,3(mod4)
n^2≡1(mod4)

nが偶数の時
n≡0,2(mod4)
n^2≡0(mod4)

ってことで

n^2≡1,2、3のうち1しかあり得ないので
n^2=4m+1

(3)不等式の右辺が4m+1より大きくならなくてはならず、そのためには
(8/√m)+(9/m) > 1 (ただしmは正の整数)
これを解いて 0 < m < 81

で、n^2は80*4の320以下になるはずで、それに一番近い平方数が
17*17=289 その時m=72
ってことで、(n,m)=(17,72)
198名無しさん@XEmacs:01/09/08 23:49
ある標本から、ある人が正解を取り出した正当率をαとすると、
その
z-scoreはどのような結果になりますか?
どうか教えてください。
>>196

全て極大元だよ。(藁
200132人目の素数さん:01/09/09 00:03
>>190
なりたたないじゃん。
m=2,n=3,k=4のとき
C[7,3]=15,C[3,2]=3
201132人目の素数さん:01/09/09 00:11
>>195
ほんとだね。めちゃくちゃじゃん。質問者はこれでなっとくしたの?
202195:01/09/09 00:13
>>201
むちゃくちゃだよね、、やっぱ。
質問者はどうなったか知らない。
方針はいいとは思うけどね
203私文サル:01/09/09 00:16
X∈[0,Y−Z]の意味するところは何ですか?
204132人目の素数さん:01/09/09 00:20
0≦X≦Y−Z
205私文サル:01/09/09 00:26
>)204
ありがとうございます。日本史で経済学部にはいったもので数学がどうも苦手です。
失礼しました。
206縞栗鼠(シマリス)の親方:01/09/09 00:34
大検、大学受験の学習相談やってます。
全教科対応です。
お気軽にご質問ください。

「縞栗鼠(シマリス)の親方」まで
http://www.tkcity.com/renbbs/1/user/daiken.html
207132人目の素数さん:01/09/09 00:48
>>193
以下[XY]でXYベクトルをあらわすことにする。
(1)
((面積比の公式がつかえると楽なんだけど。使えそうにないので))
[BI]=(1-x-y)[BA]+y[BC]
BIとACの交点をMとすると[BM]=k(1-x-y)[BA]+ky[BC]。このとき
(1-x-y):y
=k(1-x-y):ky
=CM:AM      (∵内分点の公式)
=((△BMCの面積)):((△BMAの面積))
=BC:BA
=a:c
同様にして1-x-y:y=a:c。
∴1-x-y:x:y=a:b:c。
以下略。
(2)
b,cを固定してaを動かす。軌跡は
x=b/(a+b+c),y=c/(a+b+c),a>0。
⇔cx=by,a=(b+c)(x+y)/(1-x-y)>0。
⇔cx=by,x+y>1
だから直線cx=byのx+y>1、つまりx+y=1の右上の部分を動く。
つぎにb,cを動かすと結局第一象現のx+y>1の上側がもとめるきせき。境界は含まない。
208132人目の素数さん:01/09/09 00:48
>>196哲学板でめーテル警報発令中!
209まさ_r:01/09/09 00:51
>>192
ありがとうございます。
でもこれって命題論理というやつじゃないのでしょうか?
それとも命題論理とはまったく違うものなのでしょうか?
210132人目の素数さん:01/09/09 00:54
>>207
ごめん訂正。
x=b/(a+b+c),y=c/(a+b+c),a>0。
⇔cx=by,a=(b+c)(x+y)/(1-x-y)>0。
⇔cx=by,0<x+y<1
だから直線cx=byの0<x+y<1、つまり原点からx+y=1との交点までの部分を動く。
つぎにb,cを動かすと結局第一象現のx+y<1の上側がもとめるきせき。境界は含まない。
211132人目の素数さん:01/09/09 01:00
>>210
これもうそ。ただしくは
つぎにb,cを動かすと結局、第一象現のx+y=1の下側の三角形
がもとめるきせき。境界は含まない。
212名無しさん@XEmacs:01/09/09 01:08
ある標本から、ある人が正解を取り出した正当率をαとすると、
その
z-scoreはどのような結果になりますか?
どうか教えてください。
z-scoreとはz変換、標準化とのことだそうです。
213132人目の素数さん:01/09/09 02:11
1+1=2を証明せよ
>>213

今井と一緒に心中してくれ
215132人目の素数さん:01/09/09 02:32
みんな氏ねばいいんじゃない?
あいこなんだし…

質問は何故、こいつらあほなんですか?
216213:01/09/09 02:41
答えてくれなきゃスレたてちゃうもんねーーー
>>122
2|a|より大きい整数を一つとりそれをmとする。
m<nとすると
|a^n/n!|≦|a^m/m!|×(1/2)^(n−m)
n−>∞とすると(1/2)^(n−m)−>0なので
lim(a^n/n!)=0。
218132人目の素数さん:01/09/09 03:33
ごめんなさい。
他のとこに書いてあったんですけど、分からないもので・・・

(1)方程式:
    x^4+−6x^3+11x^2−6x=−(−x^3+6x^2−11x+6)a
    (a:定数)
   の解が3つ存在するようなaの値を求めよ。

(2)方程式:
    x^6+2ax^5+3x^4+ax^3+bx^2+cx+abc=0  (a,b,cは定数)
   の解が4つ存在するようなaの条件をb、cを用いて表せ。

(1)は解答が判明したんですが、(2)が分かりません。
お願いします。
219218と同じ板の人:01/09/09 03:39
いちおう、数学板住人さんによる(1)の解答も聞いてみたいなあ。
たぶん同じだと思うけど。
>>123
a(n)=(x(n,0),x(n,1),x(n,2),...)
x(n,m)=0(n≠m)
x(n,m)=1(n=m)
とすると{a(n)|n∈Z,0≦n}は一次独立。
221132人目の素数さん:01/09/09 04:28
>>218
(1) (x-a)(x-1)(x-2)(x-3)=0
問題の意味は、重複度を度外視して
異なる解の個数を3とする、ということか? ならば a=1,2,3
(2) f(x)=x^6+2ax^5+3x^4+ax^3+bx^2+cx+abc
と f'(x) が (2次の係数が0でない)2次式を最大公約数として持てば良い
互除法を使えば良いが、計算が大変と思われる。
>218
面倒なのでやる気が無いが、どちらも解が重複しているので
2次方程式の時と同様に

判別式=0
をゴリゴリ計算して解く

重解がある場合は(x−α)^2(x−β)(x−γ)の形に因数分解できるはずだから
左辺をxで微分した式=0
という方程式の解にもなってる筈で連立する

(2)の方は2つ重解がある場合と、3重解がある場合とあるのが
3重解がある場合はもう一回微分したものを連立させることができるので
それなりに簡単になるんではないかと
223132人目の素数さん:01/09/09 04:35
>>221
ありがとうございます
ちなみに大学受験問題なんですけど、
(2)はどの程度のレベルでしょう?
224132人目の素数さん:01/09/09 04:46
>>218の(1)は良問ですか?
センターレベルらしいんですが・・・
225223:01/09/09 04:46
あ、>>222さんもありがとうございます。たいへん参考になりました。
226132人目の素数さん:01/09/09 04:46
レベル的には高校生なんだが、
数学科に来る人は計算が苦手なのが多いからな(w
227132人目の素数さん:01/09/09 04:50
(2)はかなりきつくないか?
阪大レベルだと思うがどうよ?
228数理科学:01/09/09 05:06
>>218 の(2)
スツルムの定理を使ったら?
でも面倒か・・・
229132人目の素数さん:01/09/09 05:19
(2)って
解は実数の範囲?複素数の範囲?
230132人目の素数さん:01/09/09 05:34
ロト6で23-24とかいうような連番が出る確率って
どのくらいですか?

下らないが役に立つかも。
231名無しさん@XEmacs:01/09/09 08:59
>>212 はどうですか?
232質問です:01/09/09 09:32
∫{0→π/2}log(cosθ)dθ = -(πlog2)/2
を示して下さい。お願いします。
統計は、数学科の周りで知ってる人ほとんどいないから
言葉を定義しないことには答えてもらうのは難しいよ>231
234_:01/09/09 10:34
>>103
> 四面体OABCがあり辺の長さと頂角が
> OA=OB=OC=1
> ∠AOC=∠BOC
> ∠AOC+∠BOC+∠AOB=180°
> をみたしているとき四面体OABCの体積Vの最大値を求めよ
>>104 >>111
ボクは中学生ですが、このもんだいがとけました
ボクは中学生なので、三角関数、とか、微分、とか、よくわかりません
あと、ベクトル、もつかいません。ベクトルをつかいすぎてベクトルで全部とけますと言うとイッテヨシとみんなから言われるし、ボクも言ったと思います。だからベクトルはきらいになりました。

あと、こたえは、
Oが原点で、ABCは半径1の球の表面じょうにあるので、
左右・奥行き・高さのxyz軸として、Cはx軸上の(1,0,0)とします。
AとBは似ているので「y=0のx-z平面」に対して対称の位置にすると解りやすいので、そうします。
その位置は、奥行きy上にpと-pということして、高さzはqとしておきます。
すると左右のxは、原点からの長さが1だから、√(1-p^2-q^2)です。
それで、OとAとBとCの位置は、
O(0,0,0)
A(√(1-p^2-q^2), p,q)
B(√(1-p^2-q^2),-p,q)
C(1,0,0)
<つづく>
235_:01/09/09 10:36
となりました。AとBの位置の書き方はこれでは美しくない気がして三角にしたい気がしますが、関係ないことが後でわかるし、ボクは三角関数が知りません。
あと、この板は、絵が描けないので残念ですが、字で描きます。でも、頭の中で絵を描くと、すごく解ります。
あと、求める体積Vの形は、「y=0のx-z平面」を底面として、奥と手前に同じ形ですし、
底面、下辺OC=1高さqの三角形で面積「q/2」、掛ける、
高さp割る3、手前と奥に2つあるので2倍して、
体積V=pq/6
です。
この体積V=pq/6の最大値を求めるのが「答え」で、ここまでは値をどう表現してもよいのですが、ボクはあとあと考えてこうしました。

pとqは、問題の条件により、関係しています。pによってqが、qによってpが、決まります。
この関係を満たして、Vの最大値を言うのが答えです
で、四面体OABCの展開図を考えます。
展開図という言葉は、大人は忘れてしまいますが、単純に立体表面を紙細工で作るとき、切り開いたもののことです。
えー、この場合はABCの面は捨てて、
三角コーンOABCのOBをはさみで切って拡げます。

展開図を高さの無いx-y面上に(頭の中で)拡げます。
で、Oは原点で、ABCはO中心半径1の円上にあります
左右x軸上のO(0,0,0)とC(1,0,0)は動かさずそのままにすると、
<つづく>
236_:01/09/09 10:37
右上(x,yが正)の円周上にA(xの値が√(1-p^2-q^2)),
右下(xが正,yが負)の円周上にB(xの値がAと同じ)があるわけですが、
1つの三角形の面OCBの頂点Bが、左上のどのへんにくるかがポイントです。

展開図を想ったのは、∠AOC+∠BOC+∠AOB=180°だからです。
空間上のBに対して紙細工開きのx-y平面上のB’(√(1-p^2-q^2),√(p^2+q^2),0)
※ この座標のyの値は、B’が半径1の円周上にあることから。
一方、左上円周上に来るB”は∠AOC+∠BOC+∠AOB=180°だから
原点OについてB’と対称なのでB”(-√(1-p^2-q^2),-√(p^2+q^2),0)
このB”とy軸に対称なAとの距離は最初にpとqを決めた決め方から
B”A=2p  それで
B”A = 2p=-2√(1-p^2-q^2)
これでpとqの関係がわかりました
これを 体積V=pq/6 に代入します
ルートの微分を知らないのでVの2乗で考えます
V^2= (p^2)(q^2)/36
  = (p^2)(1-2p^2)/36
  = -(p~2-1/4)~2/72 + 1/(72*16)
だからV^2の最大値は 1/(72*16) それで
Vの最大値は 1/(6√2)

長いのでだれも読みませんが、最初に書いた人はよみます

[終わり]
237132人目の素数さん:01/09/09 10:43
>>232
複素関数論の演習書を見よう。多分のってる。
∫log(sinx)dx とか、積分区間が違うものかも。
>>234
条件 ∠AOC=∠BOC を落しても正四面体の時が最大である事を証明できる?
238212:01/09/09 10:43
そうですね。言葉が足りなかったと思います。
(自分がどんな頑張っても理かいできないため、知らず知らず言葉
が足りなくなってしまうのです)
書き直すと
信号分布(平均0)とノイズ分布(平均β)
というのがありまして、これが正規分布であるとします。
更に分散Vも等しいとします。

ここで、二肢強制選択法(2AFC)を用いたところ、観測者の正当率がαであったとする
(ここで、二肢強制選択法(2AFC)とは、名にかといいますと、
####
観測期間が2つあり、そのうち片方に信号だけが
ふくまれており、もう片方がノイズだけが含まれているとします。
このときある被験者に、どちらの観測期間に信号が含まれている
かを答えさせ、その結果の正当率αを導き出します
####
)

僕が与えられた問題(レポート)は
"この場合に、この正当率から正当率のz-scoreを求めよ。"
というものです。

ここで、標 準 得 点 (Z-Score)について自分が調べた知識を書きますと、
観察頻度 - 期待頻度
z = --------------------
標準偏差
だそうです。

僕の考えでは、
ここで、観察頻度は正当率αになるのかな?
期待頻度は?
標準偏差は?
これがわからないので、とまっています。
まず、被験者に何回2AFC実験が行われたのか、の記述が問題には無いし。

なにか分かりましたらお願いします。
>>234

けっきょく(体積)^2を平方完成して最大値を出すのなら既出の解法と同じ。
空間座標を持ち出してしまっては中学生な立場のネタとしてもいまいちのような・・・

>>104>>147は見かけ上三角関数を使ってるけど
三平方の定理と相似だけで(体積)^2を出してるよ。
240132人目の素数さん:01/09/09 11:25
age
241132人目の素数さん:01/09/09 11:51
>>238
選択の基準がないと期待頻度が出せないような・・・
当たる確率 p の予測が効くとすれば、(p=1/2 がランダム)
n 回の試行での当たりの期待値が np、分散が np(1-p) だから、
頻度については期待値が p、分散が p(1-p)/n で、
z=(α-p)/√(p(1-p)/n) で良いのかな?
242132人目の素数さん:01/09/09 11:59
1+1=2を証明しろ
243132人目の素数さん:01/09/09 13:08
覆面算
ONE+TWO+FOUR=SEVEN
244132人目の素数さん:01/09/09 13:17
なんか、難しいらしいですが、お答えが無いので・・
モナドってどなたも解らんですか?
私自身ではぜんぜんわかりませんでございまして。
245132人目の素数さん:01/09/09 13:34
>>232
置換積分だけでもできたYO!
I=∫{0→π/2}log(cosθ)dθとおいて
 I
 =∫{0→π/2}log(cosη)dη
 =∫{π/2→π}log(cos(θ-π/2))dθ (θ=η+π/2と置換)
 =∫{π/2→π}log(sinθ)dθ
 I
 =∫{0→π/2}log(cosη)dη
 =∫{π/2→0}log(cos(π/2-θ))(-dθ) (θ=π/2-ηと置換)
 =∫{0→π/2}log(sinθ)dθ
だから
 2I
 =∫{0→π/2}log(cosθ)dθ+=∫{0→π/2}log(sinθ)dθ
 =∫{0→π/2}log(sinθcosθ)dθ
 =∫{0→π/2}log(1/2)(sin2θ)dθ
 =∫{0→π/2}log(sin2θ)dθ+∫{0→π/2}log(1/2)dθ
 =∫{0→π/2}log(1/2)(sin2θ)dθ-(πlog2)/2
 =∫{0→π}log(sinη)(1/2)dη-(πlog2)/2
 =(1/2)(∫{0→π/2}log(sinη)dη+∫{π/2→π}log(sinη)dη)-(πlog2)/2
 =(1/2)(2I)-(πlog2)/2
 =I-(πlog2)/2
 ∴I=-(πlog2)/2
246132人目の素数さん:01/09/09 13:43
>>244
この数学辞典にはモナードとのってるものの事?
そうだとしたら書いてあるとうりだとおもうんだけど
どこがわからんかすらわからんではむいてないんじゃないとしかいえない。
247132人目の素数さん:01/09/09 14:47
The chef in our place is sloppy, and when he prepares a stack of pancakes they come out all different in sizes.
Therefore,when I deliver them to a customer, on the way to a table I rearrange them
(so that the smallest winds up on the top, and so on, down to the largest at the bottom)
by grabbing several from the top and flipping them over, repeating this (varying the number I flip) as many times as necessary.
If there are n pancakes, what is the maximum number of flips that I will ever have to use to rearrange them?

stack 山
wind up ・・・で終わる
grab つかむ
flip over 裏返しにする、ひっくり返す
248132人目の素数さん:01/09/09 15:22
わかる方にはすんなりわかるのでしょうが、一つ質問です。

I(k)=e^{k}∫[0,k](e^{-x}f(x))dx
(f(x)=x^{p-1}(x-1)^{p}・・・(x-n)^{p}で、n≧1、pは十分大きな素数)
とする。
|I(k)|≦ke^{k}n^{m} (0≦k≦n, m=np+p-1)
という評価を用いて、
J=-(a_{0}I(0)+・・・+a_{n}I(n)) (a_{i}は整数)

|J|<c^{p}(cはpに依存しない正定数)

という風に評価できるというわけなのですが、なぜにこのようなことができるのでしょうか?
”cはpに依存しない”というところがまたひっかかってしまって。
どのような要領でcをとってやればよいのか、どなたかお教え願います。
249132人目の素数さん:01/09/09 16:15
質問

A+B+C=180度のとき
cosA+cosB+cosC=1+4sin(A/2)*sin(b/2)*sin(c/2)を証明せよ

半角公式?
250132人目の素数さん:01/09/09 16:20
半角板逝ってください。
http://www2.bbspink.com/ascii/
251132人目の素数さん:01/09/09 16:26
>>248
I(k)≦k(e^k)(n^m)≦n(e^n)(n^(np+p-1))≦n(e^n)(n^2np)
M=max{|a_i|}とおいて
|J|
≦nM(n^2n)^p
=AB^p (A=nM,B=(n^2n))
以下略。
252132人目の素数さん:01/09/09 16:27
>>249
積→和公式を右辺につかへ。
昨日家庭教師をしていまして、高校生に確立を教えていたのですが、
§の見出しにこんな問題がありました。「平面上に等間隔に引いた
平行線があります。この幅の半分のピンをこの平面に落とすとき、
このピンが線分と交わる確率は1/πである」この証明は難しいのです
か?ご存知あれば教えてください。(一緒になんでだろう?って考え
たけど分かりませんでした…)
>>253
「ビュフォンの針」で検索してみそ。↓例えばここ。
http://www.kwansei.ac.jp/hs/z90010/sugakuc/toukei/buffon/buffon.htm
255132人目の素数さん:01/09/09 17:12
256132人目の素数さん:01/09/09 17:13
>>254
すごいかぶり方した。
257254:01/09/09 17:21
>>255=256
俺はgoogleの一番上(w
258教えてください:01/09/09 20:19
 6ケタの数字がある
これを仮にABCDEFと置き換えます
この数字を3で割ると1ケタ目の数字が一番前に移動し
FABCDEとなる

式にすると
ABCDEF÷3=FABCDE

またAを一番後ろに移動してから2で割ると
答えがFABCDEとなる

BCDEFA÷2=FABCDE

同様に
CDEFAB÷6=FABCDE
DEFABC÷4=FABCDE
EFABCD÷5=FABCDE
となる

元の数字ABCDEFはいくつでしょう


 ど〜してもわかりません!!
お力を貸してくださいm(_ _)m
>>258
x=0.FABCDEFABCDE...とする。
10x=F.ABCDEFABCDEF...
 3x=0.ABCDEFABCDEF...
より7x=F。
FABCDE*6<1000000からF≦1。
F=0とするとx=0からABCDEF=0なのでだめ。
F=1とするとx=1/7=0.142857...から
ABCDEF=428571でこれは条件を満たすので
元の数は428571。
260はなう:01/09/09 21:00
>>258
ふむ、面白い問題だぎゃ。
FABCDE*6=CDEFAB
より、Fは1か0(2以上だとかけて7桁になる)。Fが0だと一番最初の式が
ABCDE0 / 3 = ABCDE となり、ありえない。
よってF=1。
すると第1式ABCDEF÷3=FABCDE の 1の位を比べると、 E=7
これを元に、同様に1の位を比較して、
BCDEFA÷2=FABCDE より、 A=4
CDEFAB÷6=FABCDE より  B=2
DEFABC÷4=FABCDE より  C=8
EFABCD÷5=FABCDE より  D=5
ということで、元の数字は 428571
261はなう:01/09/09 21:02
>>259
あぅ、カブってら
262助けてください:01/09/09 21:25
有限体F_q(q=p^m)上の楕円曲線E(F_q)をy^2=x^3+ax+bとし、
楕円曲線上の点をP、Rとする。このとき、P =l Rとなる整数 l を求める問題で、
Weil対を使って楕円曲線上の点を有限体上の点に置き換えて求める方法の
アルゴリズムを教えてください。

よろしくお願いします。
263132人目の素数さん:01/09/09 21:44
分数の計算の仕方と分数を小数に、小数を分数に直すには
どうしたらいいんですか?
264258:01/09/09 22:02

>>259,260
ありがとうございます!!
265132人目の素数さん:01/09/09 23:22
[1]X,Yは独立な確率変数で、どちらも標準正規分布に従うものとする。
すなわち確率X,Yはともに確率密度関数

   f(x) = exp(-x^2/2)/√(2π)

をもつものとする。
V = X+2Y, W = 3X+Y とするとき、
(V,W)の同時密度関数、
Vの確率密度関数および平均と分散を教えて下さい。
266117:01/09/09 23:27
115 名前:132人目の素数さん 投稿日:01/09/07 15:01
「それぞれ一つずつ1からmまでの数字が書かれたカードm枚をよく混ぜ、無作為にn枚抜き取る。
 抜き取ったカードの中で、一番小さな数字の期待値を求めよ」

という問題です。よろしくお願いします。



117 名前:132人目の素数さん 投稿日:01/09/07 15:54
>115
Σ(k=1〜n−m+1) k*m-kCn/mCn


121 名前:115 投稿日:01/09/07 16:19
>>117
もっと簡単になりませんか?
Σを使わずに表せるようなんですけれども・・・



これ、考えたけど分からなかった。誰か出来ます?
>>266
>>186 にある。
268116:01/09/10 00:05
113 名前:132人目の素数さん 投稿日:01/09/07 14:35
>>47だけど、誰も教えてくれにゃいにょ?

実直交行列O(n)={X∈GL(n,R)|XX^t=1}がR^{n(n+1)/2}次元多様体って
授業で習ったけど、ユニタリー行列U(n)の場合はどうなるのかなあと思ったんです。



116 名前:132人目の素数さん 投稿日:01/09/07 15:43
>>47


>また、Cの線形空間による局所座標によって定義された可微分多様体は
>Rの局所座標系によって定義された可微分多様体になるのでしょうか?

は、多分直感的にはそうだと思うけど。ただ複素多様体って、恐ろしく
難しくて、そういいきって良いかどうか自信がない。

n次元複素多様体なら2n次元実多様体は言えると思うけど、
逆は言えなそう。実数2変数の2つの実関数が微分可能でも、
1複素数変数の1つの複素関数と見なしたとき複素関数として
微分可能とはかぎらないし。

複素n次行列全体はC^(n^2)のベクトル空間と同型。

ユニタリであるためには、n^2個の式が成り立たなくてはならないが、
n^2−n^2=0で、そんなはずないから、n^2個の中に独立でな
いものが混じってるんだろうな。で、たぶん実数C^同様、
C^{n(n+1)/2}でいいんじゃないかな。
で、それは多分、R^n(n+1)と同相

保証は出来ないが。


これ、後でじっくり考えたんですが。かなり直感的類推なんですが

まず、n次実直行行列

成分の数はn^2
直行行列になるための条件の式は、Aの転置*A=単位行列
で、対角成分に対象な成分は同じ式になるから、
1+2+・・・+n=n(n+1)/2
だから、直行行列の多様体としての次元は
n^2−n(n+1)/2=n(n−1)/2
となりそうだが、私はどこかで勘違いしてるのかな?
n=2のときは、1次元で、じっさい平面の回転行列は角度θ
をパラメータとして表せるが。もちろん、直行行列としては、
「裏返し」する行列も入れてだが。
269116:01/09/10 00:06

複素行列のユニタリの場合、これを実数のベクトル空間と見なす。
つまり、実2*n^2の中の部分集合を構成していると見なす。
ユニタリであることによって、成分を拘束する式の個数はやはり、
n(n+1)/2個

よって、2*n^2−n(n+1)/2=n(3n−1)/2
が多様体としての次元。

では、n^2次元複素ベクトル空間と考えると、
これは出来ない。つまり、ユニタリであるために成分を拘束する式が
複素数の正則関数として表されていないから。共役複素数を取るというのは
正則ではない。

で、複素行列について、Aの転置*A=単位行列となる物(なんて言うか知らない)
全体は、実行列の時と同様で、n(n−1)/2次元の複素多様体になりそうだが。

そしてそれは、実ベクトル空間と見なせば、次元あって、
拘束する式の数はn(n+1)/2個だが、実数としてみたら、各式が
実部と虚部の式となってるから、n(n+1)個の条件式といえる。
よって、2*n^2−n(n+1)=n(n−1)

で、一般的にいって、n次素多様体ならば、2n実多様体
逆はしらない。


で、良いんでしょうか?言葉の不正確さなどは気にしないでください
        
270117=266:01/09/10 00:14
>>267
すみません。見落としてました。じっくり考えてみます。
271132人目の素数さん:01/09/10 01:08
ライプニッツ・フォーミュラって何ですか?
272132人目の素数さん:01/09/10 01:24
>>251
なるほど・・・。そのあとのA的なものの扱いに私は困っていたわけですが、
あまり困る必要はなかったようですね。
とにかくありがとうございました。
273おじさん@素人:01/09/10 01:34
>>269
O(n) が n(n-1)/2 次元というのは正しい。
U=(u_1,...,u_n) がユニタリーである条件は
(u_i~,u_i)=1, (u_i~,u_j)=0 (i≠j) であって、
前者は実部だけ、後者は実部と虚部の式が出来るが、
(u_i~,u_j)=(u_j~,u_i)~ なので、結局 n^2個の式となって、
U(n) の次元は n^2 となる。
274132人目の素数さん:01/09/10 01:40
U(n)の次元について.
Lie環を見れば線形代数になる.
275132人目の素数さん:01/09/10 02:20
>>274
接空間が{n次元歪エルミット}だから・・・ということね。
276132人目の素数さん:01/09/10 03:14
漸化式
  y_{n+1} = 4y_n(1-y_n)   (0<y_n<1)
の一般項y_nをsinを使って表してください。
277276:01/09/10 03:55
わかったんで、やっぱいいです。
278132人目の素数さん:01/09/10 04:57
∫[∞,‐∞]exp{-x^2}dx=√π

となるのが分りません
どうか分りやすく教えてください。
お願いします。
279269:01/09/10 05:29
>>273 >>274 >>275
どうもありがとうございます。だんだん、難しい話に、、。
直感的には理解できても、証明はめんどくさそう。
逆の場合、証明されても、感覚的によく分からない物、ってのも有るけど。

多様体の次元=それが入っている空間の次元−その多様体を成立させる条件式の数
なんて、一般的には言えそうにないが、ほとんどの場合いえそう。
こういうのは、数学的にどう表現して良いやら。
「ほとんどの場合」って、測度がある場合は、「例外となるところの測度は0」となるが。
280nanasi:01/09/10 06:01
>>278
∫[-a,a]exp(-x^2) = 2*∫[0,a]exp(-x^2)

I(a) = ∫[0,a]exp(-x^2)dx
{I(a)}^2 = ∫[0,a]exp(-x^2)dx * ∫[0,a]exp(-x^2)dx
= ∫[0,a]exp(-x^2)dx * ∫[0,a]exp(-y^2)dy
= ∬_[R]exp(-x^2-y^2)dxdy (R:第1象限上にある長さaの正方形)

この正方形に内接する四分円C1と、外接する四分円C2を考える。
∬_[C1]exp(-x^2-y^2)dxdy = ∬_[C1]exp(-r^2)*|r|drdθ
= π{1-exp(-a^2)}/4

∬_[C2]exp(-x^2-y^2)dxdy = ∬_[C2]exp(-r^2)*|r|drdθ
= π{1-exp(-2a^2)}/4

C1 < R < C2だから、
π{1-exp(-a^2)}/4 < {I(a)}^2 < π{1-exp(-2a^2)}/4
また、
lim[a→∞](π{1-exp(-a^2)}/4) = lim[a→∞](π{1-exp(-2a^2)}/4) = π/4

はさみうちの方法により、lim[a→∞]({I(a)}^2) = π/4
⇒lim[a→∞]I(a) = (√π)/2
⇒∫[-a,a]exp(-x^2) = √π

こんな感じでどうでしょう?
281132人目の素数さん:01/09/10 06:38
>>268-269,>>273-275
有難う御座います。
エルミートの場合は上手くいかないんですね。
複素数の場合の方が綺麗になりそうな感じなのに。
282249:01/09/10 13:06
何回計算しても

cosA+cosB+cosC=2-sinA-sinB-sinC

となってしまいます。
計算過程も含めて教えてください
283はなう:01/09/10 13:52
では。詳しく。
cosA+cosB+cosC
=cosa+cosb-cos(a+b)
=(1-2sin^2(a/2))+(1-2sin^2(b/2))-(1-2sin^2((a+b)/2))
=1-2sin^2(a/2)-2sin^2(b/2)+(sin(a/2)cos(b/2)-cos(a/2)sin(b/2))^2
=1+2sin^2(a/2)(cos^2(b/2)-1)+2sin^2(b/2)(cos^2(a/2)-1)-4sin(a/2)cos(a/2)sin(b/2)cos(b/2)
=1-4sin^2(a/2)sin^2(b/2)-4sin(a/2)cos(a/2)sin(b/2)cos(b/2)
=1-4sin(a/2)sin(b/2)(sin(a/2)sin(b/2)+cos(a/2)cos(b/2))
=1-4sin(a/2)sin(b/2)(cos((a+b)/2)))
=1-4sin(a/2)sin(b/2)sin(c/2)
ってとこ。詳しく書いたらつかれた〜
284132人目の素数さん:01/09/10 14:07
>>282
慣れてれば cos((A+B)/2)=cos((180°-C)/2)=sin(C/2) に注意して、
cosA+cosB+cosC=2cos((A+B)/2)cos((A-B)/2)+1-2(sin(C/2))^2
=1+2{cos((A-B)/2)-cos((A+B)/2)}sin(C/2)
=1+4sin(A/2)sin(B/2)sin(C/2)
285132人目の素数さん:01/09/10 14:24
誰かこの問題わかる人お願いします。
区間(a,b)に無理数が存在することを示せ。

大学の問題です。
286132人目の素数さん:01/09/10 14:25
>285
難しいですね。
挑戦してみます。
287132人目の素数さん:01/09/10 14:42
>>285
a<r<bなる有理数rをとる。自然数n>√2/(b-r)に対し
a<r<r+√2/n<bなのでr+√2/nは(a,b)の中の無理数。
288132人目の素数さん:01/09/10 14:48
>>285
(a,b) は空でない、つまり a<b ですよね?
a<p<q<b となる有理数 p,q をとって
x=p+(p-q)/√2 とすると、x は無理数で、x∈(a,b)

(a,b) は非可算、有理数は可算だから。というのもある。
289192:01/09/10 16:10
>述語論理を使って証明しなくちゃいけなくて

209 名前:まさ_r 投稿日:01/09/09 00:51
>>192
>ありがとうございます。
>でもこれって命題論理というやつじゃないのでしょうか?
それとも命題論理とはまったく違うものなのでしょうか?

命題論理だけど述語論理じゃないから、質問の答えになってるかどうか、よくわからんが
290270:01/09/10 16:31
>>186
じっくり考えてやっと理解できた。

Σ(k=1〜n−m+1)k*m-kCn=m+1Cn+1
も、mに関する帰納法で証明できた。

しかし、結論を知らなくて左辺だけ与えられていたら、ひたすら計算して右辺に
変形出来るのか、自信がない
291???:01/09/10 16:56
MATRABを用いて円周率を求めるにはどうすればよいの?n=3;

i=1:n;
j=1:n;

r1(i,j)=rand(n);
r2(i,j)=rand(n);

R1(i,j)=2*r1-1;
R2(i,j)=2*r2-1;
ここから先がわかりません。
教えてください
292footloose:01/09/10 17:13
D={(x,y)∈R^2|0≦y≦x≦1}として、2重積分∫D(ylog(x+1))dxdy
を求めたのですが、解が1/3(log2-5/12)になりました。

簡単な問題なのですが、合ってるかどうか不安なので、
その解が、合っているかどうかだけでも教えていただきたいのですが、
どうぞよろしくお願いします。
293132人目の素数さん:01/09/10 18:02
>>292
あってんじゃない?
294(◎−◎):01/09/10 19:16
1+2+4+8+・・・て初項1公比2の無限等比級数ですよね。
和の公式(初項/1−公比)に代入すると1/1−2でマイナス1に
なるんですけど。でもどう考えても無限大のはずですよね。公式の使
い方に何か条件があるのでしょうか。ご教授願います。
295おじさん@素人:01/09/10 19:20
>>290
2項係数を C(n,k) の形に書き、k<0 または k>n の時は C(n,k)=0 とする。
一般に 納k=0,n]C(k,l)C(n-k,m)=C(n+1,l+m+1)
(例えば左から数えて) l+1 個目を k+1 番目の場所に置くとして
場合分けして和を取る。計算で無理矢理見つけるなら:
a(l,m)=納k=0,n]C(k,l)C(n-k,m) とすれば
納k=0,n]((1+x)^k)((1+y)^(n-k))=納l,m=0,...,n]a(l,m)(x^l)(y^m)
左辺×(x-y)=(1+x)^(n+1)-(1+y)^(n+1) だから、
(x^(l+1))(y^m) の係数を比較して
a(l,m)=a(l+1,m-1) (m≧1), a(l,0)=C(n+1,l+1) これによって
a(l,m)=a(l+1,m-1)=...=a(l+m,0)=C(n+1,l+m+1)
>>291
(1,1),(-1,1),(-1,-1),(1,-1) を頂点とする正方形、
つまり区間 [-1,1] の直積の中に一様に n 個の点を撒いて、
半径1の円に属する点、つまり点(0,0) からの距離(の2乗)が
1以内になる点の割合から 面積の比 π/4 を出してみようってことでしょ。
296132人目の素数さん:01/09/10 19:21
公式の証明を見返すと、絶対値が1より小さいときにしか成り立たない公式
であることが分かるはずですが。>>294
297132人目の素数さん:01/09/10 19:23
>>294 -1<公比<1
>294
無限級数の場合ではなくて、有限和の公式に入れて極限をとってみてください。
299(◎−◎):01/09/10 19:37
296さん、297さん初歩的質問で失礼しました。
1+1=2
0/0
1=0.9999999999999……
が理解できない厨房、文系、メーテル,nanashi
およびその他もろもろは、
どこの大学でも言いから、数学科を受験し(編入、社会人入学でも可)
数学科に逝って、そこの一番単位をくれそうも無い教授に
「あんたの単位はいらんから上記問題をオレ(ワタシ)にわかるように
教えろ」
と逝って下さい。
少しは世の中平和に成りますので。。。。
301(◎ー◎):01/09/10 20:04
298さん、a(1−r^n)/1−rでしょうか。たしかに
r=2でn=∞なら分子も分母もマイナスで、答は「2の無限乗−1」
で実質無限大ですね。ご指摘ありがとうございました。
302132人目の素数さん:01/09/10 20:15
 (1)
ベクトル
V(1)=[-1,9,-3,5] ,V(2)=[-5,3,-12,1] ,V(3)=[5,7,12,2] ,V(4)=[1,-5,2,0]
について、V(1)、V(2)の張るベクトル空間と、V(3)、V(4)の張るベクトル空間の
共通部分の基底を見出せ。

 (2)
   [5 4 -4]
 A=[6 7 -6]
   [7 7 -6]

とする。自然数nに対して

   [1]
 A^n[1]
    [1]

を求めよ。

という問題なんですがさっぱりわかりません。教えてください。
303132人目の素数さん:01/09/10 21:17
複素平面上の円周C:|z|=2について、次の積分を求めよ。
ただしCの向きは、半時計回りとする。

 (1) ∫[C]z^2/(z^2+z+1)dz
 (2) ∫[C]z^3/(z-1)^2dz

っていう問題ですが私にはぜんぜんわからないのです。誰か解いてください。
304溝地:01/09/10 21:21
f(x)を区間Iで定義された関数とする。
以下の2条件は同値である事を示せ。
@ f(x)はa∈Iで連続
A Iに含まれる数からなる数列{An}で
  An→α(n→∞)
  を充たす任意のものに対して
  f(An)→f(α)(n→∞)

 よろしくお願いします
これ、Iが開区間でもいえるのかい?
306132人目の素数さん:01/09/10 21:49
2次元トーラスT^2=S^1×S^1は、2次元可微分多様体になることを
どのように示せばよいのですか?

よろしくお願い致します。
307溝地:01/09/10 21:52
>305
すみません。
それすらわかりません。
一応、問題文をそのまま写したのですが
印刷ミスがあるのかもしれません。
Iが閉区間でないとだめというのなら
その方向でお願いします。
308132人目の素数さん:01/09/10 23:16
>2次元トーラスT^2=S^1×S^1は、2次元可微分多様体になることを
>どのように示せばよいのですか?

S^1を2つの開集合に分けて(当然、ダブル場所がでる)、
これを1次元と対応させる。

S^1=A∪B で、A、BにRの開集合(開区間)との対応が定義されているなら、
S^1×S^1=(A∪B)×(A∪B)=(A×A)∪(A×B)∪(B×A)∪(B×B)
として、(A×A)などは、R^2の開集合と同相となることが示される。
309132人目の素数さん:01/09/10 23:18
>>302 (1) A=(V(1),V(2),V(3),V(4)) x=(p,q,r,s)†として、
Ax=0 を解く。{pV(1)+qV(2)} が共通部分。
rankA=3 なので共通部分は1次元で任意に1つ取れば良い。
(共通部分が2次元なら V(1),V(2) が基底)
(2) 固有値は 1,1,4 となる。4 に対する固有ベクトルは
(4,6,7) で、1 に対する固有空間は W=(x,y,z)|x+y-z=0}
dimW=2 だから対角化しても良いが、(1,1,1)-t(4,6,7)∈W
より t=1/3 だから、(1,1,1)=1/3(4,6,7)-1/3(1,3,4)
(1,3,4)∈W となってあとは容易。
>>303 留数計算。(1) ω=(-1+√3i)/2 として、
∫[C]z^2/(z^2+z+1)dz=∫[C]z^2/(z-ω)(z-ω^2)dz
=2πi(Res(ω)+Res(ω^2))=2πi{(ω^2/(ω-ω^2)+ω^4/(ω^2-ω)}=-2πi
(2)∫[C]z^3/(z-1)^2dz=2πi(Res(1))=2πilim(z→1)(z^3)'/1!=6πi
>>304 α=a ですね。でないと変。
1⇒2 1を仮定。ε>0 に対し δ>0 をとって、|x-a|<δ⇒|f(x)-f(a)|<ε
とできる。An→a(n→∞) なら、自然数 N をとって、n>N⇒|An-a|<δ
とできるから、|f(An)→f(a)|<ε
2⇒1 1がダメとすると、あるε>0 に対しては、どんなδ>0 をとっても
|x-a|<δ かつ |f(x)-f(a)|<ε となる x があるから、δ=1/n (n=1,2,...)
に対する x を An とすれば、An→a(n→∞) だが f(An)→f(a)(n→∞) でない
>>306 教科書で「積多様体」をみるべし。
310309:01/09/10 23:23
訂正、スマソ
× |x-a|<δ かつ |f(x)-f(a)|<ε
○ |x-a|<δ かつ |f(x)-f(a)|≧ε
311132人目の素数さん:01/09/10 23:48
答えまでお願いします。
3直線 x+2y-5=0 2x-y+4=0 2x-4y-7=0
によって囲まれた三角形の内接円の中心の座標と半径を求めよ
312132人目の素数さん:01/09/10 23:49
確率解析における伊藤の公式について簡単に説明してください。
(数式無しでできるだけお願いします)
313278:01/09/10 23:53
>>280
はさみうちですか…
どうも有難うございました。
しかもこんなに早くに答えていただいてたなんて
返事遅くなってごめんなさい。
314132人目の素数さん:01/09/10 23:53
答えまでお願いします。

Oを原点とする座標平面上に、円C:x^2+y^2=25 と点A(8,6)がある。
AからCにひいた2本の接線の接点をそれぞれP、Qとするとき、
(1)3点O,P,Aを通る円Cの方程式を求めよ
(2)直線PQの方程式を求めよ
315 :01/09/10 23:53
>>311
円の中心の座標を適当において
その中心点とそれぞれの直線の距離がそれぞれ等しい
と言う条件で式をたててもとめよう。

◆点と直線の距離の公式◆
直線 ax+bx+c=0 と 点(x。,y。)の
距離は
|ax。+by。+c|/Sqrt(a^2+b^2)
>>311
内接円の中心を(p,q)、半径をr
|p+2q-5|/√5=|2p-q+4|/√5=|2p-4q-7|/2√5=r ←これを解く
317315:01/09/11 00:01
>>314
それも一緒。
(1)は点A(8,6)を通り傾きmの直線の式は
y=m(x−8)+6
と書けるから、この直線と円Cの中心の原点との距離が
円Cの半径5と等しいとして式をたてるとmについての
2次方程式がたつから、それぞれのmを求めて上の直線の式
に代入して整理し終わり。

(2)は2つの円C1:x^2+ax+y^2+by+c=0 と
C2:x^2+dx+y^2+ey+f=0 との2つの交点を通る
円や直線の式は、文字kを用いて
x^2+ax+y^2+by+c+k(x^2+dx+y^2+ey+f)=0
と書ける。
これが直線になる為にはk=−1でなければならないからこの値を代入
すれば直線の式は求まる。
318315:01/09/11 00:05
ああごめん。(1)は接線の式でなく円の式か…
319132人目の素数さん:01/09/11 00:05
x^2+y^2+x-2=0とx-2y+2=0
の2つの交点を通る円のうち、y軸から切り取る線分の長さが最小となるものの中心の座標と半径を求めよ
320はなう:01/09/11 00:12
>>314>>317
そうせんといても(1)は0Aを直径にする円だと思われ。
(2)もこの場合初等幾何の方が早いかも
321132人目の素数さん:01/09/11 00:15
y=x^2上にy=ax+1に関して対称な位置にある異なる2点が存在するようなaの範囲を求めよ
322132人目の素数さん:01/09/11 00:23
>319
(x-a)^2+(y-b)^2-r^2=0と置く
x^2+y^2+x-2=0から辺々引いて
(1+2 a)x+2b y-2+r^2=0
これは
x-2y+2=0
と一致するので
(1+2a):2b:r^2= 1:(-2):2

r^2=2(1+2a)
b=-(1+2a)

(x-a)^2+(y-b)^2-r^2=0とy軸の交点のy座標は
a^2+(y-b)^2-r^2=0を満たすから切り取る長さは
2√(r^2-a^2)で、これを最小とするものを求めればよい
323はなう:01/09/11 00:35
>>321
対称な位置にある2点を結ぶ直線の傾きは(-1/a)であるから対称ないちにある点のうちの片方を(k,k^2)とすると
2点を結ぶ直線はy=(-1/a)x+(k/a)+k^2でこれと曲線との交点のx座標をもとめると
-k-(1/a)。ここで、この対称な2点の中点の座標が
(1/2a,(k^2+(k/a)+1/(2(a^2)))であり、これがy=ax+1上になくてはならず、代入し、
kについての2次方程式にし、後は判別式でもといとくれ。
-√2>x、√2<x。
324KARL ◆gjHKPQSQ :01/09/11 00:54
整数 m,n に対して,実数 f(m,n) が定まっている.
この f が次の(1),(2)を満たすと仮定する.

(1)任意の(m,n)に対して f(m,n)≧0.
(2)任意の(m,n)に対して
4*f(m,n) = f(m-1,n)+f(m+1,n)+f(m,n-1)+f(m,n+1)
が成り立つ.

このとき実は f(m,n) は(m,n に依存しない)定数である
ことを証明せよ.
>324
有名な定理を持ってきましたねぇ
離散でも連続でも同じところがなんとも綺麗なもんですな
326132人目の素数さん:01/09/11 02:41
ある人が、1匹のキツネ・1匹のヤギ・キャベツ、をもって
旅をしていました。
 さて、いま川にさしかかったのですが橋がありません。
 そこで川の浅いところを渡るのですが、1つのものしかもっ
てわたれません。
 しかし、その人がいないとキツネはヤギを食べますし、ヤギ
はキャベツを食べてしまいます。

 うまく渡るにはどうすればよいでしょうか?
327132人目の素数さん:01/09/11 02:57
>>326
これはこれは古典的な問題だな
小学校上がりたての頃絵本で見た事があるぞ
キツネじゃなくて狼だったし、渡し舟だったけどな
2つ同時にいてもいいのはキツネとキャベツの組み合わせ
だけなのだから、何を持って渡るかは自ずと決まりまんがな
328縞栗鼠(シマリス)の親方:01/09/11 03:03
大検、大学受験の学習相談やってます。
全教科対応です。
お気軽にご質問ください。

「縞栗鼠(シマリス)の親方」まで
http://www.tkcity.com/renbbs/1/user/daiken.html
329132人目の素数さん:01/09/11 03:21
栗と栗鼠うざすぎ
330132人目の素数さん:01/09/11 04:26
>>302,>>309
(2)の解は、{1,3^n,4^n}となるのでしょうか?
331132人目の素数さん:01/09/11 04:36
>>330
はずれ。
332132人目の素数さん:01/09/11 04:46
>>331
(1,1,1)-t(4,6,7)∈Wより t=1/3 だから、
(1,1,1)=1/3(4,6,7)-1/3(1,3,4)
(1,3,4)∈W となってあとは容易。

の、“あとは容易”がいまいち分かりません。
教えてくださいませ。
333132人目の素数さん:01/09/11 05:41
>>332
ヨコレススマソ。
列ベクトルは書きにくいので行ベクトルで考えることにして
Aの転置=Bとおく。もとめたいのは(1,1,1)B^n。
(1,3,4)B=1(1,3,4) (∵(1,3,4)は1に対する固有空間からとったから)
∴(1,3,4)B^n=1^n(1,3,4)=(1,3,4)
(4,6,7)B=4(4,6,7) (∵(1,3,4)は4に対する固有空間からとったから)
∴(4,6,7)B^n=4^n(4,6,7)
以下略。
>>324

最大値の原理より自明
335縞栗鼠(シマリス)の親方:01/09/11 05:53
>>329 じぁお前死ね
>>335
馬鹿は消えろ
>>334
最大値の原理って何。
338ぶんけ:01/09/11 07:24
2の50乗を計算して10進法で表したいんですが
10桁表示の四則しかできない電卓しかありません(プラス紙とえんぴつ)
手順がなるべく簡単で、また、分かりやすいやりかたは
どのようなものですか

いや〜。あっちのスレに書くのはためらわれたんです
>>338
このスレを荒らさないでくださいです。。。
340303:01/09/11 07:55
309さんどうもありがとうございます。
341132人目の素数さん:01/09/11 08:15
>>302
もしかして答えは 1/3(4*4^n-1,6*4^n-3,7*4^n-4) でしょうか?
合ってたら教えてください。
342132人目の素数さん:01/09/11 09:17
あってるに一票。
343132人目の素数さん:01/09/11 10:20
>>338
計算手順はあっちの24でいいだろ。
巨大な数の取り扱いは、10000進法で計算すればなんとかなる。
344132人目の素数さん:01/09/11 11:28
n次元C^∞多様体について、連結である事と弧状連結である事とが同値な性質である事を証明せよ。

この問題が解りません。
教えてください。
弧状連結==>連結はOKです。
345132人目の素数さん:01/09/11 11:54
>>334
p∈Mを固定したとき集合X={q;pとqを結ぶ弧がとれる。}が
Mのひとつの連結成分をなすことをしめせばよい。
つまりopen&closed。つまりpとqを結ぶ弧があるときqの
近傍UでUのすべての点がpと結ぶ弧がとれる、とq_i→q
で各q_iはpと結ぶ弧がとれるときpとqを結ぶ弧がとれる
事をしめす。
346132人目の素数さん:01/09/11 13:31
切符売り場で、窓口を一つだけ開けて切符を売っていました。
30人の列ができたので、窓口を2カ所に増やしました。
4分後には並ぶ人が40人に増えました。
そこで窓口を3カ所に増やしたところ、それから8分後には列がなくなりました。
窓口を2カ所に増やしたとき、4カ所に増やしていたら、それから何分何秒で列がなくなっていたでしょうか?
347132人目の素数さん:01/09/11 14:03
>>34612分後。
348132人目の素数さん:01/09/11 14:24
>>346
え? 3分じゃないの?
349132人目の素数さん:01/09/11 14:24
>346
連続な場合で理想化をすれば、2分24秒
350132人目の素数さん:01/09/11 14:33
>>348ゴメン計算間違えてた。鬱だ、、、
351132人目の素数さん:01/09/11 14:35
350が正解と思う。
352348:01/09/11 14:34
まちがえたっ
2分だとおもう
353132人目の素数さん:01/09/11 14:35
↑349の間違いね・・・今日は天中殺か!
354132人目の素数さん:01/09/11 14:40
4分当たり、窓口2つで10人増え、窓口3つで20人減るから、
窓口4つなら50人減る。30/(50/4)=12/5(分)
355132人目の素数さん:01/09/11 14:51
>>345
申し訳ありません。
「つまり」以降が良く解らないです。
356132人目の素数さん:01/09/11 15:04
偏微分の記号ってなんて読むんですか?
ρを180度回転させた記号のことです。
あと、「div」と「grad」は何の略なんですか?
357132人目の素数さん:01/09/11 15:11
>>356
ラウンドディー
divergence
gradient
358356:01/09/11 15:18
>>357
サンクス!
これで堂々とこれらの記号が読めるようになりました。
以外に参考書に読み方が載ってなくて困ってました。
359普通の大学1年生:01/09/11 18:58
長さ1の線分をその平面の上で回転させることができるような平面の最小面積は
いくらでも小さくできる。らしいんですが、どうやって示すんですか?
参考図書だけでもいいんで教えてください。
>>356
∂はディーでいいよディーで
見てる人もそれで分かるし
361249:01/09/11 19:28
>>283,>>284
有難うございました。
熟れてますね。
362溝地:01/09/11 19:34
>309
本当に助かりました。
どうもありがとうございました。
363132人目の素数さん:01/09/11 21:51
ax+by=c
dx+ey=f
みたいな連立方程式の解についての一般論は線型代数で扱うことは理解しま
した(と思います)。しかし、
ax+bxy+cy=d
ex+fxy+gy=d
のような未知数xと未知数yの積xyが方程式に含まれるような場合の一般的な
考え方について知りたいのです。
これは、どのような分野になるのでしょうか。また、どのような本を読めば
いいですか?
お答えをいただければ幸いです。
364132人目の素数さん:01/09/11 22:00
>>363

よくわからんが、代数幾何あたりでは?
n変数の多項式=0
というのがいくつ(m個としよう)か与えられ、互いに独立なら、
複素数まで拡げれば、Cのn次元空間の中で、条件を満たす部分集合は、
n−m次元多様体になると思うので。特異点とか有るからややこしいが。
365132人目の素数さん:01/09/11 22:37
自然数nに対して
x^nの係数が1であるようなn次関数f_n(x)を次のように定める

(条件)
f_1(x)=x
f_2(x)={(x^2)-1/2}
f_n+2(x)={x・f_n+1(x)}-{1/4・f_n(x)}
-1≦x≦1

(問)
a(1).a(2).a(3)......a(n)を実数として
g_n(x)=(x^n)+{a(1)・(x^(n-1))}+....+{a(n-1)・x}+a(n)
とする
|g_n(x)|の最大値がf_n(1)より小さいとき
方程式g_n(x)=f_n(x)は少なくともn個ことなる実数解をもつことを示し
またf_n(x)は|g_n(x)|の最大値を最小とする関数である事を示せ。

とりあえず自分で
任意の自然数nと実数θに関して
f_n(cosθ)={1/(2^(n-1))}・cosnθであることを示し
f_n(x)の極値の絶対値はすべて等しいということも示せれたのですが
その先でこまっております
どなたかよろしくお願い致します
>>365
前半部はそこまでできてんならほとんどおわりだよ。
f_n(x)はy=1/2^(n-1)とy=-1/2^(n-1)のあいだを
n回いったり来たりする関数だから。

>またf_n(x)は|g_n(x)|の最大値を最小とする関数である事を示せ。

これ意味わからん。問題文ただしい?
>>366
はい。
問題は正しいのです。
後半部で自分もよくわからなくなりました
>>367
ああ、やっと意味わかった。つまり

f_n(x)は最高次の係数が1の多項式(=モニック多項式)g(x)の中で
|g(x)| (-1≦x≦1)の最大値が最小となるものであることをしめせ。

だね。そしてそのための誘導が前半部。
もしg(x)が最高次の係数が1の多項式で|g(x)|の最大値が1/2^(n-1)未満
であるようなものが存在したと仮定する。すると前半部より
g(x)-f_n(x)=0はことなるn個の解をもつけどこれはn-1次式だから
たかだかn-1個しか解がないハズ。よって矛盾。
>>368
訂正
f_n(x)は最高次の係数が1のn次の多項式(=モニック多項式)g(x)の中で
|g(x)| (-1≦x≦1)の最大値が最小となるものであることをしめせ。
>>368-369
なるほど。
すばらしい解答です。
参考にさせていただきますm(_ _)m
371KARL ◆gjHKPQSQ :01/09/12 00:10
以下引用

324 名前:KARL ◆gjHKPQSQ :01/09/11 00:54
整数 m,n に対して,実数 f(m,n) が定まっている.
この f が次の(1),(2)を満たすと仮定する.

(1)任意の(m,n)に対して f(m,n)≧0.
(2)任意の(m,n)に対して
4*f(m,n) = f(m-1,n)+f(m+1,n)+f(m,n-1)+f(m,n+1)
が成り立つ.

このとき実は f(m,n) は(m,n に依存しない)定数である
ことを証明せよ.
---------------------------------------------------
325 名前:132人目の素数さん :01/09/11 01:12
>324
有名な定理を持ってきましたねぇ
離散でも連続でも同じところがなんとも綺麗なもんですな
--------------------------------------------------
334 名前:132人目の素数さん :01/09/11 05:43
>>324

最大値の原理より自明
--------------------------------------------------
337 名前:132人目の素数さん :01/09/11 07:07
>>334
最大値の原理って何。
--------------------------------------------------
以上引用終わり

有名な定理ってほんとですか?
連続でも同じ、というのはもしかしたら
f'(z)=0→f(z)=定数
のことでしょうか。
最大値の原理とは? 私も知りたい。
この関数には最大値が存在するという意味ですか?
372334じゃない:01/09/12 00:17
>>371
fは最大値も最小値も持たないのはすぐ分かるだろ。
でも、そこからどうやって「fが定数に限る」と言ったらいいか分からない。
>>243
350+673+9382=10405
350+683+9372=10405
530+625+9548=10703
530+645+9528=10703
630+526+9647=10803
630+546+9627=10803
940+729+8935=10604
940+739+8925=10604
>>372
>fは最大値も最小値も持たないのはすぐ分かるだろ。
そんな事は言えません。
>>374
(p,q)で最大値をとると仮定して見ろ。
定数であることがすぐに言えるぞ。
もしかして、分からない?
>>375
f(m,n)=a(aは定数)なら
fの最大値=fの最小値=aです。
>>376
つまらねえ揚げ足とってんな。ボケ
378132人目の素数さん:01/09/12 01:44
>>377
揚げ足じゃないと思うけど
どのように2点をとっても長さが1以下となる領域の面積の最大値は?

これが領域が凸である場合と凹である場合、そもそも連結な領域でない場合、
それぞれ3通りのどれも解けなくて困っております。

使う言葉が間違っているかもしれませんが、そこはどうかお許しを…
380132人目の素数さん:01/09/12 02:27
>>324
条件(2)はラプラス方程式を離散変数で書きかえたもの。
よってfは調和関数。
よって調和関数の性質を調べれば条件(1)より題意は自明。
あとは自分で調べろ。
381379:01/09/12 02:46
凹である→凸でないでしたね。すいません

あと、これは平面上の領域に関する問題でして、
一般の次元に拡張したわけじゃないです。

連結な領域でない場合ってのは…

例えば平面上に円が1つある場合は連結で、
それで連結でないってのは平面上に円の集合{A1,A2,A3…An…}があって
Anの面積はπ/ 2^(-n)となるような…そんな感じ言ったんですけど…

本当に用語の使い方意味不明ですいません。
大学入ったらちゃんと勉強しますからどうか見捨てないで頂けないでしょうか…
382132人目の素数さん:01/09/12 02:52
>>379 あの、場合わけ必要ですか?
三角不等式を使うと、凸領域に帰着できません・・・?

凸領域と思ってよいなら、まず距離が長さ1の二点をとって、その
線分の中心を考えると、そこから長さ1/2の範囲にある点が
すべて問題の領域に含まれる、で、半径1/2の円、より
面積π/4、じゃないでしょうか?
383132人目の素数さん:01/09/12 03:03
>>382訂正

三角不等式で、領域が凸であると仮定してよいことをいう。

次に、距離が1である二点を結ぶ線分をとって、線分上のすべての点から
距離が1以下になる領域を求める。(円の共通部分で、凸になるはず)

その領域のの体積を求める。(計算はどうぞ・・・)

で、できませんか?
>>382-383
確かに距離が1である二点とって、その線分上に領域に含まれない点があったとして
その点を新たに領域に加えても距離が1以上になる2点は出来ませんよね…

これについては
距離が1以下でなくて、丁度1となる2点が無いようにする最大面積は?
という問題も考えていてその問題と混同してたみたいです。
意味不明に場合分けしてしまって混乱させてしまって申し訳ありません>382-383さん

いくら分からないとは言えここら辺まで聞くのは図々しすぎるのですが、
もしこっちも簡単過ぎて暇つぶしにもならねぇという方がいたらどうか教えて頂ければ…

π/4が最大となるかはもう少し詳しく教えて頂けないでしょうか
いや、こんな深夜に何度も聞いて申し訳ありません、ほんとすいません

幅1の定幅曲線で囲まれる領域以外にこれ以上領域に新たに点を加えられない
ことさえ証明出来れば円が一番大きい定幅曲線である(という感じだったような)
ことが幾何学の本に載っていたのでそれを使えるのですが…ってこのやり方不器用ですね
385384:01/09/12 03:56
379=381=384です。すいません
386132人目の素数さん:01/09/12 07:48
>>380
本当に自明なの?じぶんでやってみた?
387132人目の素数さん:01/09/12 08:43
>386
何か不思議なことでも?
388132人目の素数さん:01/09/12 12:02
>>384
集合Sが条件を満たすとする。そのとき、S内の二点p、qで
距離が最大となるものをとる。このとき、三角不等式で、線分p、q上の
すべての点はSに含まれると仮定してよい。線分pqを直径とする
円Cを描くと、S⊆でなくてはならない。一方、Cも問題の
条件を満たすので、仮定よりC=S、よって面積=π/4

で、できませんか・・・?
389132人目の素数さん:01/09/12 12:10
>>387
偏差分はどう定義するの?
>>380
どんな調和関数の性質を使えば自明なの。
391132人目の素数さん:01/09/12 17:00
複素関数論が終わったら次ぎは何をやれば良いのですか?
392132人目の素数さん:01/09/12 17:02
大学での
線形代数T・U
微分積分T・U
確率・統計
複素関数論が終わったら次ぎは何をやれば良いのですか?
393132人目の素数さん:01/09/12 17:05
普通に考えると、積分論(ルベーグ積分)じゃないかと、>>392
394132人目の素数さん:01/09/12 17:10
なるへそ>393

東京大学大学院数理科学研究科数理科学専攻の具体的なカリキュラム
って知ってたら教えてください
>>394
取りあえず4学期は、
複素解析
代数
集合と位相

っていうか、ネット上で探せば見つかると思うぞ。
396132人目の素数さん:01/09/12 22:07
>>380
証明の方針だけでも教えてほしいんですけど
397379:01/09/13 00:36
>>388
>円Cを描くと、S⊆でなくてはならない。
こうなる理由が分からないので詳しく教えて頂けないでしょうか?

例えば正三角形の一つの頂点を中心として1辺を半径とする円弧を描き、
これを各頂点についてやって出来た図形は条件を満たすのですが、
この図形上で距離が最大となる2点の長さは1。
そしてこの二点を直径とした円を描いてもこの図形がその円の中に
収まりきらないのですが、この場合は↑の文が成り立たない気がするのです…

もしかしたら勘違いしてるかもしれません。失礼なこととは分かっているのですが、
あと少しでもいいですので詳しく教えていただきたいです。
398*@:01/09/13 01:29
>>367
具体的な計算法はグレブナー基底の話になると思います。
http://www.google.com/search?q=%83O%83%8C%83u%83i%81%5B%8A%EE%92%EA&hl=ja&start=0&sa=N
399398:01/09/13 01:30
>>363 でした。御免
400クレクレ249:01/09/13 05:40
その後,
三角形ABCの内接円の半径をr,外接円の半径をRとして
r=4sin(A/2)sin(B/2)sin(C/2)
の証明を教えてください
>>26 が分かる人いませんか。
402132人目の素数さん:01/09/13 06:25
>>390

本を読め
>>402
どれを使えば自明なんですか。
404132人目の素数さん:01/09/13 09:14
因数分解の質問です。
1) (x-y)^3+(y-z)^3+(z-x)^3
2) x^2*(y^3-z^3)+y^2(z^3-x^3)+z^2(x^3-y^3)
3) x^4*(y-z)+y^4*(z-x)+z^4*(x-y)
405>401:01/09/13 09:39
証明をぬかせば g(x,y)=xy
はこれをみたす。

f(x+y) = f(x)+f(y) をみたす f:R->R 関数が
f(x) = px しかないことが言えれば後は割と簡単と思われ。
406132人目の素数さん:01/09/13 10:00
>>405
>f(x+y) = f(x)+f(y) をみたす f:R->R 関数が
f(x) = px しかないことが言えれば後は割と簡単と思われ。

この部分は、言えないです。Q線形空間としてのRの基底をつかえば、Q線形写像f:R→Rはいくらでも作れるから。
(ただし、fに連続性があれば、言えます。容易。)
407132人目の素数さん:01/09/13 11:59
>401
f:R×Q→R
ならば
f(x,y)=xy
だけど、これだけを導くのなら
>g(g(a,b),c)=g(a,bc)
とか必要ないから何か関係あるのかなぁと思ってしまう。
408132人目の素数さん:01/09/13 14:03
>>404 全部交代式だから、(x-y)(y-z)(z-x) で割りきれるよ。
409132人目の素数さん:01/09/13 15:01
>>400 A+B+C=π のとき、sinA+sinB+sinC=4cos(A/2)cos(B/2)cos(C/2)。
r=2S/(a+b+c)=2・(1/2)absinC/(a+b+c)
=2RsinA・2RsinBsinC/(2RsinA+2RsinB+2RsinC)
=2RsinAsinBsinC/(sinA+sinB+sinC)
=2RsinAsinBsinC/(4cos(A/2)cos(B/2)cos(C/2))
=4Rsin(A/2)sin(B/2)sin(C/2)。
410132人目の素数さん:01/09/13 16:34
>>401
>>32で答え書いたつもりなんだけど。

 gが条件をみたす。
 ⇔環準同型h:R→Hom_Q(R,R)が存在してg(a,b)=h(b)(a)
 と書ける。

そしてこのようなものはいっぱいある。たとえば
例1)s:R→RをGal(R/Q)の元としてg(a,b)=s(b)aとさだめる。
例2)右からそのまま書けるという作用でしぜんにQ⊂Hom_Q(R,R)
  とおもえる。Hom_Q(R,R)の中心はQなのでr∈R\Qとk∈Aut_Q(R,R)
  を非可換であるようにとる。g(a,b)=k^(-1)(k(a)b)とさだめると
  これも条件を満たす。

自作問題つくりそこなったんじゃね〜の?
411132人目の素数さん:01/09/13 17:44
>>410
ごめん。例1は撤回。でも例2はいける。
412132人目の素数さん:01/09/13 21:03
>>397
ワリー、前の証明撤回ね。あーおっちょこちょいだ。
なかなかむずかしい問題ですな。長すぎて不細工だから参考にならんかもしれんけど、
一応の証明のアウトライン

問題の最大値を与える領域をD、その境界をBとする。
1、まず、Dが凸なことをいう。
2、次に、Bの任意の点pにたいして、D内のpから最遠の点qをとると、線分pqの
長さがちょうど1になると仮定してよいことを、三角不等式で示す。
3、凸領域の境界はほとんどいたるところ二階微分可能ということを用いて、
上の線分pqはp、qでのBの接線と直行することを言う。
(直行しないと、最遠という過仮定に反する)
以上から、微分方程式とみなして、
Dは一点(pqの中心だけど)に関して点対称なことがいえる。
これから、Dは実は直径1の円であることが容易に出てくる。

ってなもんでどうでしょうか?
413132人目の素数さん:01/09/13 21:10
哲学系の人に質問されたんだけど、ゲーデルの対角化定理、
知ってる人いますか?
スマリやンの本を読んでる層なんだけど・・・
基礎論の人、いたら解説お願いします。
なお、技術的な定式化だけでなく、理論のなかでの機能とか、
見通しがつくような解説だとよりよいと思われます。
よろしくお願いします。
414132人目の素数さん:01/09/13 21:17
↑ 訂正 前原昭二の「数学基礎論」します。
415132人目の素数さん:01/09/13 23:09
かなり稚拙な質問と思いますが、

代数的数αに対し、iαも代数的数。

これって・・・ある意味自明ですか?ちょっと説明がほしく思いまして。
よろしくお願いします。
416132人目の素数さん:01/09/13 23:19
exp(ax1) exp(bx1) exp(cx1)
exp(ax2) exp(bx2) exp(cx2)
exp(ax3) exp(bx3) exp(cx3)
こ3・3行列で、a.b.cはそれぞれ異なる実数、各xも異なる実数として、
行列式が0でないことを示せ・・・なんていう問題なんですが、
分かる方がいらっしゃいましたら教えていただけないでしょうか?
417132人目の素数さん:01/09/13 23:32
>415
αが代数的数ならば
α^2が代数的数なのはOK?
α^2を根とする代数方程式f(x)=0
に対してf(-y^2)=0はiαを根とする方程式だよ。

もっと一般に代数的数も係数に許した代数的方程式の根も代数的数だよ
418132人目の素数さん:01/09/14 00:12
413さんに質問した者です。
ゲーデルの対角化定理の証明が、カントールの対角線論法によっているらしいのですが
よく分かりません。

1変数xの論理式は全体として加算個であるから
それらおのおのの変数xに対象式0,1,2…を代入したものを表にできる。
A0(0) A0(1) A0(2) … A0(k) …
A1(0) A1(1) A1(2) … A1(k) …
A2(0) A2(1) A2(2) … A2(k) …

Ak(0) Ak(1) Ak(2) … Ak(k) …

対角線上にある要素 A0(0) A1(1) A2(2) … Ak(k) …
を考えて、論理式に論理式を対応させる一意対応Φが与えられるとすると
対角線要素からΦによって得られる論理式の列も表の上にあるから
Φ(Ak(k))=An(k) (k=0,1,2…)
とくにk=nとすれば Φ(An(n))=An(n) となり、論理式An(n)をAと表せば
Φ(A)=A すなわち、与えられたΦに対してΦ(A)=Aとなる論理式Aが存在する。

以上のことが教科書に書いてるんですが
これが対角化定理と、どう結びつくのかわかりません。
文系の数学素人ですが、説明よろしくお願いします。
419132人目の素数さん:01/09/14 00:41
マンデルブロード集合って具体的にどう言うものを言うんですか??
420132人目の素数さん:01/09/14 00:52
高校入試で解けなかった問題です。
今すると解くのに20分かかりました。年かな..
1.円筒の底に半径rの三個の球体を隙間なく配置する場合の、円筒の半径を求めよ。
2.三個の球体の上に同じ大きさの球体を1個、隙間なく重ねたとき、全ての球体の中心を結んでできる立体の堆積を求めよ。
421132人目の素数さん:01/09/14 01:31
>>417
やはり稚拙だった・・・。とにかくありがとうございました。
最後の行の記述は興味深げ。
422420:01/09/14 01:48
堆積>体積の間違いです。
高校レベルなら書きこみ含めて2分で解けるでしょうね。
>>421
x^n+a(1)x^(n-1)+…+a(n)=0…(1)

の係数a(i)が代数的数とする
これをa(i)=g(x)の形の有理式に直す

a(i)を根とするような代数方程式をf(i,x)=0
とすればf(i,g(x))=0なる有理式が得られ、左辺を分母を
払って多項式にすれば
でてくる代数方程式はa(i)を含まず、その根には方程式(1)の根を含む方程式ができる。
全ての係数に対して繰り返し行えば代数的数を係数に持っていても根は代数的であることが
わかる。
424132人目の素数さん:01/09/14 02:17
ドキュンな質問で申し訳ありません。
小学校で球面の面積を4πr^2と習ったのですが、
どうしてそうなるのでしょうか?
今、大学生で多様体も習ったのですが、どうしてそうなるのか良く解りません。
425132人目の素数さん:01/09/14 02:31
>>424

パラメタ表示した曲面積を計算する公式知りません?
p、qをパラメタにして、それぞれで座標ベクトルを偏微分したのを
ベクトル積でかけて、絶対値を積分する公式。
あれででるよ。
ちなみに、多様体と思って表面積を計算するのは微分形式の積分(上と同値)
で、面倒くさいよ。R^nの部分多様体と思ってるのかも知れんけど・・・
426132人目の素数さん:01/09/14 02:34
>424
体積わかるよね円の面積πr^2と積分を使えば
(4/3) πr^3
となる。
これを微分すると4πr^2

なんでそれで出るかといえば
半径rの時の体積をV(r)、表面積をS(r)として
δを小さな正数だと思えば

V(r+δ)-V(r)≒S(r)δ…*
だからさ

お饅頭にアンコを注入していってお饅頭の皮が薄くなっていって
はじける直前、お饅頭の皮の内側の面積と外側の面積は大体一緒で
厚さがδだ。このときの皮の体積が*の右辺
お饅頭全体の体積から、アンコの体積を引いたのが*の左辺
ってなわけだ。

ちなみに4次元以上の超球のときも、n次元の体積を微分すればn次元の表面積になるし
簡単に2次元の球(円)でみれば面積πr^2を微分すれば円周2πrになるだろ?
427132人目の素数さん:01/09/14 03:49
>>425-426
有難う御座います。

>>425
申し訳ありません、知らないです。
私が持っている教科書には載っていないようです。
もう一度式だけ書いて頂けないでしょうか?

>>426
少々追加で質問したい事があります。
それは一般の凸型の図形で使えるのでしょうか?
楕円面で考えようとしたのですが上手くいきません。
円という特殊な図形でしか使えない方法なのでしょうか?
428132人目の素数さん:01/09/14 03:59
>427
もちろん球面の場合は一様だから厚さが一定値δとしてもいいけど
楕円面の場合は回転軸のところだけ見ても明らかに厚さδが
違うのでうまくいかないよ

楕円で、長軸の方をδ伸ばしたとき短軸の方の伸びもδかっていうとそうじゃないだろ?
だから、一般の凸型図形ではうまくいかない

楕円体の場合、体積の方は単位球をそれぞれの方向に定数倍しただけだから
単位球の体積にその倍率をかければでるけど
もし楕円体の体積から表面積が簡単にもとまるのなら
楕円の面積から楕円の周長が簡単に求まることになってしまって
大変なことになるね
429132人目の素数さん:01/09/14 04:05
>>427
V=(a,b,c)とする。ただし、a,b,cはp、qの函数で、
Vはパラメタ表示の位置座標ね。Dpでpにかんする偏微分を表すと、
Vp=(Dpa,Dpb,Dpc)を表す。qも同じ。
このとき、D→S(p、q)→(a,b,c)がC1級全単写なら、

Sの表面積=∬D[DpV×DqV]dpdq(積分範囲はDね・・微分とややこしいけど)

[]はベクトルのノルム、という公式。
杉浦光夫著『解析入門U』に詳しい説明があるよ。
430??:01/09/14 04:27
1回転って、なんで360度なの?100や200じゃいかんの?
431132人目の素数さん:01/09/14 04:28
かなり考えたのですがわかりません、、

A,Bは定円Oの周上の2定点であって、線分ABは直径でないとする。円周上
の動点Pにおける接線A、Bから引いた垂線の長さの和が最大になるような点P
の位置を求めよ。
432132人目の素数さん:01/09/14 04:55
訂正

×動点Pにおける接線A、B→○動点Pにおける接線にA、Bから
433132人目の素数さん:01/09/14 07:33
>>416
有名な行列だね。α=expa,β=expb,γ=expcとおいてその行列をAとおくとき

 A=
 α^1 β^1 γ^1
 α^2 β^2 γ^2
 α^3 β^3 γ^3

だからdetA=αβγP(α,β,γ)となる多項式Pがとれる。α=βのとき0になるので
Pは(α-β)でわれる。同様にしてP(α、β、γ)=c(α-β)(β-γ)(γ-α)
あとα=1,β=2,γ=3ぐらいいれてcをもとめればいい。
(あるいはどっかの係数比較する。)
434132人目の素数さん:01/09/14 07:34
>>431
ABの中点をMとすると、動点Pにおける接線にA、Bから引いた垂線の長さの和
はMからの垂線の長さの2倍だから、MPを最大にすれば良く、
Mを通る直径と円Oの交点でMから遠いほう。
435132人目の素数さん:01/09/14 07:48
>>420 1. (1+2√3/3)r 2. (2√2/3)r^3
436132人目の素数さん:01/09/14 09:29
教えてください。
   α+β+γ+δ=360°   sinαsinγ=sinβsinδ
   ならば, sin(α+β)sin(γ+δ)=0
となることがわかりません。
m(__)m お頼みもうします。
437132人目の素数さん:01/09/14 11:07
>>436
タイプし難いから、a,b,c,d でいいね?
a=b=120°、c=d=60°とすると問題のミスがわかる。
#詰まったら、具体的な値を入れてチェックしてみよう!
a+b+c+d=360°, sinasinc=sinbsind のとき、
2sinasinc=2sinbsind
-cos(a+c)+cos(a-c)=-cos(b+d)+cos(b-d)
∴ cos(a-c)-cos(b-d)=cos(a+c)-cos(b+d)
=cos(a+c)-cos(360°-(a+c))=0 (注)
一方、2sin(a+b)sin(a+d)=-cos(2a+b+d)+cos(b-d)
=-cos(a+360°-c)+cos(b-d)=-{cos(a-c)-cos(b-d)}
∴ sin(a+b)sin(a+d)=0
(注) a-c≡±(b-d) mod 360°となるから、
a+b≡c+d mod 360°または、a+d≡b+c mod 360°
a+b+c+d=360°により、
a+b≡0 mod 180°または、a+d≡0 mod 180°
で、sin(a+b)sin(a+d)=0 を示す問題と考えられる。
438132人目の素数さん:01/09/14 12:14
>>437
 よく問題を見直してみたらその通りでした。
 これがいわゆる,自分で自分の首を絞めるということでしょう。
 迅速な解答ありがとうございました。
439132人目の素数さん:01/09/14 13:21
誰か逆関数の定理のステートメント教えてクレー。
440132番目の素人さん:01/09/14 13:58

A:オレさぁ物理板行ってみたんだ
B:逝ってよし
A:「ガリレオ・ガリレイの苗字はガリレオ?orガリレイ?」ってスレがあってさ、
  オレにも解るネタと思って中見たらさ、
  「科学全般板に書くべきものをいちいち物理板でスレ立てるなよ」っていきなり言われてた
B:逝ってよし
A:じゃこの数学板はなに〜? 数学あんま無いし
C:イヤー、これはほとんど数学科じゃないニセモノが書いてんだよ
B及びD,E,F及び132番目の素数さん(複数):えー!? オレ院の数学科だけど
B:しかもオレ博士だし
C,D,E,F:まじーー!?
B:知らんの? 過去スレにガイデだよ
A:ガイデ?
B:イッテヨツ
441132人目の素数さん:01/09/14 14:53
どなたかCPアルゴリズムについて教えてください.
(The Cayley-Purser Algorithm)
概要は理解できるのですが,
安全性が理解できません.
よろしくお願いします.
442132人目の素数さん:01/09/14 16:06
>>428-429
有難う御座います。
443132人目の素数さん:01/09/14 19:27
質問です。
ていうか、ずっと前から疑問だったのですが‥‥‥。

「ベキ零群」っていうのは、一体ぜんたいナニが「ベキ零」なんで
しょうか?
詳しい人、名前の由来を教えて下さ〜い。
444132人目の素数さん:01/09/15 00:26
教えてください。

2次元球面 S^2 上の接ベクトル場で,零点を1個だけ持つものを構成することは可能か,可能ならば構成せよ.
445132人目の素数さん:01/09/15 00:30
>444
球面を複素平面に無限遠点を付け加えたものとみなす。
446choco:01/09/15 03:16
一辺がaの正5面体の体積は?
>>446
そんなものはない。
448真美:01/09/15 03:33
こんばんは。質問させていただきたいと思います。
a(1)=1,a(n+2)=3a(n+1)-2a(n),lim_[n→∞]a(n^2)/a(2n)=3
を満たす数列a(n)の第2項を求めよ。
という問題なのですが…。ご解答をお願い致します。
449縞栗鼠(シマリス)の親方:01/09/15 03:59
a(n+2)=3a(n+1)−2a(n)   n>=1 ★
の式は、次のように変形できます。
a(n+2)−2a(n+1)=a(n+1)−2a(n)
この左辺右辺を比べると、添え字が右辺のほうが1小さいだけで同じ形をしているか
ら、順次n=n-1,n-2,・・・1まで成り立つので
a(n+2)−2a(n+1)=・・・=a(2)−a(1)=a(2)−1 ☆
が成り立つ。
 一方★は、次のようにも書ける
a(n+2)−a(n+1)=2(a(n+1)−a(n))
これが n>=1 のすべての n について成り立つから
a(n+2)−a(n+1)=2(a(n+1)−a(n))
  =2^2(a(n)−a(n−1))=・・・・
  =2^(n)(a(2)−a(1))=2^n
だから、
 a(n+2)−a(n+1)=2^n(a(2)−1)  ※
※から☆を左辺どうし、右辺どうし引き算すると
 a(n+1)=2^n(a(2)−1)−a(2)+2  n>=1
となる。n>=2のばあいには
 a(n)=2^(n−1)(a(2)−1)−a(2)+2
となって、a(n)がnの関数としてあらわされた。
 で、
 a(n^2)/a(2n)
=(2^(n^2−1)(a(2)-1)-a(2)+2)/(2^(2n-1)(a(2)-1)-a(2)+2)●
が、n→無限、になっても、発散しないためには、(a(2)−1)=0でなくてはなら
ない(この理由は、2^(n^2-1)/(2^(2n-1))がn→無限で発散するから<この証明は
また別の問題になる、簡単でもない>である)。
すると、a(2)=1となる。このとき、●式の値は3にはならず、1になってしまう。
 したがって、この問題には答えはない。(a(2)がどんな値になっても、●の極限値
は3にならない)

 多分、まみさまの問題の写し間違えと思いますが、真美様の問題のとおりだったら
「適するa(2)はない」というのが答えになります。
450縞栗鼠(シマリス)の親方:01/09/15 04:02
>>448
大検、大学受験の学習相談やってます。
全教科対応です。
お気軽にご質問ください。

下記の掲示板で質問の回答をしております。
「縞栗鼠(シマリス)の親方」までどうぞ
http://www.tkcity.com/renbbs/1/user/daiken.html
451132人目の素数さん:01/09/15 04:19
論理関数を簡単化して主項を求める問題です。はEXORです。

f = (abcd)'(bd)'+ab'c'+a'bc
= (a'+b'+c'+d')(b'd+bd')'+ab'c'+a'bc
= (a'+b'+c'+d'){(b'd)'(bd')'}+ab'c'+a'bc
= (a'+b'+c'+d'){(b+d')(b'+d)}+ab'c'+a'bc
= (a'+b'+c'+d')(bd+b'd')+ab'c'+a'bc
= a'bd+a'b'd'+b'd'+bc'd+b'c'd'+ab'c'+a'bc

どっか間違ってますか?どうしても正解と合いません。
ちなみに正解はb'd',a'cd',a'bc,a'bd,bc'd',ac'd,ab'c'です。
>450
荒らすのはヤメレ
453プラズマ:01/09/15 04:29
>>449
暇だからやってみた。
これ、一般式出して,極限出すと絶対に発散するね。
よってやはり答えなし。
454縞栗鼠(シマリス)の親方:01/09/15 04:36
>452
荒らしてなんかいませんよ
455132人目の素数さん:01/09/15 05:03
親方怖いです>>335
456縞栗鼠(シマリス)の親方:01/09/15 05:12
>455
スマソ だがね、やはり人の足を引っ張りたがるヤツが
たくさんいるからね、そういった輩にたいする反撃だから
気にせんでください。
457132人目の素数さん:01/09/15 05:43
>>448
君=>>16?だったら>>27で答えもらってんじゃん。一回答えもらって
導出部分がわからないなら“〜で答えをもらったけど導出もおしえてたもれ。”
って聞かないと教えてくれたひとに失礼だよ。君=>>16じゃないならスマソ。
>>456
>>173 >>206 >>328 >>450 は荒らしです。
459縞栗鼠(シマリス)の親方:01/09/15 06:03
>458

テメーぶっ殺すぞ!!!
面積がSである閉曲面DがR^3の中にあり、
Dから距離がx以内にある点で出来る立体の体積をf(x)とするとき、
f(x)の最大値と最小値を微分を使わないで求められないでしょうか?
>>460
fは単調増加でx−>∞のときf(x)−>∞だから
最小値0最大値なし。
462460:01/09/15 07:17
すいません。書き方まずったです。
xは自由変数じゃないです。
Dの方を色々と変えたときにDからの距離がa以内にある点で出来る
立体の体積Vの範囲を求めて欲しかったんです。
Sとaは固定して下さい
>>462
Dを縮めていくことでDからa以内にある点でできる立体は
半径aの球に近づき半径aの球を含むので
(4/3)πa^3<V(D)。
Dを伸ばしていけば長さがbのとき
(πa^2)b<V(D)で
bはいくらでも大きくできるので上限は∞。
464132人目の素数さん:01/09/15 10:36
>>444
x^2+y^2+z^2=1 上で、ベクトル場 (x^2-z-1,xy,xz+x) でいい?
>縞栗鼠さん
>>449 は貴方の掲示板のコピペですね。リンクにして下さい。
465132人目の素数さん:01/09/15 11:50
Vを体Kの元による数列{x(n)}の全体であって、相並ぶ m + 1 項の間に
漸化式 x(m+n) + a(m-1) x(n+m-1) + ・・・ + a(1) x(n+1) + a(0) x(n) = 0
( 任意の i について、a(i)∈K、n =1,2,… ) が成り立つものの全体とする。

数列の和とスカラ倍を、{x(n)}+{y(n)}={x(n)+y(n)}、α{x(n)}={αx(n)}で定義する。

1) VはK上のベクトル空間になることを示せ。
2) VはK上のmベクトル空間であることを示せ。
466132人目の素数さん:01/09/15 12:20
>465
(2) e_i(n)=1 (i=n),=0 (i≠n) で e_i を定義すれば、
e_1,e_2,...,e_m が基底
467132人目の素数さん:01/09/15 12:31
1 1 9 9=10
+−×÷()を使って、上記の式を完成してください…♪
468132人目の素数さん:01/09/15 12:36
>>467
11−9÷9=10
469132人目の素数さん:01/09/15 12:40
>>467
(1+1÷9)×9=10
470132人目の素数さん:01/09/15 13:23
>>445
有難う御座います。
471132人目の素数さん:01/09/15 14:05
>>467
1−1+9−9=1×0
1×1×9÷9=1+0
>>28

20からはそういう意図は全く読み取れんのだが…数学苦手?(ワラ
473472:01/09/15 14:14
うわぁ誤爆…
474132人目の素数さん:01/09/15 14:23
次のような密度関数f(u)がある。
0 u<−λ
f(u)= 1/2λ −λ≦u≦λ
0 λ<u

このときあるaが−λ≦a≦λの範囲にあるとすると
F(a)={(a+λ)/2λ}∫[-λ,a]uf(u)du
=(a^2-λ^2)/4λ
であることを示せ。ただしF(・)はf(・)の分布関数をあらわす。

これがわかりません・・・・。どうしてこうなるのか教えてください。。。
475132人目の素数さん:01/09/15 14:25
>>474
すみません。最初の部分ミスってます。
u<-λのとき f(u)=0
−λ≦u≦λのとき f(u)=1/2λ
λ<uのとき f(u)=0
です。大変失礼しました。。。
476132人目の素数さん:01/09/15 14:25
愛と呼ぶには妖しすぎると最初に感じた
きわどい視線からみあわせてたわいない会話に
誰にも め・い・わ・く かけなく わ・く・わ・く
秘密は無いと同じこと
 EROTIQUEね いつだって落ちるのに
 とっておく関係があったっていいじゃない
知り尽くしたらさよならだけがそこに待ってるの

心配 と・き・ど・き 純情 ド・キ・ド・キ
ゲームとは似ていて違う
 ROMANTIQUEね 合鍵を持っていて使わない
 大切なドアなんていいじゃない
それでもそれでもあるとき負けそうよ
その腕にその腕に崩れるわ
 EROTIQUEね いつだって落ちるのに
 とっておく関係があったっていいじゃない
この広い世の中にあなたさえいてくれて
いつだって私 オンナしている ずっと
         松任谷由実作詞作曲「EROTIQUE」1987
477132人目の素数さん:01/09/15 14:49
>>474 −λ≦u≦λのとき f(u)=1/(2λ) だよね?
F(a)=∫[-∞,a]f(u)du=∫[-λ,a]1/(2λ)du=(a-(-λ))/(2λ)=(a+λ)/2λ
∫[-λ,a]uf(u)du=∫[-λ,a]u/(2λ)du={(a^2)/2-(λ^2)/2}/(2λ)=(a^2-λ^2)/(4λ)
だと思うけど?
478馬鹿物:01/09/15 15:20
コンビネーションって何? 7C2 とかやりかたがわかんない(;_;)
479132人目の素数さん:01/09/15 16:27
>>477
ありがとうございました。わかりました。
f(u)=1/(2λ) というのが思いつきませんでした。。。
>>467
1×19−9=10
481132人目の素数さん:01/09/15 18:16
>>478
とりあえず「順列・組み合わせ」あたりの話です。
ネットで調べてみるか、数学の教科書を熟読してください。
482まさ:01/09/15 18:19
誰か横断性条件って何か説明できる人はいませんか?
月曜日テストなんです〜。助けてください。
483132人目の素数さん:01/09/15 18:39
>482
文字通り接しないということなんだが?
扱っているものによって、表現は多少違うけどね
484256人目の素数さん :01/09/15 18:48
うんうん、それはね・・・・
            ∧___∧
           ∧∧ ´∀`)
 オマエニハイワーン!(゚Д゚)⊃  ヽ
           |ヽ) 、  ノ、`つ
           (/∪ | | |
              (_,(_)
485132人目の素数さん:01/09/15 20:06
すみません、>>474についてなんですが。。。
このf(u)が、λ=1ならsymmetricだとかいてあるのですが、
それってどういうことですか?またどうしてそうなるんですか?
すみませんが、どなたかアドバイスください・・・
486132人目の素数さん:01/09/15 21:12
次の問題が解りません。
教えてください。

3次元lens空間L(p,1)がS^1をfiberとするS^2上のfiber bundleの構造を持つ事を示せ。

ここでL(p,1)はS^3={(z,w)∈C^2;|z|^2+|w|^2=1}
に同値関係〜を入れて
L(p,1)=S^3/〜
としたもので、

(z,w)〜(u,v)<==>
「z=g^a*u
w=g^a*v
a∈Z
g=exp(2πi*1/p)」

です。
487132人目の素数さん:01/09/15 21:22
fiber bundleは理解できてます?
488132人目の素数さん:01/09/15 21:33
>>487
h=(E,p,B)
E,B:C^∞多様体
p:E->B C^∞ map
で次の条件(locally trivial)を満たす。

任意の点t∈Bに対してある開近傍Uが存在して、
C^∞ diffeomorphism
f:p^(-1)(U) --> U×F
が存在して、
p*f^-1(t,a)=t
を満たす。

と習いました。
図で書いたほうが分かり易いのですが・・・。
489132人目の素数さん:01/09/15 21:37
>>486
(z,w)〜(u,v)⇒[z:w]=[u:v]なのでf(z,w)=[z:w]で写像
f:L→CP^1≡S^2 (≡は同相の意)が定義できる。これが局所自明fibre
になることをいえばいい。
490132人目の素数さん:01/09/15 21:45
>>489
解ってきました。
有難う御座います。
491パーキン損:01/09/15 23:01
明日「写像」のテストがあるのですがあの分野って何を勉強すればいいのでしょうか。難しすぎます。
492132人目の素数さん:01/09/15 23:11
>>491
あした?
明後日じゃなくって?
493数学苦手:01/09/16 02:27
どなたか教えてください。

1)|1-exp(iθ)|<=2 (θは任意の実数)
2)|1-exp(iθ)|>=2|θ|/π (-π<=θ<=π)

これが成り立つことを...
494choco:01/09/16 02:29
すんげー間違えました。一辺がaの正12面体の体積?
495132人目の素数さん:01/09/16 02:43
>>493 面倒だから t を使うよ。
|1-exp(it)|=√{(1-cost)^2+(sint)^2}=√{2-2cost}=2|sin(t/2)|
1) は簡単。2) は、「0≦x≦π/2 で sinx≧2x/π」 を使うべし
>491
写像のテストってどの分野の写像?
何年生?
どんな教科書使ってる?
難しいってどんな単語が出てる?

全く何言われてるのかわからん
497132人目の素数さん:01/09/16 13:25
f(x)=x^2-ax+a+2
g(x)=x^2+(3-a)x-3a

g(x)≦0 であるどんなxの値にたいしてf(x)>0となるaの値の範囲が
-2<a<2+2√3
となるのが理解できません。
どなたか教えてください。
498132人目の素数さん:01/09/16 13:53
>>497
高校一年生かな?
平方完成すれば、二次不等式に持ち込めるはずだよ。
499497:01/09/16 15:49
g(x)≦0から
a<-3の時 a≦x≦-3
a>-3の時 -3≦x≦a
a=-3の時 x=-3

f(x)を平方完成して
f(x)=(x-a/2)^2-a^2/4+a+2>0から
2-2√3<a<2+2√3  だいたい -1.4<a<5.4から
a>-3の時について考える

-3<a/2≦aの時
最小値f(a/2)>0となるのは
0≦a<2+2√3

a<a/2の時
f(a)=a+2>0 よって
-2<a<0

aの範囲は 2-2√3<a<2+2√3
よって 2-2√3<a<2+2√3 だと考えたのですが、
どこがまずいのでしょう。
a<-3,a=-3の場合は何故考えないの?
g(x)>0なxに対しては、f(x)>0でなくてもいいんだよ。
501パーキン得:01/09/16 17:26
明日、多様体の試験があるのですが、何を勉強すれば良いのでしょうか?
難しすぎます。
>>501
試験範囲を勉強すれば?

ていうか、そんなんでまともに答えてもらえるとでも思ってるの?
503本当に高2?:01/09/16 18:28
二次関数の質問です。
「x+3y=kのとき,x^2+y^2の最小値は4である。
定数kの値を求めよ」

x+3y=kをx^2+y^2に代入して
k^2-6ky+10y^2と出ました。
この式をyの関数と見ればk=±2√10
とでます。
でも,kの関数と見てしまうと答えが変になってしまいます。
このとき,kの関数とみることのできない理由を教えて下さい
504同上高校二年:01/09/16 20:58
微分方程式を勉強しているのですが、
x*(dy/dx)=yの問題で(基礎 汗
∫(1/y)dy=∫(1/x)dx
∴log(y)= log (x)
のあとどうして
y=nx(nは任意の実数)とできるのか良く判りません。
普通に解くとnxになるのは明らかですが、、、nがどこから出てくるのやらと。。
505132人目の素数さん:01/09/16 21:08
@自然数nと正の数tに対して
f_n(t)=∫(1→n){1/x)・|log(t/x)|}dxとする。
(1)各nに対して1≦t≦nにおけるf_n(t)の最大値A(n)と最小値B(n)を求めよ
(2)lim(n→∞){A(n+1)-A(n)}を求めよ

すいません。この定積分を一つ質問おねがいいたします
506505:01/09/16 21:09
名古屋大学の問題です
507132人目の素数さん:01/09/16 21:49
>>504
正確には
∫(1/y)dy=∫(1/x)dx
∴log(y)= log (x) +C (Cは積分定数)…@

となりC=log(n) (←Cは定数なのである定数nを使ってこう表せるはず。)
と考えれば@式は
log(y)= log (x) +log(n)
=log(nx)
となり
y=nx
となる
508504:01/09/16 21:57
>>507
なるほど、、不定積分の公式にCがなかったのが気になりますが、
そうすれば確かになりますね。。有り難うございました。
509 :01/09/16 22:11
>>503
この問題の場合、kは定数であるからkの関数としてみることはできない。
5102回目ですが…:01/09/16 22:11
>>503
kが定まったときに、xとyをx+3y=kの中で動かしたときの最小値でしょ。
だから、kの関数と見るならばそれは「(xやyによる)最小値の(kによる)最小値」
ってことにってしまう。kの前にkに制限されたx,yが動くんだよね。
説明が難しいな、ゴメン。
>>504
∫(1/y)dy=∫(1/x)dxの次は積分定数Cで
log(y)= log (x)+Cだよね。
e^c=nって置き換えをして見やすくすると…
log(y)= log(x)+log(n)
y=nxとなるんだよ。
+C(積分定数)のときlogの中身は勝手に何倍してもいい。のだよ。

ところで、長さ1の線分をその上で回転させることが可能な図形の面積の最小値が0に近づくってのの説明を誰か知りませんか。
どんな本を調べればいいかだけでもいいんで
511 :01/09/16 22:15
>>508
不定積分だからこそ積分定数がいるんだよ。
定積分だと[f(x)+C](a→b)
を計算するとき
Cが打ち消しあって消えるので
Cがどんな値だろうが関係ないので
定積分のときはわざわざ積分定数を
書く必要がない。
512510:01/09/16 22:46
504がカブッタお詫びに…
>>505
f=∫(1→t)(logt-logx)/x dx+∫(t→n)(logx-logt)/x dxとすると
logt/xはlog(t)log(x)で、log(x)/xは{(logx)^2}/2で積分できます。
あとは二次関数で、A(n)は(log(n))^2の定数倍ぐらいになって
(2)は、分母にn+1-nを置くと平均値の定理より、{(log(n))^2}'の極限値になって
log(n)/nはlog(n)<{n^(1/2)}+100とかで適当に押さえて、答え0
513はなう:01/09/16 22:53
頭ん中で考えてみたが至極普通の答えになつた。自信なし。間違ってたらスマソ。

>>505
f_n(t)=∫(1→t){(1/x)・log(t/x)}dx+∫(t→n){-(1/x)・log(t/x)}dx
   =[-log(t/x)](1→t)+[log(t/x)](t→n)
   =-logt-log(t/n)=-2logt+logn
(1)A(n)=logn B(n)=-logn
(2)0
514379=381=384=397:01/09/16 22:58
>>412
レスが送れてしまって申し訳ありません。
あなたのおかげでやっと問題が解けました。

n次元の場合と距離が丁度1にならないようにする場合が
自分で出来たのもここのおかげ。どうもありがとうございます。
515513:01/09/16 23:05
おっちょこちょいしとった。かぶった上に間違った。スマソ。正しくは>>512の通り。
516 :01/09/16 23:21
>>505
log(t/x)≧0のときx≦tであり
log(t/x)<0のときx≧tであるから
f_n(t)=∫(1→n){(1/x)・|log(t/x)|}dxは
f_n(t)=∫(1→t){(1/x)・log(t/x)}dx −∫(t→n){(1/x)・log(t/x)}dx
∴f_n(t)=(logt)^2-(logn)・(logt)+(1/2)・(logn)^2 (計算省略)

ここで X=logt (0≦X≦logn)とおくと
f_n(X)=X^2-(logn)・X+(1/2)・(logn)^2
={X-(1/2)・logn}^2+(1/4)・(logn)^2 (平方完成)…@
明らかに 0≦(1/2)・logn≦logn であるからグラフから
@は X=(1/2)・logn (つまり t=sqrt(n) )のとき最小値 B(n)=(1/4)・(logn)^2
   X=logn (つまり t=n )のとき最大値 A(n)=(1/2)・(logn)^2
をとる。
(2)は自分でやってください
517516:01/09/16 23:26
被積分関数に絶対値が含まれる場合は、真っ先に絶対値をハズすことを考えよう。
この場合は log(t/x)が0以上か0以下で場合分けします。
518505:01/09/16 23:30
みなさんご解答ありがとうございます。
さっそく拝見させていただきます
519132人目の素数さん:01/09/17 00:01
底面の円の半径がr、高さがhの円柱を考える。
この円柱表面上に任意に4点をとり、それらを頂点とする四面体を
つくるとき、この四面体の体積の最大値を求めよ。

てのつけ方もわかりません
520132人目の素数さん:01/09/17 00:56
体積最大をとる四面体の頂点をABCDとする。(注 >>521)

3点ABCを含む平面をPとする。
点Dは円柱の表面上で、平面Pからの距離が最も大きくなる点である。

(1)Pが底面と平行なとき
Dはどちらかの底面の上にある。その円板上では、Dを動かしても四面体の面積は
変わらないので、Dを円板の円周上に動かすことが出来る。

(2)Pが底面と垂直なとき
Dは、底面上でPから一番遠い点Eを通り底面に垂直な線分の上にある。その線分上では
Dを動かしても四面体の面積は変わらないので、Dを円板の円周上に動かすことが出来る。

(3)その他のとき
このときは、Dはどちらかの底面の円周上にある。

以上より、点Dが円周上にあるとしてもよいことがわかった。
同様の議論により、点ABCも円周上にあるとしてよい。
4点が二つの円周上に何個ずつわかれているかによって場合分け

(A)4-0のとき
体積は0

(B)3-1のとき
このとき体積は 1/3 × (円周に内接する三角形の面積) × h である。
円周に内接する三角形の面積が最大になるのは正三角形のときで、
このとき
体積 = 1/3 × (3√3 r^2 / 4 ) × h = (√3 / 4) h r^2

(C)2-2のとき
一方の円周上にAB、もう一方の円周上にCDがあるとする。
ABをもう一方の円周に正射影した像をA'B'とする。
Dは平面P(三点ABCを含む平面)から最も遠い位置にあるので、
点Dは 弧A'B'(点Cを含まない方) の中点であることがわかる。
同様にして、点Cは、弧A'B'(点Cを含む側)の中点である。
以上より、線分CDは円周の直径であり、直線A'B'に直交することがわかった。

同様にして、線分ABも円周の直径である。
ABの中点をMとすると、

四面体ABCDの体積
= 2 × (三角錐 A-MCDの体積)
= 2 × (1/3 × r × (1/2 × 2r × h))
= 2/3 h r^2

(√3 / 4) h r^2 < 2/3 h r^2
なので、体積の最大値は 2/3 h r^2 。
521132人目の素数さん:01/09/17 00:56

このような四面体が存在することは別に証明が必要。
「コンパクト集合からの連続関数は最大値を持つ」
ことからわかる。
52214:01/09/17 01:04
>>520
オレが東大入試・・・スレで書いた方針を実行してるだけじゃん。
523520:01/09/17 01:10
>522
ホントだ…。かぶった。
鬱だ氏能。
524131人目の虚数さん:01/09/17 02:50
exp(2kπi)=1 (読みにくいんで()内は、2 ケー パイ アイ)
kは整数、iは虚数単位。
この式の意味がわかりません。
誰か教えてください。
525 :01/09/17 03:10
>>524
オイラーの公式から明らか

◆オイラーの公式◆
exp(iθ)=cosθ+i・sinθ

この式に θ=2kπ (k:整数)を代入すると
cos(2kπ)=1 ,sin(2kπ)=0 より
exp(2kπi)=1
526131人目の虚数さん:01/09/17 03:15
524です。質問の意味がわかりにくかったので,詳しく書きます。
量子力学、波動関数に関する質問で,
1次元の箱(0<x<L)の話なんですけど,
波動関数の一般解
ψ(x)=aexp(ifx) + bexp(ifL)
で、両端(x=0,L)のとき、ψ(確率)は0となり、
ψ(L)=aexp(ifL) + bexp(ifL)=0
ψ(0)=a+b=0 と書ける。
( f=√( (8π^2)*m*E / h^2) )

で、Eの場合わけをして, 0<E のとき、
exp2(ifL)=1
ってなって、
ここから上の質問の exp2(kπi)は1だから、
exp2(ifL)=exp2(kπi) より
fL=nπ (nは整数)っていうのなんですけど、

exp(2kπi)=1 → lnexp(2kπi)=ln1
→2kπi=0 → 0+2kπi=0 ??? ってなちゃいました。
誰か教えてください。
お願いします。
527131人目の虚数さん:01/09/17 03:18
525さん。
ありがとうございます。
答えてくれてたの知らなくて,
たらたら、526をかいてました。。。。。
528 :01/09/17 03:25
>>526
俺も物理科出身だけど
>exp(2kπi)=1 → lnexp(2kπi)=ln1 →2kπi=0 → 0+2kπi=0 ???
がまちがえてるよ。
lnexp(2kπi)=ln1 ⇔ 2kπi=1
529528:01/09/17 03:33
>>526
ごめん。あってた。
530131人目の虚数さん:01/09/17 03:34
何度もすいません。
ln1 = 1 ですか?
531131人目の虚数さん:01/09/17 03:35
ほい
>>526
exp(a)=exp(b)<=>a=b+2kπi(k∈Z)
であって
exp(a)=exp(b)<=>a=b
ではない。
sinπ=sin0からπ=0と言えないことと同じ。
533最近院試終了:01/09/17 04:13
今年の京大数学系の院試の問題です.
もう合格発表も終わったんですが全然分からないのでお願いします.
 標数0の可換体Fの代数的閉包をF^と記す.F^の部分体Lに対してF^に含まれ
るLの有限次アーベル拡大体全ての合成体をLabと記す.
 F^に含まれるFの全ての有限次拡大体Kに対して、KabはFabとKの合成体に
なるという性質を体Fが持つとする.
 このとき、Fab=F^ であることを示せ.
534131人目の虚数さん:01/09/17 04:26
528さん、532さん
ありがとうございました。
また、よろしくお願いします。
535132人目の素数さん:01/09/17 05:21
>>520
すごく分かり易い、要領のいい解答ですね。
感心しました。
で、大学入試でこんな問題が出題されたとき
『円周に内接する三角形の面積が最大になるのは正三角形のとき』
っていうのは証明無しに使っていいのでしょうか?
536520:01/09/17 06:07
>535
『円周に内接する三角形の面積が最大になるのは正三角形のとき』
大学入試なら証明は必要だとおもわれ。
537132人目の素数さん:01/09/17 07:42
>>535
ほとんど自明だから使っていいよ
538132人目の素数さん:01/09/17 08:11
>>533
これでだう?
Fab=F'≠F^とする。LをF'の有限次Galois拡大でAをGal(L/F')の単位群でない
アーベル部分群、KをAの固定体とする。このときLはKのアーベル拡大なので
L⊂Kab。しかしKF'=KはLをふくまない。
539132人目の素数さん:01/09/17 08:51
>>535
>『円周に内接する三角形の面積が最大になるのは正三角形のとき』
>っていうのは証明無しに使っていいのでしょうか?

いいのかいけないのか知らないが
↓これと同じ話だとゆうことを一応言っておく(藁

>α,β,γはα>0,β>0,γ>0,α+β+γ=πを満たすものとする。
>このときsinαsinβsinγの最大値を求めよ。
540132人目の素数さん:01/09/17 09:00
>>539
そんなことせんでも、絵を描けばわかるよ
背理法で長さの異なる2辺を選んでもう一辺は底辺として固定すれば
もっと面積の大きい三角形が存在することが
541132人目の素数さん:01/09/17 09:11
>>540
面積最大の三角形が存在するってのは自明なの?
コンパクト性とかを使うの?
542整数論:01/09/17 10:39
1. x^2-3x-3を(poly mod5)で1次式に分解せよ。
2. p-j≡-j(modp)を利用して、pが奇素数p=2k+1であるとき、
   (p-1)!≡(-1)^k((p-1/2)!)^2 (modp)を示せ。
3. n>4の合成数のとき、nl(n-1)!を示せ。これから、(n-1)!≡-1 (mod n)
ではないことを導け。

だれか頭の良い方、お願いします。m(- -)m
543132人目の素数さん:01/09/17 11:09
(x-a)(x-b)=x^2-(a+b)x+abとおいて
a+b≡3 (mod 5), ab≡-3≡2 (mod 5)を求めるだけでしょ。
544132人目の素数さん:01/09/17 12:39
>>542
3. p を素数として、n=ap^k, k≧1 とすると (n-1)! の中の p の個数は
(i=1→∞)[(ap^k-1)/p^i] ([ ] はガウス記号)
≧(i=1→k)(ap^(k-i)-1)=a(p^k-1)/(p-1)-k
≧k{((k-1)(p-1)/2+1)a-1}。n>4 が合成数なら、これは ≧k
従って nl(n-1)!であって、(n-1)!≡0 (mod n)
545最近院試終了:01/09/17 13:31
>538
ありがとうございます.
でも、KはFの有限次拡大体でのみ、この性質をもつので、そのKに対しては
使えないと思います.
もしかしたら、無限次拡大体についても成立する事が示せるのか。。。とも思い
ますが。。。うーんと。。。無理っぽいかな.
>542
ij=1(modp) の時、(p-i)(p-j)=1(modp)
を使えばすぐ出そうですね
>>542
3はnの素因数をpとしてn=p^2とn≠p^2に分ければできる。
548132人目の素数さん:01/09/17 17:44
>>541
何を聞かれてるのかよくわからないのだけど
正三角形であることが必要条件なのはOK?
549整数論:01/09/17 18:01
>>543,544,546,547さん、ありがとうございます。
2.と3.の模範解答としてはどのように書いたらよいですか?
分かりやすく書いていただけたらと思います。
私には非常に難しくてやっと理解できる程度なので。
550整数論2:01/09/17 18:07
合同式x^2 -1≡0 (mod p)を利用して、
(a,b)=1ならば、

 a^{(p-1)/2}≡±1 (mod p)

であることを示せ。
551お願いします:01/09/17 18:26
楕円曲線について詳しい方有限体F_q(q=p^m)上の楕円曲線E(F_q)をy^2=x^3+ax+bとし、
楕円曲線上の点をP、Rとする。このとき、P =l Rとなる整数 l を求める問題で、
Weil対を使って楕円曲線上の点を有限体上の点に置き換えて求める方法の
アルゴリズムを教えてください。

よろしくお願いします。
552132人目の素数さん:01/09/17 18:46
AをKの元を成分とするn×n行列、(v(1),....,v(n))をK^n の基底とする。
このとき、(Av(1),....A,v(n))がK^n の基底となるための必要十分条件は
Aが正則であることを証明してください。
553>549:01/09/17 18:48
1*2*…(p-1)=1*2*…*k*(k+1)*…(p-1)
=1*2*…*k*(p-k)*…*(p-1)
=k!^2*(-1)~k

という感じでいいのでは?ちなみに等号が全てmodp
554>549:01/09/17 18:55
3.
はnを素因数分解したその各因子p^aが(n-1)!を割り切ることを示せばよい.
(n-1)!を素因数分解した時のpの指数がk=[n-1/p]+[n-1/p^2]+…
で与えられるので、k≧aを示せばよい.
あとはp=2の時、p≠2で場合分けすればほとんど明らか.
555>552:01/09/17 19:00
(v1,v2,…,vn)=B を行列だと思えば、
(Av1,Av2,…,Avn)=AB これが基底になるので、ABの行列式は≠0
よって、Aの行列式も≠0

。。。というより、当たり前過ぎて何を使っていいのやら。。。
行列式は使っていいのかな??
それとももっと1次独立の定義を用いて示さないとダメなんかな?
556552:01/09/17 19:07
行列式は使わないで下さい。
1次独立の定義を用いて示してください。
557整数論:01/09/17 19:32
>>553-554さん
ありがとうございます!!m(- -)m
558先生、むずいです!:01/09/17 20:46
この問題どうやって解くのでしょうか?

問題4
R^2上の任意のC^∞ vector fieldは完備か?証明つきで答えよ。

問題5
g>0とする。symplectic group
Sp(2g;R)={A∈GL(2g;R);tAJpA=Jp}
がGL(2g;R)の部分多様体である事を示せ。ただし、
tAはAの転置行列
Jp= (0 1g)
  (-1g 0)

1gはg×g単位行列
どっかで見たことある問題だと思ったら・・・・
560132人目の素数さん:01/09/17 21:02
>>545ほんとだ。>>538は不充分だった。むづい。
つまりF<K<F^とx∈F^について

 (0)x∈Kab
 ⇔(1)∃L st. K儉,Gal(L/K)がabelian,x∈L
 ⇔(2)F<∃K'<∃L'<F^ st. K'儉',Gal(L'/K')がabelian,[K':F]<∞,x∈L'

を示さんとだめなのね。(A傳はBがAの有限次Galois拡大を意味するとする。)
(0)⇔(1)は定義。
(1)⇒(2)
(1)をみたすx,Kにたいし[K':F]<∞,K'儉',x∈L'となるK',L'をとる。
Gal(L'/K')がabelianであることをしめせばよい。
s∈Gal(L'/K')をとる。sはGal(L/K)の元に拡張される。
逆にs∈Gal(L/K)の元を自然にL'に制限するとs(K')=K'(∵F僵')
かつs(k)=k(∀k∈K)よりs∈Gal(L'/K')。
つまりGal(L/K)→Gal(L'/K')が制限写像としてwell-definedで全射。
∴Gal(L'/K')はabelian。
(2)⇒(1)
(2)をみたすx,Kに対しL=K(L')とおく。Gal(L/K)がabelianであることを
示せばよい。任意のs∈Gal(F^/F)にたいしs(L)=LゆえK儉。
さきほどと同様にGal(L/K)→Gal(L'/K')が制限写像で定義できるが
L=K[L']ゆえこれは単射。∴Gal(L/K)はabelian。

でだう?
561工房:01/09/17 21:03

            ∩
               | |
               | |
               | |
              | |
        ∧_∧   | |   / ̄ ̄ ̄ ̄ ̄ ̄ ̄ ̄ ̄ ̄ ̄ ̄ ̄
       (´∀`)//  < 先生!実数a,bを係数と
                 する2つの2次方程式X2×aX+b=0...(A)
X2×bX+a=0(B)...
      /       /     \について(A)(B)が共通解を
                  持つための条件をa,bを用いて表せってのが
                  わかりません!!!__
     / /|    /  
  __| | .|    |
  \   ̄ ̄ ̄ ̄ ̄ ̄ ̄\
  ||\             \
  ||\|| ̄ ̄ ̄ ̄ ̄ ̄ ̄|| ̄
  ||  || ̄ ̄ ̄ ̄ ̄ ̄ ̄||
     .||              ||
562工房:01/09/17 21:05
ちなみにXの後の2って言う文字は2乗の2です。
参考書で探したのですが、似たような問題ないんでぜひ
お願いします!!
563132人目の素数さん:01/09/17 21:09
[京都大学]
次の不等式を示せ
(1)0<a≦xのとき∫(a→x){e^(-t^2/2)}dt≦{(1/a)・e^(-a^2/2)}-{(1/x)・e^(-x^2/2)}
(2)3<bのとき∫(3→b){e^((-t^2/2)+2t)}dt<e^(3/2)

すいません。
この京大の問題を質問させてください
おねがいします
564132人目の素数さん:01/09/17 21:23
>>561-562
2次方程式ってことは
x^2+ax+b=0...(*)
x^2+bx+a=0...(**)
のまちがい?だったらこれが実数解もつのは
x^2+ax+b=0...(*)
(a-b)(x-1)...(***)=(*)-(**)
が実数解もてばいい。
(i)a=bのとき(***)の解はすべての実数。よって
このとき(*)が実数解をもつことがもとめる条件。
(ii)a≠bのとき(***)の解はx=1のみ。よって
x=1が(*)の解であることがもとめる条件。
あとやってみそ。
>>564
実数解を持つという条件は無い
566工房:01/09/17 21:32
ありがとうございます!!!
やってみます
567132人目の素数さん:01/09/17 21:42
よけいな事をしてしまったために混乱を招いたようです。
申し訳ないです。ではもっかい書きます。

実数a,bを係数とする2つの2次方程式x^2+ax+b=0...(A)
x^2+bx+a=0...(B)について

(1)(A)(B)が共通解を持つための条件をa,bを用いて表せ

(2)(A)と(B)がそれぞれ相異なる実数解を持ち、
しかも共通解をもたず、かつ(A)の解の間に(B)の解の
うちの一つだけが存在するための条件をaとbを用いて表せ。

すみません。もう一問追加です。ほんまにわからん・・・・
568>560:01/09/17 21:50
それで、F'=F^ を示してることになるん?
それがまず分からなかったことと、
K'は一般にFの無限次拡大のはずだから、(1)⇒(2)で、[K':F]<∞
となるK'は簡単にとれないはず.
あと記号の問題ですが、K'=Kab とおいてるんですよね?
569>563:01/09/17 21:52
微分するだけでは?
570>567:01/09/17 22:04
564さんの言う通りです.
かみくだいて言えば、共通解が存在する
⇔x^2+ax+b=0(1) x^2+bx+a=0(2) となる実数xが存在する.
⇔(a-b)(x-1)=0 ((1)-(2)) x^2+ax+b=0 となる実数xが存在する.
⇔「a=b又はx=1」かつx^2+ax+b=0
とすればいいんやけど、もしかして問題文に「a≠b」ってついてない?
a=bのときはめんどいし.
a≠bのときはx=1かつx^2+ax+b=0 だから1+a+b=0 が求める条件

後半は、1つの解がx=1であることと解と係数の関係を使えば一発
571132人目の素数さん:01/09/17 22:08
>>520
>(3)その他のとき
>このときは、Dはどちらかの底面の円周上にある。

これは証明なしに使ってういいの?
572132人目の素数さん:01/09/17 22:13
>>568
>あと記号の問題ですが、K'=Kab とおいてるんですよね?
ちがうよ。>>560でしめしているのは

 Kab=∪[K'はFの有限次拡大]K'ab

という等式。これがいえれば任意のF⊂K⊂F^に対し仮定から

 Kab
 =∪[K'はFの有限次拡大,K'⊂K]K'ab
 =∪[K'はFの有限次拡大,K'⊂K]K'Fab
 =KFab

がいえることになって以下>>538が通用するんじゃない?

>K'は一般にFの無限次拡大のはずだから、(1)⇒(2)で、[K':F]<∞
>となるK'は簡単にとれないはず.

xはK上代数的だから最小多項式P(t)∈K[t]をとる。P(t)の各係数をすべて
ふくむFの有限次拡大のGalois閉包をK'、L'はK'(x)のGalois閉包をとれば
いいとおもう。
あと(2)の条件のなかにF僵'がいるね。証明のなかでかってにつかってる。
>(∵F僵')
ってとこ。スマソ。
573132人目の素数さん:01/09/17 22:15
>>572
訂正。
×:Kab=∪[K'はFの有限次拡大]K'ab
○:Kab=∪[K'はFの有限次拡大,K'⊂K]K'ab
574算数がわからない人:01/09/17 22:26
10個の500円玉と2個の10円玉が財布の中に・・
これを無造作に取り出し1列に並べた際、10円玉が隣同士になる確立は?
教えてくださいませm(__)m
575算数がわからない人:01/09/17 22:31
もひとつ!
1から9までの数字を使って5桁の数字を作る時、
奇数の数字を1個だけ含むようにして、かつひとつの数字の組み合わせで
同じ数字は使えないとすると・・・
何通りの整数ができますか?
576132人目の素数さん:01/09/17 22:38
>>574
10円玉2枚を1つ(20円玉)と考えると、20円玉1枚と500円玉10枚を1列に並べる場合の数は11!*2
(*2は20円玉の作り方に2通りあるから)
これが10円玉が隣り同士になる場合の数。

一方、10円玉2枚と500円玉10枚を1列に並べる場合の数は12!
これが全ての場合の数。

よって
(求める確率)=(10円玉が隣り同士になる場合の数)/(全ての場合の数)
だから
11!/12! = 1/12
577>572:01/09/17 22:38
オッケーです.
538でFab=F'と書いてたので勘違いしてました.
560の所はまだ分からないところもありますが、同じ考え方(多分)自分なりに解けました.
ありがとうございます.
578576:01/09/17 22:39
まちがえた。
11!*2/12! = 1/6
579算数がわからない人:01/09/17 22:42
576さん ありがとー(^o^)丿
580132人目の素数さん:01/09/17 22:46
>>548
正三角形であることが必要条件なのはOKだけど
正三角形のときに面積が最大になることはどう示すの?
581132人目の素数さん:01/09/17 22:47
>>575
1〜9までの自然数の中で
奇数は 1、3、5、7、9 の5つ
偶数は 2、4、6、8 の4つ
奇数は1つしか使えないので5桁の数をつくるとき偶数は全部使わないといけない。
よって
(求める場合の数)=(4つの偶数を1列に並べる場合の数)*(奇数の選び方)*(奇数の入る場所の数)
         =4!*5*5
         =600
582576:01/09/17 23:02
>>579
正解は >>578 の方だかんね。
583132人目の素数さん:01/09/17 23:33
>>563
これは、、一筋ではいかなくない?
簡単そうに見えたがいまだ示せず
>>580
半径rの円に内接する△ABCに対して、まず弦BCを固定。
△ABCの内部に円の中心、かつ、AB=ACかつのとき△ABCが最大。
次に0<|BC|=x<=2rで弦BC=xを動かして△ABC=f(x)の増減を見る。

ベタでスマソ
585132人目の素数さん:01/09/17 23:46
>>563
(1) e^(-t^2/2)<(1+t^2/t^2)e^(-t^2/2)
をa→xで積分すると不等式がでてくる。(2)は(1)の応用。
∬変数を置換せよ。式は解読してくれ。
>>585
>(1) e^(-t^2/2)<(1+t^2/t^2)e^(-t^2/2)

右辺が出てきた理由を書いたほうがいいべさ。

F(t)=(1/t)・e^(-t^2/2)を微分すると
f(t)=-((1+t^2)/t^2)e^(-t^2/2)

{(1/a)・e^(-a^2/2)}-{(1/x)・e^(-x^2/2)}
=∫[a,x]f(t)dt
587工房:01/09/18 00:36
>564さん,570さん
ありがとう
588致死性家族性不眠症:01/09/18 00:40
>>563

(2)
∫(3→b){e^((-t^2/2)+2t)}dt
=∫(3→b){e^(-1/2(t-2)^2 + 2)}dt
=e^2 ・∫(3→b){e^(-(t-2)^2/2)}dt
(ここで t-2=u とおくことにより)
= e^2・∫(1→b-2){e^(-u^2/2)}du ・・・(☆)
となる。
ここで(1)の不等式を用いると、3<bにおいて、
∫(1→b-2){e^(-u^2/2)}du
≦{e^(-1/2)}-{(1/x)・e^(-x^2/2)}
<{e^(-1/2)}
が成り立つので、3<b において
(☆)<e^2・{e^(-1/2)}=e^(3/2)
が成り立つことが示された。
589工房:01/09/18 00:56
>570
「a≠b」はついてないです。
>>567
(1)a=bまたはa+b+1=0。
(2)a≠bかつa+b+1<0。
591大学3年:01/09/18 05:45
高校の数IIIでy=2x-e^xのグラフを書かせる問題があったんですが、
このグラフのx軸との交点って求められるんですか?
もちろん2x-e^x=0を解けばいいって言うのはわかるんですが、いざ解こうと思うと全然だめで・・・
解法つきでどなたか解説していただけないでしょうか。
592132個目の素数さん:01/09/18 06:06
確率と組み合わせの複雑な問題が分かりません...。(-_-;)

1,2,3,4,5,6,7,8,9,10
の番号が割り当てられたカードがあるとして、
それぞれをひく確率が
a,b,c,d,e,f,g,h,i,j
(1>a>b>c>d>e>f>g>h>i>j>0 , a+b+c+d+e+f+g+h+i+j = 1)
と与えられています。
ここで、3枚のカードを選びその
3枚のカードの数字が 1,2,3 (順不問)である
確率はどういった式で表されるのでしょうか?
593132人目の素数さん:01/09/18 06:54
>>591
その場合、x軸との交点なんて求める必要はありません。
まあ、y=2xとy=e^xの交点はグラフを描けばx>0の範囲に
2つあることが分かるのでそれを守って書いていればOKです。
594132人目の素数さん:01/09/18 07:18
(1)位数18のアーベル群を同型のものは同一視して分類せよ
(2)Z/2Z係数の既約な4次の多項式をすべて求めよ。
>>593
x の方程式 ax - e^x = 0 が実数解を持つaの条件は a >= e ( > 2) だから
y = 2x と y = e^x は交わらないと思われ

仮に 3x - e^x = 0 だとしても解けないけど
596???????:01/09/18 07:40
>>592
1枚ずつ順番にカードを取り出し、取り出した順番は考慮するものとする。
10枚の中から@を選ぶ確率はaだが
Aが欠けている9枚の中から@を選ぶ確率は a/(a+c+d+e+f+g+h+i+j)
Bが欠けている9枚の中から@を選ぶ確率は a/(a+b+d+e+f+g+h+i+j)
@Bが欠けている8枚の中からAを選ぶ確率は b/(b+d+e+f+g+h+i+j)
などと(あるカードを引く確率)/(引くときに残っているカードを引く確率をそれぞれ足したもの)
となるから、このことに注意して計算する。

@→A→Bの順で取り出す場合
(@→A→Bの順で取り出す確率)=
(10個の中から@を取り出す確率)*(9個の中からAを取り出す確率)*(8個の中からBを取り出す確率)
であるから、
a*{b/(b+c+d+e+f+g+h+i+j)}*{c/(c+d+e+f+g+h+i+j)}=abc/{(1−a)(1−a−b)}
(上式の左辺→右辺に変形するときに a+b+c+d+e+f+g+h+i+j = 1を用いた)

同様に
@→B→Aの順で取り出す場合 abc/{(1−a)(1−a−c)}
A→@→Bの順で取り出す場合 abc/{(1−b)(1−a−b)}
A→B→@の順で取り出す場合 abc/{(1−b)(1−b−c)}
B→@→Aの順で取り出す場合 abc/{(1−c)(1−a−c)}
B→A→@の順で取り出す場合 abc/{(1−c)(1−b−c)}
となるので、
求める確率はこれらの全てを足したものであるので後は計算する。
597132人目の素数さん:01/09/18 07:50
>>594
(1)有限生成アーベル群の分類定理から求めるものはZ/p^eZの形の直和
よって

 Z/2Z○Z/3Z○Z/3Z or Z/2Z○Z/9Z (○は直和)

(2)既約な二次式はx^2+x+1しかない。よって既約2次×既約2次
の形のものはx^4+x^2+1しかない。
また一次因子をもつ⇔x=0orx=1で0になる⇔定数項が0or0でない係数が偶数個
だから結局
既約4次⇔x^4+x^2+1でないand定数項が0でないand0でない係数が奇数個
よって

 1+x+x^4 1+x^3+x-4
598593:01/09/18 07:57
>>595
本当だ。寝ぼけててうっかりしてた。
よく考えたらln2は0より小さいし、このグラフ、x軸なんかと交わらないね。
>>591 で交わるみたいなこと書いてたから、勘違いしちゃった。(えへッ
言い訳です。
599598:01/09/18 07:59
またまちがえた。
ln2は0より大きいよね。ゴメン。寝ます。
600593:01/09/18 08:11
>>591
最大値 2ln2−2 が0より小さいので当然x軸とは交わらない。
601132人目の素数さん:01/09/18 10:56
教えてください!!

Mをmo"bius band とし、
TMをMの接vector全体とする時
TMとM×R^2とは微分同相になりますか?
602132人目の素数さん:01/09/18 11:31
>>601
ならない。TMとM×R^2が微分同相なら、Mが向き付け可能になる
ことに注意。
603132人目の素数さん:01/09/18 12:10
>>602
有難う御座います。
多様体Mに対して同様に考えて
「向き付け可能<==>微分同相」
は成り立つのでしょうか?それとも
「微分同相==>向き付け可能」
しか成り立たないのでしょうか?
604132人目の素数さん:01/09/18 12:24
>>603
子ホモ路地−をやると証明できるんだけど、
向き付け可能多様体の接バンドルが自明(R^nとの直積)とは
限らないです。たとえば、球面S^2の接バンドルは自明じゃないです。(不動店定理を思い出して)。
結論は微分同相⇒向き付け可しかいえません。
605132人目の素数さん:01/09/18 12:25
>>603
「Mが向き付け可能<==>T(M)がM×R^nと微分同相」
のことをきいてるの?なら全然だめだよ。
右側の条件は“平行多様体”といってそうでないのはいっぱいあるよ。
(S^2など。)微分位相幾何学、田村、岩波等参照。
606132人目の素数さん:01/09/18 12:26
>>604
カブスマ
607132人目の素数さん:01/09/18 12:35
>>604-605
有難う御座います!!
608高校3年:01/09/18 17:27
a,b,を実数とする。 関数f(x)=x^3+ax^2+1/3bx-1/3(a^2+3a+2)
は極大値と極小値とを持ち、その差は4/27である。
(1) aとbの関係式を求めよ。
(2) 曲線y=f(x)はaの値によらず1つの定点を通ることを示せ、また、その定点を求めよ。
(3) 方程式f(x)=0が重解を持つようなaの値を求めよ。
609高校3年:01/09/18 17:32
aは負の定数とする。関数f(x)=2x^3-3(a+1)x^2+6ax の区間 -2<=x<=2
における最大値、最小値を求めよ。

以上の2問ですが私はまったく分かりませんでした。誰か解いてください(願)。
610厨×廚:01/09/18 19:12
「立方体ABCD-EFGHのA,C,EHの中点,GHの中点を結んでできる
四角形の面積」は一辺の長さの有理数倍になりますが,
そのため小学生でもこれを求められるそうです。
やり方教えてください。リアル消防歓迎。
611算数がわからない人:01/09/18 20:07
大きな三角形があります。わかっているのは 2つの辺が
18cmと33cm で その間の角が 90度ということです。
他の 2つの角度 と 辺の長さをそれぞれ求めなさい

これ 誰か答え下さい お願い申し上げます m(__)m 〜 
612591:01/09/18 20:12
私が聞きたいのは
e^x+ax+b=0 (a,bは実数の定数)
のような形の方程式の解を求めることはできるのかということなんですよ。
グラフがどうのこうのと書いてしまったので、皆さんの誤解を招いてしまったようでしたが・・・
どなたか詳しい解説つきでお願いいたします。
613132人目の素数さん:01/09/18 20:14
>>611
余弦定理つかえばあっさりと解けるんでないの?
614算数がわからない人:01/09/18 20:21
余弦定理がわかりません(^_^;)
>>612
無理。解説略。
616132人目の素数さん:01/09/18 20:35
>>612
4次以上の方程式の一般解は存在しないから無理。
で、いいのか?
617132人目の素数さん:01/09/18 20:39
>>616

4次は解けるで
618132人目の素数さん:01/09/18 20:42
>616
>4次以上の

よくない
>617
>618
あれ、五次以上だっけ、スマソ。
620132人目の素数さん:01/09/18 20:50
>>611
正弦定理のほうがよくない?
斜辺は三平方の定理で√(18^2+33^2)
残りの角は
√(18^2+33^2)/sin(π/2)=18/sinx=22/siny
621132人目の素数さん:01/09/18 21:15
>>608

(1) a^2−b=1
(2) 定点(1,0)を通る
(3) a=−5,−2,−1
622621:01/09/18 21:30
(1)は極大値をとるxをx_1、極小値をとるxをx_2とすると
y=f(x)は3次関数でx^3の係数が正であることを考慮して
x_1<x_2という条件でf'(x)=0を解くと
x_1={−a−sqrt(a^2−b)}/3 ,x_2={−a+sqrt(a^2−b)}/3
となり、
4/27=f(x_1)−f(x_2)
を要領よく計算して解けばOK。

(2)は「aの値に関わらず」→「aについての恒等式」を思い浮かべ
f(x)をaについて整理して、a^2とa^1の係数が同時に0になるとして
解けばよい。

(3)は(2)よりy=f(x)はx軸上の点(1,0)を通ることが分かったので、
点(1,0)でx軸に接する場合と、点(1,0)で交わる場合に分けて考える。
前者だとy=(x−1)^2・(x−α) …@
後者だとy=(x−1)・(x−α)^2 …A
と書けることに注意して(αは別の解)
@式、A式をそれぞれ展開したものと、もとのy=f(x)の係数をそれぞれ
比較して解く。

>>621 の答えはざっと計算したのでひょっとしたら間違ってるかもしれないけど…
623132人目の素数さん:01/09/18 21:46
>>558を教えてください。
むずいっす。
624工房:01/09/18 21:58
561の問題(2)をやっていたのですが、
まず、判別式で(A)>0、(B)>0で
異なる二つの実数解もつ。次に、共通解をもたないので、
b≠a b≠-a-1
そして(A)の解の間に(B)の解のうちの一つだけが存在するので
(B)をf(x)=x^2+bx+aとおいて、(A)の解をα、βとおく
解と係数の関係式よりα+β=-a αβ=b
f(α)×f(β)<0
で(α^2+bα+a)(β^2+bβ+a)<0
となりますよね。問題はここなんです。
ほんとに初歩的で申し訳無いんですが、どうやってαとβを消去
できるのでしょうか?展開してから解と係数の関係式を
いれても消せないし困ってます。お願いします
625621:01/09/18 22:05
>>609>>608 とちょっと似たような感じだけど
f(x)を微分するとa<0の条件から、y=f(x)のグラフの概形は分かる(x=aで極大値、x=1で極小値)ので
あとはaの値について場合分けする。

@)−2≦a<0 のとき
 x=2で最大値をとる(f(2)>f(a)のとき)か、x=aで最大値をとる(f(2)≦f(a)のとき)。
x=1で最小値をとる(f(1)<f(−2)のとき)かx=−2のとき最小値をとる(f(1)≧f(−2)のとき)。

A)a≦−2 のとき
 x=2で最大値をとる(f(2)>f(−2)のとき)か、x=−2のとき最大値をとる(f(2)≦f(−2)のとき)。
 x=1で最小値をとる。

あとは@)A)をまとめればOK。
626132人目の素数さん:01/09/18 22:22
問題ではないのですが、シグマ集合族の定義について教えて下さい。
イメージが全然わかないので、困っています。

標本空間Ωの部分集合族の1つがσ集合族なのに、
どうしてΩもまたσ集合族に属するのでしょうか。
「Ω=σ集合族」ではないようですし、
Ωとσ集合族の関係がよく把握できないのです。
どうぞ、よろしくお願いします。
627132人目の素数さん:01/09/18 22:23
>>624
たぶんそれでもできるけどグラフを考えたほうが楽だよ。
f(x)=x^2+ax+bとg(x)=x^2+bx+aはa≠bのときx=1,y=a+b+1
で交差する。“それぞれの2解が入れ子になる⇔交点がx軸の下”
に気づけばあとは楽。
628621=622:01/09/18 22:27
>>622

>4/27=f(x_1)−f(x_2)
>を要領よく計算して解けばOK。

というのは、4/27=f(x_1)−f(x_2)はx_1とx_2についての対照式なので
f'(x)=0から解と係数の関係からx_1+x_2とx_1*x_2を求めてから計算すると
計算が楽と言うことです。
念のため…。
629621=622:01/09/18 22:29
>>628
あっ、厳密には対照式ではないか。。。
でもx_1+x_2とx_1*x_2で表せるので大丈夫。
630132人目の素数さん:01/09/18 22:32
>>623
問題4を教科書(=多様体、服部、岩波)よみながら挑戦。
Noが答えでは?R^2はU={(x,y);x^2+y^2<1}と同相なので
U上のベクトル場が常に完備か?と同値だけどR^2上のベクトル場
X(x,y)=(x,y)をかんがえるとexptX(x,y;t)=(xexpt,yexpt)
となるけどこれはU上で閉じていないのでXはU上で
完備ではないのでは?
631132人目の素数さん:01/09/18 22:36
>>627
“f(x)=0,g(x)=0の2解が入れ子になる⇔y=f(x),y=g(x)の交点がx軸の下”
といわんとおかしいね。訂正。
632132人目の素数さん:01/09/18 22:48
プログラムを作る上で考えてみたのですが
どうしてもわからないので教えてください!

正弦波の曲線上を半径Rの円が転がるときに、
その円の中心点の軌跡を表す関数というものは
どのように表されるのでしょうか?
633>626:01/09/18 23:03
σ集合族ってのは、ある条件を満たす集合全体のことで、
(よく覚えてないけど加算無限の加法性のこと?)
Ωの部分集合でもその条件を満たすものがあった、ということでしょう.
6341234:01/09/18 23:24
わからないので教えてください。

  次の各問いに答えよ。ただし、対数はeを底とする自然対数である。
(1)方程式 logx+x+1=0 は 0<x<1/e の範囲にただ1つの実数解をもつことを示せ。
(2)f(x)=y={x/(1-x)}^x (0<x<1) は 0<x<1/(e+1) の範囲に極小値をもつことを示せ。

(1)は解けたのですが、(2)でつまずきました。
(2)は、まず両辺に自然対数をとってその後両辺をxで微分してみたのですが計算がおかしくなってしまいました。
(1)はいいので(2)を教えてください。もし僕のやり方と違うやり方で簡単な方法があれば、そちらの方も教えてください。
635>624:01/09/18 23:24
次数下げをしたらいいのでは?
別に展開してからでも代入できると思うけどね
636>626:01/09/18 23:32
Ωも「Ωの部分集合」のうちのひとつなんだから、条件を満たしていれば
σ加法族(完全加法族)に属します。
定義
φ∈σ加法族
A∈σ加法族⇒Aの補集合∈σ加法族
からφの補集合Ω∈σ加法族がいえます。
637あん:01/09/18 23:35
アレキサンドロフ空間の具体的な例を教えてください。
638132人目の素数さん:01/09/19 00:00
>>634
>(2)は、まず両辺に自然対数をとってその後両辺をxで微分してみたのですが計算がおかしくなってしまいました。

それでやってもオカシくなんかならないでしょ?
log(y)=x log(x/(1−x))
の両辺をxについて微分すると
(1/y)・(dy/dx)=log(x/(1−x))+x/(1−x)+1 ってなって
dy/dx={log(x/(1−x))+x/(1−x)+1}y
   = {log(x/(1−x))+x/(1−x)+1}{x/(1−x)}^x ← y={x/(1−x)}^x を代入

ここで 0<x<1 の範囲では {x/(1−x)}^x>0 であることは明らかなので dy/dx=0 となるためには
log(x/(1−x))+x/(1−x)+1=g(x)−h(x)=0
(g(x)≡log(x/(1−x)) ,h(x)≡−{x/(1−x)+1}とした)
でなければならないので、後はg(x)−h(x)=0、つまり g(x)=h(x) を満たす
xがどこにあるのか y=g(x)、y=h(x) のグラフを描いて色々吟味すれば大丈夫だよ。
>>637
アレキサンドロフ空間って何。
640132人目の素数さん:01/09/19 00:11
アレキサンドロス大王なら知ってるよ。
641132人目の素数さん:01/09/19 00:14
>>630
まちごう〜た。これウソだね。完備みたい。
XとYが位相空間として同相でも完備性って同値じゃないのね。
証明はXをR^2上のベクトル場とするときあるS^2上のベクトル場と
C^∞級埋め込みf:R^2→S^2があってf^*Y=X、S^2\imf上でY=0
となるものがとれることがいえる。これがいえるとS^2はコンパクト
なのでYは完備でexptYの定義域をS^2×Rでとれる。S^2\imf上でY=0
からimfとかさなる積分曲線はぜんぶimf上でとじているので
Xの完備性がいえるみたい。....たぶん今度はあってる。(かも)
6421234:01/09/19 00:23
>>638
ありがとう!わかりました。
643638:01/09/19 00:29
>>638
つーか、(1)で方程式 logx+x+1=0 は 0<x<1/e の範囲にただ1つの実数解をもつことをしめしているので
x→x/(1−x) と書き換えてやればグラフを描かなくてもOKだね。
644638:01/09/19 00:33
log(x)+(x)+1=0 は 0<x<1/e の範囲にただ1つの実数解をもつ
           ↓
log(x/(1−x))+x/(1−x)+1=0 は 0<x/(1−x)<1/e の範囲にただ1つの実数解をもつ
           ↓
log(x/(1−x))+x/(1−x)+1=0 は 0<x<1/(e+1) の範囲にただ1つの実数解をもつ
ってな感じで。
645132人目の素数さん:01/09/19 00:46
>>641
申し訳ありません、解らないです。
Yって何でしょうか?
f^*Y=Xって何でしょうか?
C^∞級埋め込みf:R^2→S^2とは、教科書に書いてあるような
S^2から一点を除いた部分への微分同相写像の事でしょうか?
ドキュソな質問で申し訳ありません。
すみませんA and B ならば C を証明したいのですが:

If A and B, then D を証明
C'とDは矛盾するよってif A and B, then C

これっって問題無いですよね。
647132人目の素数さん:01/09/19 00:54
608.(1)@ f(x)を微分する
   A f(x)をf´(x)で割ってf(x)=s*f´(x)+tの形にする
   B f´(x)=0となるxの2つの値をそれぞれu、v(u>v)とし、
    α=f(u),β=f(v)とすると
     α−β=4/27*(b-a^2)√(a^2-b)(これが差4/27)
   C(b-a^2)^3=1
    b-a^2=1
   b=a^2+1
609.f´(x)=6(x−1)(x−a)
  a<0<1<2
〔1〕a<−2のとき
    f(1)=3a−1で最小
     f(−2)=−6a−28
    {
     f(2)=4
    
     f(−2)−f(2)
    =−6a−32
    =−2(3a+16)
     f(−2)−f(2) の符号でf(−2)−f(2) の大小を決めると、(a<−2に注意)
     −16/3<a<−2のとき、−6a−28<4
    { a=−16/3のとき、−6a−28=4
     a<−16/3のとき、−6a−28>4
    
      −16/3<a<−2のとき、f(2)=4で最大

     {a=−16/3のとき、f(−2)=f(2)=4で最大
      a<−16/3のとき、f(−2)=−6a−28で最大
以下、〔1〕と同様に〔2〕a=−2、〔3〕−2<a<0の範囲で場合分けする
648132人目の素数さん:01/09/19 01:01
>>647
 608・609が既に解かれていたならば、647は削除してください。
 私は647を書いた者です。
649132人目の素数さん:01/09/19 01:05
>>646
C'とDは矛盾する==>「DならばC」
650>647:01/09/19 01:06
極大値と極小値の差って言えば、
∫f'dx を使えば計算簡単になるんが、高校のときかなり「うまいなぁ」と思ったのを思い出しました
651数学苦手君:01/09/19 01:34
ここで出すのも恥かしい問題なのですが、
数学が得意なみなさんに教えていただけたらと思い、
書き込みをさせていただきます(私は根っから文系、しかもアホ)。

ある夫婦が子作りをして、8人連続で女の子が生まれたとします。
それは何分の一の確率で生じる事態なのでしょうか?

すいませーん、教えて下さーい!!
652132人目の素数さん:01/09/19 01:48
>651
女の子が生まれる確率が毎回 1/2 で互いに独立ならば、
(1/2)^8 = 1/256 ≒ 0.4%
但し、実際には女の子が生まれやすい体質の人もいるだろうから、その場合は
確率はこれよりはるかに高くなる。

ところで、皇族の誕生は最近8人連続で女子だが、これが元ネタか?
653数学苦手君:01/09/19 01:54
>>652
そうでーす!(バレたか……)
ありがとうございました!!!!!
654612:01/09/19 02:11
5次以上の方程式に一般解はないということの証明を読めるような本があったら紹介してくれませんか?
655132人目の素数さん:01/09/19 02:17
>>654
体論の本みれば大抵載っているけど,それだけの
モチベーションで読むのは大変.
カンタンなのは
志賀浩二「数学が育っていく物語5週 方程式」(岩波書店)
656132人目の素数さん:01/09/19 02:25
>>654
それだけ載っている本があったような気がするが忘れた
657132人目の素数さん:01/09/19 02:34
>>654
高木貞治『代数学講義』という古めかしい本に
ガロア理論つかわないアーベルの証明がのってるよ。
易しい本だけど、キチンと証明が書いてある。
658132人目の素数さん:01/09/19 05:09
数学板初心者です。
今井と田中洸人は今までなにか愚行でもやったんですか?
何者なのか分かりません。
教えて下さい。
659132人目の素数さん:01/09/19 07:23
>>645
もちゃんとかくと

任意のR^2上のベクトル場Xに対しあるC^∞級埋め込みf:R^2→S^2
が存在して(R^2はS^2の部分空間とみなせるのでS^2上のベクトル場
をR^2上のベクトル場と(制限を利用してみなせる。)のでこの対応を
f:^*{S^2上のベクトル場}→{R^2上のベクトル場}とかいた。

>C^∞級埋め込みf:R^2→S^2とは、教科書に書いてあるような
>S^2から一点を除いた部分への微分同相写像の事でしょうか?

それだとだめ。(それが元のアイデアだけど。)
たとえばX(x,y)=(x,y)で定義されるベクトル場だと
lim[(x,y)→∞]X(x,y)が0に収束してくれないのでS^2上に拡張できない。
たとえばこんな場合だったらまづいったんf:(x,y)→(arctan(x),arctan(y))
などとうめこむ。Y'=f_*(X)とかくとするとf(x,y)=(u,v)として

 Y'(u,v)
 =(f_*X)(u,v)
 =(Jf)X(x,y)
 =(x/(1+x^2),y/(1+y^2))

となってこれはlim[(u,v)→∞)]Y'(u,v)=(0,0)となるのでY(∞)=0と拡張してやると
S^2上のC^∞級ベクトル場となってS^2\imf={∞}上では0になってる。
こういうfがかならずみつかるってのが>>641の主張。
つくり方は上の計算でJf(=fのヤコビ行列)が(1/(1+x^2,1/(1+y^2))がX=(x,y)
が→∞にいくスピードよりはやく0に収束するものをみつけてくればいい。
つまり正値C^∞関数a(x),b(x)を(Xのx成分)/a(x)→0、(Xのy成分)/b(x)→0
とえらんでおいてf(x,y)=(∫[0→x]a(t)dt,∫[0→x]b(t)dt)とさだめてやると
うまくいくみたい。
660132人目の素数さん:01/09/19 07:25
>>659
(Xのx成分)×a(x)→0、(Xのy成分)×b(x)→0 のまちがい。訂正。
661132人目の素数さん:01/09/19 07:29
>>660
ごめん。これでもダメだ。とにかくこんな感じ。でもたぶんこれで
いけるとおもう。
>>616
e^x+ax+b=0は代数方程式ではないし
代数的に解かなくてはいけないわけでもない。
あと5次以上の代数方程式も解は存在する。
663132人目の素数さん:01/09/19 11:50
>>661
ゴメソ。嘘八百だった。ベクトル場X(x,y)=(1+x^2,0)って完備じゃないや。
方程式
x'=1+x^2,y'=0
をとくと
x=tan(t+t_0),y=ct
は有限時間で→∞だもんね。逝ってきます。
664132人目の素数さん:01/09/19 13:04
x・lnx+y・lny=1

って解き方あります?
665132人目の素数さん:01/09/19 13:13
>>664
左辺=log(x^x*y^y)
より
x^x*y^x=e
と思われます。
666132人目の素数さん:01/09/19 13:28
>>665
どうもです。

ついでに、

煤@Xi ・lnXi =1


を教えていただけないでしょうか。Xiの値がわかりません。
667132人目の素数さん:01/09/19 14:25
>>666 条件足りなくない?
f(x)=xlnx とすると f((0,∞))=[-1/e,∞)
したがって、悩i=1, Yi≧-1/e となるように {Yi} をきめて、
それに応じて f(Xi)=Yi となる Xi をとればよい。
668132人目の素数さん:01/09/19 14:27
>>666
同じようにすれば良いのではないでしょうか?

>>665の訂正
x^x*y^y=e
スマソ。
669132人目の素数さん:01/09/19 14:46
次の問題が分かりましぇん。

二重積分∫∫[D]{1/(|x|^a +|y|^b)}dxdyが収束するような
a,bの範囲を求めよ。ただし、領域D:x^2+y^2≦1

∫∫[D]f(x,y)dxdy は f(x,y)を領域Dで積分することを意味する。
670664:01/09/19 19:18
度々申し訳ありません。

664の問題なんですけど、xとyの比を求めたいのです。
どうすればよいでしょうか。
>>612を見て思ったのですが、そもそも
e^x+x=0
の解を求める事は出来るのでしょうか?
解をuとした時
0<u<1
uは無理数
である事はすぐにわかるのですが・・・。
>>670=>>664
求まらないのでは?
x^x*y^y=e
で関数f(x)=x^xは1≦xの範囲で単調増加関数で
1≦f(x)<∞
になるんだから、xとyの比は求まらないんじゃないかなあ?
673664=670:01/09/19 20:12
>>672
レスどうもありがとうございます。
駄目押しに、『x+y=1(x、yは正)』
という条件下でもダメでしょうか・・・。

我侭ばかり言って申し訳ありません。
>>671
>の解を求める事は出来るのでしょうか?

675132人目の素数さん:01/09/19 20:21
>>669
発散収束を考慮するだけなら、原点が含まれている限り領域の形が関係ないことに注意。
x^a+y^b=t とか変数変換すると、式が簡単になるはずなので、計算してみて。
y=kx     一変数の場合に帰着できるはず。(が、面倒くさそう)
>>673
x+y=1とするとxは2つの解
x=1/2+a
x=1/2-a
0<a<1/2
を持つ。
具体的にどうやって比を求めるか迄はちょっと解らないです・・・。
>>676

そうですか・・・。
色々どうもありがとうございました。
678132人目の素数さん:01/09/19 20:52
あのー・・
544が565になれば何パーセントの上昇になるのですか?
どういう式を書けば弾き出せるのですか?
中卒なので分かりません。教えてください・・
679132人目の素数さん:01/09/19 22:15
544/565=1.0386…
を、100倍するとパーセント。
544を100%として考えると565は104%くらい。
なので大体4%の上昇。
…数値から察するに、年収計算?
680678:01/09/19 22:45
>>679さん
ども。ありがとうございました。
年収ではないんですけどね・・
681132人目の素数さん:01/09/19 23:50
>>675
領域は区間とってやるのは分かってたんですけど
変数変換って(x,y)→(t,k)ってことですよね?
それやると複雑になって出てきませんよ?
682132人目の素数さん:01/09/20 00:00
長さ5cmのマッチ棒を接着剤でくっつけて作った、
タテ5cm、 横10cm、高さ5cmの直方体です。
この直方体を作るには20本の マッチ棒を使います。

同じ様にして、タテ45cm、横35cm、高さ25cmの直方体
を作るとマッチ棒は全部で何本使う事になりますか。

この問題おねがいします。
683132人目の素数さん:01/09/20 00:31
>>673
x+y=1 とすると、x,y>0 から x,y<1 で、
xlnx+ylny<0 となって =1 とはならないよ。
嚢_i=1に対する、H=-嚢_ilnX_i の振舞いについては
「エントロピー」で調べてみれば?
684132人目の素数さん:01/09/20 00:35
>>682
>この直方体を作るには20本の マッチ棒を使います。
なんで?
685132人目の素数さん:01/09/20 00:37
箱を積み上げたような形だったら、ちょうど20本になるよ。
686132人目の素数さん:01/09/20 00:40
>>685
すべての隣り合う格子点を結ぶようにするってこと?
687132人目の素数さん:01/09/20 00:41
そうそう。
一つの「床」に必要な縦のマッチは、9×6=54本
横のマッチは7×10=70本。で系124本。
床が6個いるから124×6=744本。
垂直方向には、5本でつくった柱が10×8本要るから
80×5=400.合計して1144本。
689132人目の素数さん:01/09/20 01:14
>>688
ミスった。立てマッチ計算ミス!
690はなう:01/09/20 01:16
>>682
おもしろい問題。

たて5a、横5b、高さ5cとすると必要なマッチ棒は
a(b+1)(c+1)+(a+1)b(c+1)+(a+1)(b+1)c
だね。
ってことで、
9*8*6 + 10*7*6 + 10*8*5= 1272 本 かな。
>>674
何故。
>>682
辺の長さがa本,b本,c本の直方体で表面だけなら
4(ab+ac+bc)本なので
4(9×7+9×5+7×5)=572本。
>>691
解く時に許されている操作は?
というか解を表現する時に使ってもいい演算は何か?を考えてみてください
>>693
どういう操作を使う場合に否となるの。
そのときe^x−2=0は解けるの。
695132人目の素数さん:01/09/20 02:39
fn(x)=x^x^x^…^x とする
f1(x)=x , f2(x)=x^x である
fn(x)の導関数f'n(x)を求めよ。

答えはかなり複雑になるので表記法は
各自工夫する事。

昔できたんだけど、なんか出来なくなっちゃった
ちなみに自作問題です。高校までの数学で必ず解けます。
>>694
日本語を書いてください。
>>696
説明できないの。
698132人目の素数さん:01/09/20 07:02
x^2+2ax+1<0をみたす整数xが1個のみになるaの範囲を求めよ。
↑数Tの範囲で解けると思うんだけど解けない・・・
699132人目の素数さん:01/09/20 07:23
>>698
解けるよ。f(x)=x^2+2ax+1とおく。
D≧0よりa^2≧1からa≧1or a≦-1。
(i)a≧1のとき
f(0)=1>0、f(-1)=2-2a≦0よりf(-2)>0が必要十分。
∴(1≦)a<5/4
(ii)a≦−1のとき
f(0)=1>0、f(1)=2+2a≦0よりf(2)>0が必要十分。
∴-5/4<a(≦−1)
700132人目の素数さん:01/09/20 07:33
>>698
別解)
平方完成してf(x)=(x+a)^2-a^2+1<0
-a^2+1<0 ,f(-a+(1/2))>0となればよいので
-5/4<a<-1 ,1<a<5/4
グラフ書いて考えてみやがれこのやろう
701132人目の素数さん:01/09/20 07:44
「φ」ってどういう意味なんですか?
>>701
空襲壕
703701:01/09/20 08:47
>>702
空襲壕ってなんですか?読み方ですか?
今レポートやってるのですがこの記号の意味がわからないんです。
一応わかってるような書きかたしていますがうちの教授たまに機嫌が悪いと
いろいろ質問してくるんです。
だからできれば記号の意味くらいは理解しておきたいと思って質問させてもらったのですが・・・・
やっぱりこの板で聞くとネタっぽいですか?(もちろん違いますよ)
704132人目の素数さん:01/09/20 08:53
オイラー関数
>>703
それだけじゃ解らないよ。
どういう文脈で出てきたか書いてみたら?
>705
文をよく読んだら理解できました。
ただの未定ベクトルでした。
すみませんでした。
>>26 が分かる人いませんか。
(「sin(x)=0を解け。」という問題が
「xは一意ではない。」や
「−π/2≦x≦π/2とすれば一意になる。」や
「sin(2π)=sin(−3π)=0なのでx=2πとx=−3π。」という
答を求めているわけではないのと同じで
一意であるかを求めているわけでも
答が一つになる問題を作りたいわけでも
答の一部を知りたいわけでもありません。)
708132人目の素数さん:01/09/20 10:42
どうも、ラウンジからきた者ですが・・。
http://saki.2ch.net/test/read.cgi?bbs=entrance&key=1000901876&st=113&to=113&nofirst=true
↑これって合ってるんでしょうか?教えてください
>>708
実際に3乗してみれば?
>707
レス番違ってないか?
711132人目の素数さん:01/09/20 11:19
>>707
どうゆう答えをもとめてるの?今まででてる答え以上の答えがあると思ってる根拠は何?
数学の世界にはきれいな答えがでない問題なんていっぱいあるよ?
712132人目の素数さん:01/09/20 11:37
大学の数学の勉強しているはずなのに、
「あれ?これはどうするんだっけ?」と
その都度高校の参考書を引っ張り出している俺。
非常に効率悪し。

高校の参考書を初めから勉強し直す時間もないし、
どうすれば良い?
笑わずにまじめに答えてくれ!
きっと同じ悩みを持ってる輩も多いはず…

ちなみに工学部(土木)です。
713はなう:01/09/20 12:53
>>695
数列ではないが数列積を使わせてもらうと、
f_n'=f(n-1)'f(n-1)x^{f(n-1)-1}
  =f(n-2')f(n-2)x{f(n-2)-1}f(n-1)x{f(n-1)-1}
  =・・・・・
  ={Π[k=1,(n-1)]f(k)}*x^{Σ[k=1,(n-1)]f(k) -n+1}
だね。
714はなう:01/09/20 13:40
>>26
>>707
>>87さんの議論の通りだとおもふ。この条件では、

「g(a,b)=ab
ただしbが無理数且つa≠0の時はgの値は不明」
ということになりますの。
g(a,1)=aがあるからaは無理数でも良いんだけどbがな〜。
せめて、g(1,b)=bでもあれば。
715はなう:01/09/20 13:44
>>713タイプみすった
f_n'=f(n-1)'f(n-1)x^{f(n-1)-1}
  =f(n-2')f(n-2)x^{f(n-2)-1}f(n-1)x^{f(n-1)-1}
  =・・・・・ です。
>>714
補足すると、これがこの条件での「答え」なので、「答えの一部」でもなんでもないですよ。
716はなう:01/09/20 14:32
>>715
まだちがうじゃん
f_n'=f(n-1)'f(n-1)x^{f(n-1)-1}
  =f(n-2)'f(n-2)x^{f(n-2)-1}f(n-1)'x^{f(n-1)-1}
  =・・・・・
>>714
R−>Rの関数fが任意のa,bに対して
f(a+b)=f(a)+f(b).f(1)=1を満たすとき
a∈R−Qのときのf(a)は一つに決まりませんが
RをQ上のベクトル空間としての基底で1を含むものをAとすると
fはgをA−>Rの関数でg(1)=1となるものに対して
f(狽秩Qi*a_i)=狽秩Qi*g(a_i)
(r_i∈Q,a_I∈A)
と表せます。

>>711
>どうゆう答えをもとめてるの?今まででてる答え以上の答えがあると思ってる根拠は何?
今まで出ている答以上の答がないという理由がないからです。

>数学の世界にはきれいな答えがでない問題なんていっぱいあるよ?
この問題がきれいな答が出ない問題であるという理由が分かりません。
718132人目の素数さん:01/09/20 16:08
教えてください。

V:有限次元実線形空間
V^*:Vのdual space
T={V^*×VからRへの線形写像全体の集合}
M={VからVへの線形写像全体の集合}
とした時、
TとMとの間に自然な同型が存在する事をどうやって示せばいいのですか?
>>714
例の追加です。
R−>Rの関数fが
(d/dx)f(x)=f(x)を満たすというだけでは
b∈Rに対してf(b)は決まりませんが
fはf(x)=a・exp(x)(aは実定数)と求まります。
720:01/09/20 17:08
Tの元f は v^*とv をとれば、f(v^*,v)=<v^*,Av>となる行列
Aが唯1つ決まるっぽい.
あんま考えてないけど。。。
あ、ちなみに<,>は内積
f(v^*,v)=<v^*,Av>が成り立てば
Tと、行列Aの全体の間に同型が存在すると言う事ですね。
そうすればMとの同型はすぐ出てきますね。
有難う御座います。
722132人目の素数さん:01/09/20 18:26
>>717
君は外出の解答は全部理解できてるの?
理解できていないならここここがわからないといわないと
答えてくれた人たちに失礼だよ。

>>26の条件をgがみたす。
⇔ある環準同型φ:R→Hom_Q(R,R)が存在して
 g(a,b)=φ(b)(a)となる。

は理解できてる?
>>722
>君は外出の解答は全部理解できてるの?

できてないから↓こう答えたんでしょう・・・・
もう相手にしないほうが・・・・・

711>>数学の世界にはきれいな答えがでない問題なんていっぱいあるよ?
711>この問題がきれいな答が出ない問題であるという理由が分かりません。
↑の訂正  ×711  ○717
725まおまお:01/09/20 19:56
>>718
一般に、(U x V)^* = (U^* x V^*)、だよ。だから、
(V^* x V)^*
= (V^** x V^*)
= (V x V^*)
= (V^* x V)
= Hom (V,V)
726132人目の素数さん:01/09/20 19:58
ページの下の方にある加護のバナークリックしてくださーいカッカッ!
http://www5d.biglobe.ne.jp/~musume/
727132人目の素数さん:01/09/20 20:21
>>723
うん。そうみたい。というか“ここここがわからないのでもう少し
説明してください。”とくればもうちょっと教えてあげるつもり
だったんだけど...
そんなにきれいにはかけない説明。もうこれ以上はしらん。
>>722がわからないとこれ以上説明できないけど仮に>>722
理解できたとして一般に体F上の代数AとFベクトル空間Vにたいして
AからHom_F(V,V)へのF代数としての準同型のことをAの表現または
A加群という。よって問題は

 問題 RをQ代数とみなしたときそれにR表現(またはR加群)としての
    構造はどれぐらいはいるか?

に近い。仮にこう解釈すればRはただの体だからそのR加群は次元だけで
解釈されてR^(γ)とかける。(γはCardinal number。)
ではどんなR^(γ)がRと(Qベクトル空間として)同型か?だけどそれは
Cardinalだけできまって答えはγ≦Card Rとなる。
もし連続体仮説を仮定すればすくなくともγ=1,2,3,...,c(=加算無限),2^c
でパラメタライズできる。
でも近いだけで実際には1次元でないかぎり自分自身への非自明な
同型があるから同型なものを同一視しなければさらにクラスはふえるし
連続体仮説を仮定しなければ“連続体濃度=アレフ53でも矛盾しない。”
なんて論文もあるほどだからもうどうしようもない。
どうせえっちゅうねん。
728132人目の素数さん:01/09/20 21:22
http://news.2ch.net/test/read.cgi?bbs=news&key=1000831850
あの〜IDで「USA」って出る確率は何分の一ですか?↑「USA」出すスレ
>>725
おぉ、有難う御座います。
そう考えると解りやすいですね。
730解答解説を御願いします:01/09/20 23:12
tを定数としてxy平面上の放物線 l:y=e^-t x^2+tを考える。tがt>0の範囲を変化するとき、lが通る範囲を求め、その概形を図示せよ。
>>722
「R−>Hom_Q(R,R)の環準同型がいっぱいあると
gを全て表す方法がない。」ということ以外は分かります。
>>731
訂正
>いっぱいあると
いっぱいあるので
733132人目の素数さん:01/09/21 00:19
>>730
f(t)=e^(-t)x^2+t とすると、t→+∞ のとき f(t)→+∞。
f'(t)=-e^(-t)x^2+1 だから、
x^2≦1 の時は f は単調増加で、f(t)>x^2
x^2>1 の時は f(t)≧f(2log|x|)=1+2log|x|
734132人目の素数さん:01/09/21 00:51
tiny basic のプログラム(150行)ください。
735132人目の素数さん:01/09/21 00:57
>>730
733さんに先を越されてしまいましたがせっかく解答を作ったので
もったいないお化けが出るといけないのでうぷします

l:y=(e^-t)x^2+t=f(x)
まずグラフがどんな風に動くか見てみましょう
t=0のときはy=x^2となります
tをでかくしてゆく事により(e^-t)→0ですから
l:y=(e^-t)x^2+tはつぼみを開きながら上に移動してゆくわけです
このときの包絡線を求めれば良いわけです
(包絡線ってなに?って場合は細野数学の「受験数学のテク○ックが面白いほどわかる本」を立ち読みして下さい、文字だけでは説明できません)
x=αと固定したときの
y=(e^-t)α^2+t=g(t)
の最小値を調べます
g'(t)=-(α^2)(e^-t)+1
増減表を書いて
t┃   ┃log(α^2)┃
g'┃ - ┃  0  ┃+
g┃右下┃ 最小 ┃右下
よってt=log(α^2)のときにg(t)は最小値1+log(α^2)をとる
ここでxを動かしてf(x)=1+log(x^2)=1+2log|x|
ただしt>0であることもかんがえて
lが通る範囲は
y=h(x)=┃ x^2   (-1≦x≦1)
     ┃1+2log|x|(x≦-1,1≦x)
の上側。
ただしy=x^2の線上は含まないがy=1+2log|x|の線上は含む
>735
>もったいないお化けが出るといけないのでうぷします

(´-`).。oO(なんで、リソースもったいないお化けは無視なのかな?)
737132人目の素数さん:01/09/21 01:32
B=Cである△ABCを用いて、0°<θ<90°のとき、次の等式が成り立つことを示せ。

sin2θ=2sinθcosθ
cos2θ=2cos(2乗)θ-1

正弦定理を利用するのは解るのですが、解説にある「a=2bcosθからsin2θ=2sinθcosθ」のくだりが理解出来ません(a=2bcosθがどうやって出るのかが)
よろしくお願いします。
738はなう:01/09/21 01:41
>>737二等辺三角形の等しい二角をθにしたのなら底辺の半分はbcosθなんだからaはその二倍、ってことでしょう
739695:01/09/21 05:03
>>713
>>715
>>716
それは間違いだと思います。
反例として
f'2(x)=x*x^(x-1) ではなく
f'2(x)=(logx+1)f2(x) だと思います
このスレのみなさまもう一度
>>695 の御再考をお願いします。
740132人目の素数さん:01/09/21 06:24
すいません
昨日先生に「√2とはなんだ?」「√2=2^(1/2)は真か」
と問われたのですが
考えてみると混乱してきてしまいます。

みなさんはどうおもわれますか?
741132人目の素数さん:01/09/21 07:55
真だよ。
指数関数の計算は
a^m+a^n=a^(m+n) ……@
    と
(a^m)^n=a^(mn) ……A
の2つの式のm、nを自然数だけでなく
色々な数に拡張したもの。
例えばA式で
m=1/2 ,n=2 とすると
(a^(1/2))^2=a^1
となって
(sqrt(2))^2=a
の式との比較から
a^(1/2)=sqrt(2)
でなければならないことが判ります。
その他、a^0=1 やa^(-1)=1/a なんかも@式A式で
m やn を色々変えて見ればわかるよ。
742741:01/09/21 07:58
ごめん。
@式は
(a^m)・(a^n)=a^(m+n) ……@
の間違いです。
スマソ。。。
743741:01/09/21 08:03
@式やA式が正しいのはすぐ判るでしょ?
中学校のとき、
a を n 回掛けたものを a^m と書くと習ったでしょ?
なら
(aをm回掛けたもの)*(aをn回掛けたもの)=(a^m)・(a^n)
で、左辺は
(aをm回掛けたもの)*(aをn回掛けたもの)=(aをm+n回掛けたもの)=a^(m+n)
となるのは当たり前だもんね。
A式についても同様に考えればすぐ分かる。
744735ですよ:01/09/21 10:11
>>736
(´-`).。oO(お望み通りのつっこみを返してあげましょう。
       リソースもったいないお化けはあんたのとこにも出ます。よかったね♪
       それに私の解答と733さんの解答は同じでないのでうぷする価値は十分あると思います。ほら、本にも別解って載ってるでしょ。
このレスを無駄にするともったいないお化けが出ちゃうので>>735の訂正です。
増減表のgの右の欄は右下じゃなくて右上にして下さい。
745132人目の素数さん:01/09/21 14:46
>>725を見て思ったのですが
{(V^*)^(l+1)×V^k}^*
=(V^**)^(l+1)×(V^*)^k
=(V^*)^k×V^(l+1)
が一般に成り立つのですか?
>>744 別解と言うより、733が略解で735が詳しい説明でしょう。
普通はタイプが面倒なわけで・・・
ある程度詳しいにこした事は無いでしょうね
(´-`).。oO(詳解書くのは説明を要求されてからでいいのに・・・・ウザイだけだし)
748132人目の素数さん:01/09/21 15:40
標準偏差とある値を用いて
パーセンタイルを求めるときの式を教えてください。

たとえば
標準偏差5.6、
平均値170.8のとき、
176.4は何パーセンタイルかていうことなのですが。
749735:01/09/21 15:52
>>747
はいはい、すぅ〜〜いませんでしたああああ。
あなたのおかげでウザイという事がよーく分かりました。

(´-`).。oO(「はい」は一回でいいのに‥‥)
とか言い出しそうだな‥‥鬱だもう消えてくれ‥‥
(´-`).。oO(「はい」は一回でいいのに‥‥)
751132人目の素数さん:01/09/21 16:26
f(z)=2^z(zは複素数)に対し、
limsup[r→∞]((log|f|_r)/r)=log2(log|f|_r = max[|z|≦r]|2^z|)

えっ?どうやって計算したんだ?そんな一瞬の内に・・・。
初歩的過ぎる質問でしょうが、どうか課程をお教えください。
752132人目の素数さん:01/09/21 16:46
2^z=e^(zlog2)
log|f|_r=max[|z|≦r]|2^z|=e^(rlog2)=2^r
limsup[r→∞]((log|f|_r)/r)
=limsup[r→∞](2^r/r)
=∞

log2にならない、鬱だ・・・。
753132人目の素数さん:01/09/21 17:48
nPrを階乗で表すと、n!/(n−r)!だが、
これを証明せよ。

が、わかりません。解説も添えて、丁寧に教えてください。
お願いします。
754132人目の素数さん:01/09/21 17:53
非常にレベルの低い質問で申し訳ないのですが
1/√13+√8
はどのように有理化すればいいんでしょうか?
>>751
lim_{r−>∞}(log(2^r)/r)=log2。
756735:01/09/21 18:44
>>750
そうくるとは思わなんだ
‥‥もう返す言葉も見つかんないよ。

ねぇみんな、「鬱」って「うつ」って読むんだぜ、知ってた?オレ昨日まで「いや」って読んでたよ。

鬱だ氏のう‥‥

>>754分子分母に(√13-√8)をかける
757eeeee:01/09/21 20:18
(´-`)<>>749で「わかった」んなら「返す言葉」を探す必要は無い。マターリいきませう。)
758132人目の素数さん:01/09/21 20:20
Yahoo!で下のように言ってるのがいるんだが(@「数学は完成するのか」というスレ)、
ここでカッコよくレスするには、どお?

> ・・・では、対象を絞って、素数問題の完成とはいったい何でしょうか。
>
> (1)一つ前の素数から、次の素数を確実に予想すること。
> (2)n番目の素数をnの関数として記述できること。
> (3)実数体系の中で素数を誤差を容認して確定できること。
> (4)素数は乱数と同じ分野に属して予測が不可能であると証明すること。
> (5)自然数の体系自身に記述の限界があって素数はその範疇外とあきらめること。
>
> 等々でしょうか?
> その他、数学者を悩ましている素数の解決目標はどのような形でしょう?

そう、みんな数論って好きだから、ビシっと決めたいゾ!
759まおまお:01/09/21 20:24
>>745
成り立つと思われ
760祖山:01/09/21 21:00
球面上にある5点のうち、少なくとも4点を含む(開)半球面は存在するか否か?
>>759
有難う御座います!
762はなう:01/09/21 22:30
>>739
正直、スマンカッタ。あんな初歩的な間違いする漏れっていったい・・・

って、ことで、
>>695
fnの式を両辺対数取り、
log(fn(x))=f(n-1)(x)logx
微分。
(1/fn(x))f'n(x)=f'(n-1)(x)logx+(1/x)f(n-1)(x)
f'n(x)={f'(n-1)(x)logx+(1/x)f(n-1)(x)}fn(x)

これをやると、こうなる。わかりやすい解答が書けなくないか、これ。

f'n(x)=(logx)^(n-1)Π[k=2,n](fk(x))+Σ[p=0,(n-2)]{(1/x)(logx)^p*Π[q=(n-1-p),n]{fq(x)}
ですかな。鬱々。
763はなう:01/09/21 22:33
>>762
もちろん、f'1(x)=1だけは別で。
764132人目の素数さん:01/09/21 22:33
>>760
赤道上に90°間隔で4点をとり、北極に第5の点をとれば、
一個の半開球では被えない。
765735:01/09/21 23:05
>>757
(´-`).。oO(指図されなくともまた〜りさせていただきますよ―だ。

       >>746おっ見逃してた、いいこと書いてあんじゃん

       ‥‥最近侍魂更新しないなぁ、ちぇっ)
766名無し:01/09/21 23:26
急ぎ。誰か解いてください。
『AB=11,AC=13の僊BCがある。AB上に点PをAP=9となるように,
AC上に点RをAR=7となるようにとる。また,BCの中点をQとする。
PQ=RQ,PQ⊥QRのとき,儕QRの面積を求めよ。』
小学生にも分かるような(*)解法でお願いします。
(*) 相似比,三角形の面積=底辺*高さ÷2 を使う程度。
>>766
文頭の一文がなければ解答書いたのに。
つまらない問題だし、放置決定(w
768名無し:01/09/21 23:46
>>767
ああ,そんなこといわないで。おねが―――い。
769パスカルのファン:01/09/21 23:46
ここにひとつのサイコロがあります。
いかさまサイコロかどうかわかりません。
さて何回サイコロをふればいかさまか
そうでないかわかるのですか?
誰か教えてください。
770はなう:01/09/22 00:03
>>767
放置かい・・766困ってるじゃろーに。
771132人目の素数さん:01/09/22 00:07
>>752
>>755
なとーく。supって嫌いです。ありがとございました。
772「急ぎ。」って:01/09/22 00:11
会社で上司が部下に対して使う言葉のような…
773766:01/09/22 00:16
うわ―ん。すまんかった,言葉づかい謝る。スマソスマソ
頼みますよ。お願い。
>>766 を解いて・・・・・・。
774はなう:01/09/22 00:19
「急ぎ」は教えて貰う言葉じゃない気がするが、
766が「つまらん問題」ってことはない。小学生が解けるようなやり方を書くのは至難のわざだよ。
775はなう:01/09/22 00:23
>>766 いろいろあったが答えたろう。
小学生系はまかしとき。わしゃその道はプロだかんね。
△APRは全体の9/11*7/13=63/143
△BPQは全体の2/11*1/2=1/11
△CQRは全体の6/13*1/2=3/13
で△PQRは全体の34/143。
で、PQRは直角二等辺3角形なので、Qとは反対側に合同な三角形をつけると正方形ができる。
その後ABCからその正方形を除いた穴あきみたいな形を作る。その面積を考える。
こっからは図がないと厳しいが、自分で書いてくれい。
さっきの穴あき図形の
BPQをPを中心に回転してQを上の図形にぴたっとつける。
CQRをRを中心に回転してQを上の図形にぴたっとつける。
すると、穴がぴったり埋まり(Qが中点で角BQP+角CQR=直角だから。)
角Pと角Rが直角な四角形が出来る。その面積は
9*2/2 + 7*6/2 = 30
これが、全体の1-(34/143 * 2)=75/143なので、
PQRは30 * 34/75 = 13.6
776766:01/09/22 00:32
>>775
ありがとー。あなたに
“おいらが今までに出会った人の中で,一番親切な人で賞”
を贈呈いたします。
やっぱ,プロは違うね。
777はなう:01/09/22 00:39
>>776
ちなみにこの問題は去年の算数オリンピックのファイナルの問題にクリソツ。ちょっと違ったが。
あそこ解答非公表だから漏れも最初思いつくのに2週間くらいかかった。
3平方の定理を使って連立2次方程式とか立てると出来るんだけどねー。小学生用だからむずい。
778パスカルのファン:01/09/22 00:41
あのう>>769のほうは教えて
いただけないのでしょうか?
779はなう:01/09/22 00:43
>>778いかさまサイコロって、なに?
780パスカルのファン:01/09/22 00:46
>779
細工がしてあるいんちきなサイコロのことです。
781はなう:01/09/22 00:52
>>780そりは一つの目しかでないの?とにかく条件がないよ。
782パスカルのファン:01/09/22 00:57
>781
特定の目(たとえば3だけ)が出やすい
サイコロです。統計的にある程度わかると
思ったのですが。
783ラスカルのファン:01/09/22 01:12
統計的にある程度‥‥を求めるんですか、
それなら、何回サイコロをふればわかるかという問題文はおかしい気がする。
784はなう:01/09/22 01:15
>>782
どれくらい出やすいのかがわからない以上答えられない。
たとえばもし1/6より1/1000だけ3が出やすくなっているさいころだっていかさまでしょ。そのときゃ。試行回数は最低数千はいくと思われ。
もしどれか一つの目しかでないサイコロであるとすると、
2回目の試行終了時に違う目だったら100%いかさまじゃない。2回ともおなじ目だった時は5/6の確率でいかさまである。
3回目終了時まで同じだったら35/36でいかさま
・・と、限りなくいかさまである確率が1に近づく。いついかさまと見なすかはこの話の主人公に委ねられる。
なんかいたちごっこじゃない?もっと一般的な議論出来る方募集。
785はなう:01/09/22 01:17
>>784
>もっと一般的な議論出来る方募集。
かきかた悪かった。ようは、もっと論じられる方に聞いて下さいって事です。わちしはおてあげ。ネマス
786パスカルのファン:01/09/22 01:31
>783
もうしわけありません。
当方文系なもので。
たとえばわたしがサイコロ賭博の胴元として、
いかさまサイコロだけを排除したいわけです。
正しいサイコロはそれぞれの目が1/6
の確率ででるわけですよね。
ここに正しいサイコロが1つあります。
でも私にはいかさまサイコロか正しいサイコロか
わかりません。いったい何回くらいサイコロを
ふればある程度正しさが測れるのかということです。
実務上何回もサイコロをふれないので、ある程度の
ところで切り上げたいのです。何回位振ったところで
やめればよいのでしょうか?
787132人目の素数さん:01/09/22 01:34
カッシーニの軌道計算をおしえて
788はなう:01/09/22 01:47
>>786
満足な結果を得たいなら60〜100回は振らないと無理かな。
例えば120回振って1から6までのうち一番出たのが
22回くらいで一番でなかったのが18回くらいだったらそれは相当優秀なサイコロでしょう。
おすすめは120回です。
簡単コースは60回。その時13回以上出たものと7回以下しか出ないものがなかったら一応及第とみなすってとこでしょうか。
残念ながらそれ以下は無意味と言わざるを得ません。
789132人目の素数さん:01/09/22 01:59
>786
何回ふったら分かるかというよりは
統計学的には、ある観測結果が十分起こり得る確率かどうかで
決めます。
例えば平均値を計算した時にその平均値になる確率を求めるとします。
出目の平均値が何回ふっても6だったりしたらいかさまですよね?
こういった起こりそうもない結果を除外して
結果が除外されているどうかで、正確なサイコロかどうかを決めます。

全体の5%のケースを除外するとき5%検定と呼ばれますが、
これは1%でもいいです。起こりそうもない水準は場合によって変えます

例えば50回投げた時1〜6まで出た回数が
10回7回8回11回6回8回
だった場合は希少なケースには含まれなくて
いかさまサイコロではありません。(統計学入門:東京大学出版会)

実際の感覚ではわかりづらいですけどね
790パスカルのファン:01/09/22 02:07
はなう様ありがとうございました。
ということは50人以下の人に対する統計調査など
無意味なものが多いのでしょうか。
実はアンケート調査等に数学を利用したいのですが、
大学の教員は数学ができないので困っています。
おすすめの本などありませんでしょうか?
791ラスカルのファン:01/09/22 02:07
>>786
そんなの自分の勘だろ(爆)
おいらも、もうお手上げ。はなうさんのを参考にしてくれ。
十回くらいふりゃい―んじゃね―の。だってさー、いかさまのサイコロって1/2くらいの確立で目的の数字が出ないと役にたたねーんじゃね―の?
つーか今どきサイコロ賭博って存在するんですかね‥‥?
792:01/09/22 02:23
皆様くだらない質問につきあっていただき
ありがとうございました。
793132人目の素数さん:01/09/22 02:34
>790
50人以下の人に対する調査が有意かどうかは場合によりけりです。
どういう調査をするかにもよりますし、ただその時にわかることは
推定された平均値が妥当かどうかというようなことです。

統計学入門:東京大学出版会
入門数理統計学:ホーエル:培風館

あたりを読んでください。
>>790
アンケート調査だったんか!
>おすすめの本などありませんでしょうか?
マジレスすると、数学板の人の読む統計なんとかの本はあるけど(本屋で見るだろ?)、
キミには役に立たないかも。
マーケティング用の統計数学みたいな本がいいんじゃないかな。
それで「偏差」とか理解すればいい。もっというと計算よりも、例えば50人に聞くとして
1人がまったくのウソを答えたとき、結果にどのくらい影響するか「イメージ」できることが
大事なんじゃないか?

そうするとキミのほうが専門だろうけど、(アンケートの)設問のしかたのほうが正確な統計を
とるために必要なんだろうな。
ちなみに surveyml ってMLはなかなかのもんらしいぜ。
795初日ので:01/09/22 04:46
どーも場違いなとこで書いてしまったせいか、昨日かいたのが見当たらないので、こっちで質問しなおします。
1=2
これがどーゆー意味なのかわかる人教えてくれませんか?
中学生位の計算で、こうなっちゃうんですけど。
大学クラスだとなんか理由とかあるのかなーと思った次第。
>>795
↓ ここに3=7っていう似たのがあるよ。これのレスを見よ。
http://cheese.2ch.net/test/read.cgi?bbs=math&key=1000128785&st=59&to=59&nofirst=true

それと書いたのが見当たらないとき、ブラウザがInternetExplorerなら、「Temporary Internet Files」を見て、
名前でソートをかけ「read.cgi?・・・・」ってのを見ると、キミの書き込み記録が出てるぜ。
797695:01/09/22 06:17
>>762
はなうさんめんどくさい計算を有難うございます。
(その上こんな事お願いしちゃって良いのかな?)

>f'n(x)={f'(n-1)(x)logx+(1/x)f(n-1)(x)}fn(x)
実は上の漸化式までは出せたのですが
その先の変形が出来なかったのです。。
よろしければ教えて頂けないでしょうか?
掲示板では説明しにくいようでしたら自分で勉強します。
ではお願いします。
>>797
(Π_{0≦i≦n}f_i(x))(log(x))^nで割る。
799132人目の素数さん:01/09/22 09:17
669にも書いたけど、まだわかりません。誰か教えてください。

二重積分∫∫[D]{1/(|x|^a +|y|^b)}dxdyが収束するような
a,bの範囲を求めよ。ただし、領域D:x^2+y^2≦1

∫∫[D]f(x,y)dxdy は f(x,y)を領域Dで積分することを意味する。
800はなう:01/09/22 11:23
おはようございます(藁
>>790
アンケートだったんですか・・・
もう皆様レスしてますが設問が的確なら無意味な統計調査になることは少ないと私は思います。もうそっから先は私は専門外。
完璧に勉強するなら、793さんの
>統計学入門:東京大学出版会
をよんで、なお その続刊の「人文・社会科学の統計学」ですかね〜。そこまで理解していたらそーとーになれるとおもいます(私は挫折)。
こういうサイコロとか云々の簡単な確率のカンを少しでも見てみたいだけならgoogleで「ゲーム理論」とかで検索して引っかかったとこみると良いかもしれません〜。

>797
っていうか、私は「やりまくった」結果(藁
f1'=1
f2'=(logx+1)f2
f3'={logxf2'+(1/x)f2}f3={(logx)^2    + logx +(1/x)                    }f2f3
f4'={logxf3'+(1/x)f3}f4={(logx)^3*f2   + (logx)^2*f2   +(1/x)logx*f2     +(1/x) }f3f4
f5'={logxf4'+(1/x)f4}f5={(logx)^4*f2*f3 + (logx)^3*f2*f3 + (1/x)(logx)^2*f2*f3+(1/x)logxf3 +(1/x) }f4f5

ここで後ろにあるf3f4とかf4f5とかも展開してから論じる。
2項目は実は(1/x)*f1が消去されていることにも注目すると、
1項目を除く他の項は(1/x)(logx)^p*fn*f(n-1)*f(n-2)*・・・*f(n-1-p) と一般化できる。2項目がp=(n-2)のときで3項目がp=n-3の時で・・・n項目がp=0の時であるので、その総和としてシグマかいときました。
で、1項目は(logx)^(n-1)f2f3f4f5・・・fnなので、

f'n(x)=(logx)^(n-1)Π[k=2,n](fk(x))+Σ[p=0,(n-2)]{(1/x)(logx)^p*Π[q=(n-1-p),n]{fq(x)}

>>799 なんか無理矢理積分してけば綺麗になるっぽいけど〜。仕事なので〜。誰かまかした。世間は休日なのでだれかやってくれるかな。
801132人目の素数さん:01/09/22 11:37
ちゃんと教えてくれよ〜
前の奴もこうしたら言いみたいなこと言って全然できなかたよー
802132人目の素数さん:01/09/22 12:31
参考書の解説をよんでも、なぜ、ここがこうなるのかが理解できずに困っている問題があります。
どうか教えて下さい。

問題 △ABCにおいて、AB=6cm、BC=8cm、∠ABC=90°であるとき、△ABCの外接円の面積
   と内接円の面積の差はいくらか。

解説 三平方の定理から AC=10cm。
△ABCの内接円の中心をIとすると、△ABC=△IAB+△IBC+△ICA
  
   内接円の半径をrcmとすると、

    @24=6r/2+8r/2+10r/2=(6+8+10)r/2=12r

よって、r=2cmであるから、内接円の面積は4π[cu]。一方、∠ABC=90°であるから、
   外接円の直径はACである。よって、外接円の半径は5cmであるから、外接円の面積は、
   
    25π[cu]。従って、外接円と内接円の面積差は

    25π-4π=21π[cu]

    ここで、@の隣の「24」という数字がなぜでてくるのかが理解できません。
    図を表せないことが残念ですが、詳しく解説して下さるかたのレスを
    お待ちしております。よろしくお願いします。

  
   
803735ですよ〜:01/09/22 13:05
24は三角形の面積ですよ〜
804132人目の素数さん:01/09/22 14:34
教えてください。
Vを実線形空間、V^*をそのdual spaceとした時に
どうして(V^*)^*=Vとなるのでしょうか?
また、VもV^*は同じ次元の実線形空間だから、
VとV^*とは同型なのでしょうか?
805132人目の素数さん:01/09/22 14:49
735ですよ〜さんへ
 早速のお答えをありがとうございました。
言われてみると解説も何もないですね。私の頭が凝り固まっていたようです。
 でも、本当にわかって嬉しいです。すっきりしました。また、わからない
問題にぶつかりましたら、教えて下さい。
806132人目の素数さん:01/09/22 15:07
次の公式(Wool houseの公式)の証明をお願いします。

a<bは整数、nは正の整数
fは[a,b]で定義された無限回微分可能な関数


f(a)+f(a+(1/n))+f(a+(2/n))+…+f(b-(1/n))+f(b)
=n{f(a)+f(a+1)+…f(b)}-{(n-1)/2}{f(a)+f(b)}
-{(n^2-1)/(12n)}{f'(b)-f'(a)}+{(n^4-1)/(720 n^3)}{f'''(b)-f'''(a)}+…

f'は一階微分f'''は3階微分をあらわします。
aから1/nずつ足してbまでの和が欲しい時分点の数を減らしても近似できるという
公式です。
807yanyan:01/09/22 15:18
>>804
(V^*)^*とV が同型になるには、Vが有限次元という条件が必要です。
Vから(V^*)^*への線形写像、F: V --> (V^*)^* を
F(v)(u)=u(v) とすると、Vが有限次元なら同型。一般には
単射です。

Vが有限次元のとき、V^*も同じ次元を持つので、VとV^*は同型
ですが、内積が入っていない状況下では自然な同型写像
V-->V^* はありません。実数上の有限次元ベクトル空間などで
内積(x、y)が定義されていれば、G: V-->V^*
を G(v)(w)=(v,w) と定めれば同型です。

以上は位相を考えていない場合です。
808735ですよ〜:01/09/22 15:29
>>805
おお、この2chにも「お礼」という概念が存在したとは。
いいですよ、わからない問題がありましたらいつでもどーぞ♪
809132人目の素数さん:01/09/22 16:35
誰か>>799おせーて。
810yanyan:01/09/22 16:55
>>799
簡単のため4分円(第1象限)で考え、一応 a >= b とします。
不等式
1/(x^b+y^b) <= 1/(x^a+y^b) <= 1/y^b
から、b<1 なら収束します。b>1 なら収束しません。
b=1 のとき、a=1 なら計算できて収束。
b=1, a>1 なら変数変換 x=u, x^a+y=v とでも
おけば、発散がわかる。

結論:a<1 or b<1 or a=b=1 でのみ収束
(多分正しいと思うけど、誰か再チェックを)
811132人目の素数さん:01/09/22 16:59
教えてください。

有理整数環Zのmod nでの既約剰余類群(つまりZ/nZの
乗法可逆元の全体)の部分群、特に指数2のときについて調べた
いのですが、どのようにしたらよいのでしょうか?
(とりあえずnを素因数分解して中国式剰余定理で直積にしました。)
もし知っている方がいれば教えてください。
812132人目の素数さん:01/09/22 17:13
>>810
いっとることが無茶苦茶
813yanyan:01/09/22 17:23
>>812
なぜですか?
810をあなたのまわりの数学の人に見せてから
言って下さい。細かくは書いてはありませんが、
必要なことは810に含まれています。
814132人目の素数さん:01/09/22 17:30
なんでb>1なら収束しないといえるの?
∫(1/y^b) dyが発散しても 上からしか押さえてないから
もとの積分は収束するかもしれないじゃん
815132人目の素数さん:01/09/22 17:32
つまづいています。どうか助けてください。

複素数zと十分大きな自然数nに対して、以下の評価

|(z-1)(z-2)…(z-(n+1))|≦(n+[|z|]+2)!

が成り立つ・・・みたいなのですが、なんでそんな芸当ができるのかと
涙しています。特にガウス記号が出現するあたりがなんとも・・・。
何卒、よろしくお願いします。
816132人目の素数さん:01/09/22 17:38
n>2で考えて
オイラー関数φ(n)の性質よりφ(n)は偶数であるから、
アーベル群の基本定理より巡回群の直積に分解したとき
位数が2のベキ乗の部分群を含んでて、
指数が2の部分群Hは位数がφ(n)/2だから、H
は位数が2のベキ乗じゃない直積因子を含んで・・・・みたいに考えるんじゃないの?
817yanyan:01/09/22 17:40
>>814
もちろん左側の不等式で、下から評価します。
1/(x^b+y^b) を4分円で評価するには、
極座標を使えば簡単ですが、長くなるので
省略しました。やや難しいのは a>1,b=1
の場合です。
818132人目の素数さん:01/09/22 17:46

     / ̄ ̄ ̄ ̄\
   /:::::::::::::::::::::::::::::::::::\
  /::::::::::::∧:::∧::::∧::::::::::::ヽ
  .|::::::/V V  V  V\::::::|
  |::::::|三  \   /   |::::::|
 .|::::::|三 ⊂⊃ ⊂⊃  :|::::::|
 |;;;;(|三          |);;;;;|   / ̄ ̄ ̄ ̄ ̄ ̄ ̄ ̄ ̄ ̄ ̄ ̄
   ヽ三  (⌒⌒)  ノ     <  >>817 a<2 b<2で収束しとんじゃゴルァ
     `ー――――´       \____________a=b=3/2でやってみゴルァ
    /||:|(_乂_)|:||\
   //||:|    |:||\\
⊂l_/  ||:|     |:||  \lつ
     [二二二二二]
    .//::::|:::::::|::::\::\
  /:::/:::::::|::::::::|:::::::::\:::\
  ~ ̄ ̄| ̄|~ ̄~| ̄| ̄ ̄~
     | ̄|   .| ̄|
     ノ_|   |_ヽ
    (___|   |___)
819-:01/09/22 18:08
その画像ならココ
http://members.tripod.co.jp/bingo852/
820yanyan:01/09/22 18:24
>>818
そうですね、間違えました。a<2, b<2 で収束ですね。
810、813、817は取消します。
821132人目の素数さん:01/09/22 18:32
>>815
|z-k|≦|z|+|k|≦[z]+1+k をk=1〜n+1までかけりゃいいんじゃない?
822132人目の素数さん:01/09/22 18:37
>>811
たしかZ/nZの乗法群って巡回群だったような。
だからどうしたって?いや、どうということはないんだけど、それしか知らんのでいってみたかった。
823132人目の素数さん:01/09/22 18:49
それはそうとa<2,b<2は十分条件に過ぎないので
問題はまだ全く解決されておりませんよ。>>820
824132人目の素数さん:01/09/22 18:52
一知半解でものを言うべからず>>822
825132人目の素数さん:01/09/22 19:03
すみません、教えていただきたいのですが・・・。

1個のサイコロを6回投げる時、偶数の目が出る回数をXとする。
Xの平均をm、標準偏差をσとしたとき
確率P(|X−m|<σ)
を求めよ。
826132人目の素数さん:01/09/22 19:08
磁束密度Bの磁界で質量Mの物体が角速度ωで回転運動をしている。
この物体は?
827132人目の素数さん:01/09/22 19:20
角速度ωで回転運動している質量Mの物体。
828もったいないお化け:01/09/22 19:29
↑の何を求めるの?
っていうか磁束密度って物理の問題では?
829132人目の素数さん:01/09/22 20:21
>826
ちたま
830132人目の素数さん:01/09/22 20:44
BMW
831132人目の素数さん:01/09/22 21:14
>>830
おまえすげーよ
正解かどうかわからないけど
>>831
電荷(電流)がないと磁場と相互作用できないから
数学として正解にするのは難しいが
2ch的には許してしまうのか?
833132人目の素数さん:01/09/22 21:49
>>824
君のいふたうりだった。>>822はうそだ。わすれて。
834132人目の素数さん:01/09/22 21:54
「ガウスの消去法」と「ガウス・ジョルダンの掃出し法」の
それぞれの特徴と違いについて教えてください。
835132人目の素数さん:01/09/22 22:02
>825
統計の教科書見れ
ちゃんと書いてある筈
836132人目の素数さん:01/09/22 22:04
>>832
それとBMWとどう関係があるの?
837132人目の素数さん:01/09/22 22:55
Iアンペアの電流が525秒流れたとすると
BMW525iだ。
838132人目の素数さん:01/09/23 00:32
>>821
・・・おっしゃるとおりです。私って・・・。
とにかくありがとうございました。
>>837
こらこら
いちおう数学板のさくらスレなんだから
嘘は書くな
>>799
0<x,0<yの部分だけ考えればよく(0,0)付近以外は有限。
a≦0またはb≦0のとき有限。
0<aかつ0<bのとき
x^a=r^2・(cos(s))^2
y^b=r^2・(sin(s))^2
つまり
x=r^(2/a)・(cos(s))^(2/a)
y=r^(2/b)・(sin(s))^(2/b)
と変数変換すれば
1<1/a+1/bのとき有限と分かる。
841799じゃないけど:01/09/23 04:39
>840 なるほど!
842132人目の素数さん:01/09/23 04:41
教えてください。e1=1と仮定して進めても良いんでしょうか?

V={(x1,x2,・・・xn)|xi∈Z,|xi|≦n,|Σxi|≦n}
V上でe1x1+・・・+enxn=0 ならば xi=0(∀i)
を満たすei∈Nの中でΣeiを最小にするものを求めよ。
843132人目の素数さん:01/09/23 09:27
>839
嘘っていうより洒落だよ
844132人目の素数さん:01/09/23 12:00
カッシーニの軌道計算をおしえて
845132人目の素数さん:01/09/23 13:16
799
>>840
ありがとうございます。その変数変換は試行錯誤でみつけるんでしょうか?
>>807
有難う御座います。
>>802
三角形の面積=1/2(三辺の和)×内接円の半径
面積=24、AB+BC+CA=6+8+10だから。
848132人目の素数さん:01/09/23 17:54
教えてください!

Vを有限次元実線形空間
T^k_l(V)をV上のk,lテンソルとした時にtrace
tr:T^(k+1)_(l+1)(V) --> T^k_l(V)
とはいったいなんですか?
説明が書いてあるのですが、読んでも良くわかりません。
849132人目の素数さん:01/09/23 18:01
>>842
かりに試験問題でその問題がでていきなり
“e1=1としてよい”
から証明がはじまってたらまずいんじゃない?
850132人目の素数さん:01/09/23 18:06
偏微分方程式
1/2(σ^2S^2)∂^2C/(∂S)^2+rS∂C/∂S+∂C/∂t-rC=0

をCについて解きたいのですが
どうすればいいのか分かりませんのでおしえてください
851132人目の素数さん:01/09/23 18:28
>>850
どれが独立変数で、どれが何を変数とする関数なのかわかりません。
852132人目の素数さん:01/09/23 20:22
|f(z)|≦|z^2 + 1|で、g(z) = f(z) / z^2 + 1 、zは全ての複素数の時、
z = iのとき、|g(z)| = |f(z)| / |z^2 + 1| ってどうなるの?
1?それとも値がないの?limでz→iの時はどう?
言うまでもなくiは虚数単位のiね
853811ですが:01/09/23 20:26
>>816

>アーベル群の基本定理より巡回群の直積に分解したとき
>位数が2のベキ乗の部分群を含んでて、
>指数が2の部分群Hは位数がφ(n)/2だから、H
>は位数が2のベキ乗じゃない直積因子を含んで
>・・・・みたいに考えるんじゃないの?

すみませんが、ものわかりが悪いのでよくわからんのです。
たとえばn=35などのときはどうなるのでしょう??
854132人目の素数さん:01/09/23 20:38
>>852
z=iの時には定義されていない。
極限はf(z)の関数形が決まらないと定まらない。

(example)
f(z)=(z^2+1)/k
(k=7,9,112)
で考えると、それぞれ違う値なのが分かる。
855132人目の素数さん:01/09/23 20:46
次の問題の解説、お願いします。

二人の先生と四人の生徒が手をつないで輪をつくるとき、
先生どうしが向かい合う並び方は何通りあるか。

先生同士が向かいあうのはどういう形のときかと、
問題の解説お願いします。よければ、下のもんをコピーして、使ってください。

          ○
        ○   ○

        ○   ○
          ○
856132人目の素数さん:01/09/23 21:41
>>853
φ(35)=24=2^3×3
既約剰余群G=Z_8×Z_3Z, Z_4×Z_2×Z_3, Z_2×Z_2×Z_2×Z_3
の三つが考えられるが
G={1,2,3,4,6,8,9,11,12,13,16,17,18,19,22,23,24,26,27,29
31,32,33,34}で、このうち位数が2であるものは6,29のみ。
よって、G=Z_2×Z_4×Z_3となり
このうち指数に2の部分群はZ_4×Z_3とZ_2×Z_2×Z_3がある
後は位数4の元と位数3の元を適当に見つけてやるんじゃないのかな。。。
>>853,>>856
結論をみてからどうこうゆうもんじゃないかもしれんけど。
Z/35Z≡Z/5Z×Z/7Z
からZ/35Z^≡Z/5Z^×Z/7Z^(ただしR^はRの単数群=可逆元のなす群)
で有限体\{0}の乗法群は巡回群をしってれば
Z/5Z^=Z/4Z,Z/7Z^=Z/6Z=Z/2Z×Z/3Z
でもいいよね。
>>857
ついでにもひとつ。可換群G,H,Kについて
Hom(G×H,K)=Hom(G,K)×Hom(H,K)
と“Gの指数2の部分群”と“Hom(G,Z/2Z)”が一対一対応しているのを
しってれば
Hom(Z/6Z×Z/4Z,Z/2Z)=Hom(Z/6Z,Z/2Z)×Hom(Z/4Z,Z/2Z)
から4つあることもわかるね。もっと一般にZ/qZ (qは奇素数pのべき)
にたいしてZ/qZ^→Z/pZ^を自然な準同型とするとこの核Kの位数は
pべきだから
{e}→K→Z/qZ^→Z/pZ→{e}
から完全列
{e}→Hom(Z/pZ^,Z/2Z)→Hom(Z/qZ^,Z/2Z)→Hom(K,Z/2Z)={e}
を得てこれからZ/qZ^の指数2の部分群とZ/pZの指数2の部分群
全体に一対一対応があって一個しかないまでわかるね。
2べきはしらん。
>>858
あ、調子こいてまちごうた。
“Gの指数2の部分群”と“Hom(G,Z/2Z)で0写像でないもの”
の間に一対一対応があるだ。よって3つしかない。
860132人目の素数さん:01/09/23 22:28
>>857,>>858
あなた方いったいどう言う身分ですか?
861もったいないお化け:01/09/23 22:40
>>855

   先生A    ←こんな感じでまずセンコーを立たせて
 ○     ○  後で生徒が間に入るの
          だもんで4!=24
 ○     ○  参考書かなんかの円順列のとこ見なさい。
   先生B
>>842
n=4のときは(2,5,21,94)で最小値122になるので駄目。
>>811
n=p^k(pは3以上の素数、kは正の整数)のとき
Z/nZの既約剰余類群は位数p^(k−1)×(p−1)の巡回群になる。
n=2^k(kは3以上の整数)のとき
Z/nZの既約剰余類群はZ/(2^(k−2))Z×Z/2Zと同型になる。
n=ab(aとbは互いに素)のとき
Z/nZの既約剰余類群はZ/aZの既約剰余類群と
Z/bZの既約剰余類群の直積と同型になる。

例えばn=600=2^3×3×5^2のとき
Z/nZの既約剰余類群は
(Z/2Z×Z/2Z)×Z/2Z×Z/20Zと同型になる。

>>845
x^a+y^b=u^2+v^2となるように
x^a=u^2,y^b=v^2とした後
u=r・cos(s)
v=r・sin(s)
としてみつけた。
864842:01/09/24 02:07
解の候補   ↓(e1,e2,・・・)
n=1のとき(1)
n=2のとき(1,3)    
n=3のとき(1,4,14)
n=4のとき(1,5,21,91)

e1≠1の方が和がより小さくなるかもしれないので
上のが最小値なのかわかりません。
一般の場合にeiをどう求めればいいんでしょうか?
>>864
n=4のとき
(1,5,21,91)ではない。
(0,0,0,0)以外に
(−3,2,4,−1),(−1,−4,1,0),
(3,−2,−4,1),(1,4,−1,0)でも0になる。
n=4のときの最小値は(2,5,21,94)のときに122になる。
866842:01/09/24 03:11
>>865
すみません。まちがえました。
(2,5,21,94)が最小であることと
eiの求め方を教えてください。お願いします。
867842:01/09/24 03:45
e0=0,e1=1,ei+1=n*ei+ei-1から
得られる列を少し修正すればいいと思うけど
やり方と最小性が不明。
868842:01/09/24 04:00
>e0=0,e1=1,ei+1=n*ei+ei-1から
>得られる列を少し修正すればいいと思うけど
は違う気がしてきた。eiは上のものよりかなり小さく出来る?
869132人目の素数さん:01/09/24 04:11
>868
コンピュータで数値実験やってみた?
1,任意のベクトルAに対し、次の公式を導け
(1)div(rotA)=0
(2)rot(rotA)=▽(divA)-▽A
871132人目の素数さん:01/09/24 07:05
>>870
任意のベクトルAをA=(Ax,Ay,Az)とする。
rotA=∇×A (但し×は外積)
   =(∂/∂x ,∂/∂y ,∂/∂z)×(Ax,Ay,Az)
   =(∂Az/∂y − ∂Ay/∂z ,∂Ax/∂z − ∂Az/∂x ,∂Ay/∂x − ∂Ax/∂y) ……@

divA=∇・A (但し・は内積)
   =(∂/∂x ,∂/∂y ,∂/∂z)・(Ax,Ay,Az)
   =∂Ax/∂x + ∂Ay/∂y + ∂Az/∂z ……A

Δ=∇・∇=∂^2/(∂x)^2+∂^2/(∂y)^2+∂^2/(∂z)^2 ……B

(1)
div(rotA)= ∇・(∂Az/∂y − ∂Ay/∂z ,∂Ax/∂z − ∂Az/∂x ,∂Ay/∂x − ∂Ax/∂y) (∵@)
=(∂/∂x ,∂/∂y ,∂/∂z)・(∂Az/∂y − ∂Ay/∂z ,∂Ax/∂z − ∂Az/∂x ,∂Ay/∂x − ∂Ax/∂y)
=∂^2(Az)/∂x∂y − ∂^2(Ay)/∂x∂z + ∂^2(Ax)/∂y∂z − ∂^2(Az)/∂y∂x + ∂^2(Ay)/∂z∂x − ∂^2(Ax)/∂z∂y
=0
(∵∂^2(Az)/∂x∂y =∂^2(Az)/∂y∂x ,∂^2(Ay)/∂x∂z =∂^2(Ay)/∂z∂x ,∂^2(Ax)/∂y∂z =∂^2(Ax)/∂z∂y )

(2) rot(rotA)=grad(divA)−ΔA (←>>870の表記法は違うから気をつけよう)

左辺=rot(rotA)=∇×(∇×A)
今ベクトル ∇×(∇×A) のx成分 {∇×(∇×A)}x について考える
{∇×(∇×A)}x =∂(∂Ay/∂x − ∂Ax/∂y)/∂y −∂(∂Ax/∂z − ∂Az/∂x)/∂z
=∂^2(Ay)/∂x∂y−∂^2(Ax)/(∂y)^2−∂^2(Ax)/(∂z)^2+∂^2(Az)/∂x∂z
=∂^2(Ay)/∂x∂y−∂^2(Ax)/(∂y)^2−∂^2(Ax)/(∂z)^2+∂^2(Az)/∂x∂z+∂^2(Ax)/(∂x)^2−∂^2(Ax)/(∂x)^2
={∂^2(Ax)/(∂x)^2+∂^2(Ay)/∂x∂y+∂^2(Az)/∂x∂z}−{∂^2(Ax)/(∂x)^2+∂^2(Ax)/(∂y)^2+∂^2(Ax)/(∂z)^2}
= ∂(∂Ax/∂x+∂Ay/∂y+∂Az/∂z)/∂x − {∂^2/(∂x)^2+∂^2/(∂y)^2+∂^2/(∂z)^2}Ax
=∂(divA)/∂x −ΔAx (∵A、B)

同様に
{∇×(∇×A)}y =∂(divA)/∂y −ΔAy
{∇×(∇×A)}z =∂(divA)/∂z −ΔAz

ゆえに rot(rotA)=grad(divA)−ΔA
872871:01/09/24 07:09
外積はベクトル同士を演算してベクトル、
内積はベクトル同士を演算してスカラー、
gradはスカラーに作用してベクトルがでてくることに注意しよう。
>>324
これ証明できた。長いんでうぷすると板をよごすかも。
質問してた人まだよんでる?証明ききたい?
>>865
>n=4のときの最小値は(2,5,21,94)のときに122になる。
(0,−2,−4,1)で0になるからだめじゃないか?
そもそも問題文の|Σxi|≦nはあってるの?せめてΣ|xi|≦nぐらいじゃ
ないと手もつけられん気が。自作問題?
>>873
KARLタンは1年近くこの問題訊いてた。うぷせよ。
876132人目の素数さん:01/09/24 11:43
>>874
>(0,−2,−4,1)で0になるからだめじゃないか?
上の数を足すと−5なので条件をみたさないのでは?
877KARL ◆gjHKPQSQ :01/09/24 12:34
>>873
ぜひお願いします。
ここって答が用意されている問題しかきいちゃいけないの。
879教えて君:01/09/24 12:58
10月から放送大学で、微分積分入門、線形代数、数学基礎論を
聴講しようとしている者です。易しいという噂なのですが、数学は
苦手なもので、、、

授業でわからない所がありましたら、是非、ご教授ください。
よろしくお願いします。
880名無し:01/09/24 12:59
解答する人もボランティアなんだから,open-ended question
を書いて,困らせちゃダメよ。
おれ874。
べつに自作問題がいかんわけじゃないよ。
ただ過去多くの答えの出しようのない問題にチャレンジしてあとで
“これ自作問題なんです。そうですか、やっぱりとけませんか。”
なんてオチが結構あったからね。自作問題で答えがでない可能性が
あるならきちんと明示しといてほしいと思う。
882132人目の素数さん:01/09/24 15:24
>>873,>>875,>>877
ごめん。証明まちがってた。むずい。逝ってきます。
883132人目の素数さん:01/09/24 18:59
Cに沿って(?)複素積分

∫{zcosz/(z-π/2)^3}dz C:|z-i|=2

これどうやればいいんですか?
884132人目の素数さん:01/09/24 19:09
∫[C]zcoszdz/(z-π/2)^3   C:|z-i|=2

883ですけど、こう書くべきだったのかも
>>884
zcosz/(z-π/2)^3のz=π/2における留数を計算したらいい。
テーラー展開すりゃいっぱつ。面倒なのでw=z-π/2として
zcosz/(z-π/2)
=-{(w+π/2)sinw}/w^3
=-{w^(-1)-w/6+w^3/120-...}
-(π/2){w^(-3)-w^(-1)/6+w/120...}
886132人目の素数さん:01/09/24 20:11
>>877
だれかこたえていなかったけ?
887132人目の素数さん:01/09/24 20:13
>>877
つーか、カールたんはどこまでできたの?
888132人目の素数さん:01/09/24 20:16
>>878
んなばかな
889はなう:01/09/24 20:17
じゃわしも。>>610証明できたけど、まだ聞いた人みてますか〜??
890132人目の素数さん:01/09/24 20:20
>>889
みてません

610おわり
891かず小2:01/09/24 21:12
15*15ってなんですか?
892132人目の素数さん:01/09/24 21:39

申し訳ありません。
別スレのと同じことをこちらでもお聞きしてみます。↓

私は小学生ではありませんが子供のころから算数が苦手でした。
おかげで四則演算もまともに理解していないことを指摘されたりするこのごろです。
どなたか中学生あたりから高校までの授業内容を「足し算引き算→掛け算割り算→分数」のような感じで、詳しい内容までは結構ですのでずらっと全部並べていただけないでしょうか。
一つだけの線上に並べるのが妥当でないなら分類されても構いませんが、その場合レベルがわかるようにそれらをさらに段階的にわけていただけると有り難いです。
よろしくお願いします。
893132人目の素数さん:01/09/24 22:22
>777
>ちなみにこの問題は去年の算数オリンピックのファイナルの問題にクリソツ。
パンフに出てたアレ?俺は結局解けなかった。
>892
本屋で小学生用の参考書を買ってください
それが一番速いです。
895はなう:01/09/24 22:35
>>893
そうです。
896132人目の素数さん:01/09/24 23:00
>>892
>どなたか中学生あたりから高校までの授業内容を「足し算引き算→掛け算割り算→分数」のような感じで、詳しい内容までは結構ですのでずらっと全部並べていただけないでしょうか。

公文式でもやってろ
1,Bが定数の時、次のベクトルポテンシャル
A=(-By/2, Bx/2, 0)・・・@
  からなる確率密度B=(0, 0, b)であることを示せ。

2,ベクトルポテンシャル
  A=(0, Bx, 0)
に、適当なゲージ変換を実行すると上記の@が得られることを示せ。
898132人目の素数さん:01/09/24 23:07
なんで+よりxを先に計算するのか?
899132人目の素数さん:01/09/24 23:12
>>897
>Bが定数の時、次のベクトルポテンシャル
>A=(-By/2, Bx/2, 0)・・・@
>からなる確率密度B=(0, 0, b)であることを示せ。

意味不明。
BxやByって何?確率密度って何?
要はB=∇×Aを示せばいいんじゃないの?
900132人目の素数さん:01/09/24 23:20
(・∀・)900!!そろそろお引越し!!
誰かスレ作ってよ
>901
了解!
>>902
いつもの新スレさんが明朝作るだろうから待ってあげて

┌──────────────────────―─―┐
│                                      |
│                                      |
│                                      |
│                                      |
│                ∧_∧                      |
│               ( ・∀・)                |
│               (    )                 |
│               | | |                     |
│               (__)_)                  |
|              2ちゃんねる               |
│                                      |
│            スレッドを終了しています…           |
│                                      |
│                                      |
|                                      |
|                                      |
└───────────────────────――┘

━━━━━━━━━━━━━━━━━━━━━━━━━━━━━━

          このスレッドは終了しました
       質問がある人は、以下の新スレでお願いします

        ◆ わからない問題はここに書いてね 13 ◆
      http://cheese.2ch.net/test/read.cgi?bbs=math&key=1001342715

━━━━━━━━━━━━━━━━━━━━━━━━━━━━━━
>1 名前:132人目のさくらたん 投稿日:01/09/24 23:45
>4 名前:132人目のさくらたん 投稿日:01/09/24 23:47
>7 名前:132人目のさくらたん 投稿日:01/09/24 23:48
>12 名前:132人目のさくらたん 投稿日:01/09/24 23:49
>13 名前:132人目のさくらたん 投稿日:01/09/24 23:51

あーあ
準備して6分なのか?
907再度811です。:01/09/25 00:08
@皆さんありがとうございます。たいへん参考になります。
そうなると、私なりの考えでは「nが奇数のとき、その素因数分解にあら
われる素数の種類をmとすれば、Z/nZの指数2の部分群は2^m-1個ある」
となったのですがどうでしょうか?

Aついでに、858さんの
>Hom(Z/6Z×Z/4Z,Z/2Z)=Hom(Z/6Z,Z/2Z)×Hom(Z/4Z,Z/2Z)
>から4つあることもわかるね
のところで、「位数が偶数の巡回群→Z/2Zへの準同型写像」は
0写像以外に1つしかないですよね?(一応確認のため)

B“Gの指数2の部分群”と“Hom(G,Z/2Z)”が一対一対応している
これは群論の本にあるのですか?

?ばかりの長い文ですいません。。
>>907
さくら13号へだうぞ。
909132人目の素数さん:01/09/28 23:14
数学オリンピックの問題で
1^2001+2^2001+3^2001+....+2001^2001を13で割るときの余りを求めよ
で、答えは0なんですが求め方が分かりません
コンピューターを使わずに解ける方法を教えてください
>>909

>>905, >>908 参照

このスレ(さくらスレ12)は既に終わっています
911ちむ氏の使途:01/09/29 19:45
ジョーカー一枚を含む一組53枚のトランプの中から、3枚を選ぶとき、
3枚の中にジョーカーを含む選び方は何通りあるか。

a,a,a,b,b,cの6文字を一列に並べるとき、全部で何通りの
並び方があるか。

1,1,2,2,3,3,3,の7個の数字を一列に並べてできる7桁の自然数は、
全部で何通りか。

ある鉄道には、20の駅がある。この鉄道で、発駅、着駅、を明記した切符を
つくるとすると、何通りの切符ができるか。

数学板のみなさん、どのようにC、Pをすかえば解けるかがわかりません。
解説、お願いします。また、公式があれば、紹介してください。
912名無しゲノムのクローンさん:01/09/29 19:49
>>911
[joker]52枚のトランプか異なる2枚を選ぶ問題と同値。
[abc]6P6/(3!2!1!)
[123]上記問題と同値。
[鉄道]20C2×2
913132人目の素数さん:01/10/05 11:35
a(n)=1/n のとき部分和S(n)はどうなるんですか?
いろいろやってみたり、数学公式集とか見たりしたんですがわかりませんでした。
知っている方、どうか教えてください。単純な数列だけに気になります。
914なし:01/10/05 12:11
>>913
1/1 + 1/2 = (2+1)/1・2.
(2+1)/1・2 + 1/3 = (3(2+1) + 1・2)/1・2・3.

(3(2+1) + 1・2)/1・2・3 +1/4
=(4(3(2+1)+1・2) + 1・2・3)/1・2・3・4.

(4(3(2+1)+1・2) + 1・2・3)/1・2・3・4 + 1/5
=(5(4(3(2+1)+1・2) + 1・2・3) + 1・2・3・4)/1・2・3・4・5.
915132人目の素数さん:01/10/05 12:37
>>914
ありがとうございます。やはり工夫次第なんですね。
nのときは、実際に書くとなると長くなりそうです。
916ぺこすけ:01/10/05 12:39
計算のうまいやり方を教えてください。
下の式で表される、eを2乗したいのですが、
地道に計算するしかないでしょうか?

e=Σc(j)e(n-j)-Σc(j){Xp(n-j)-hXr(n-j)-h'Xr(n-1-j)}
 -Σc(j)Xp(n-1)-hΣc(j)Xr(n-1)-h'Σc(j)Xr(n-1-j)
917132人目の素数さん:01/10/05 12:40
>>913
ついでにΣ1/n^sも考えよう。
918132人目の素数さん:01/10/05 12:50
>>917
全部の数字をs乗すればよさそうです。
919なし:01/10/05 12:52
>>916
これらのΣが全部同じ種類なら、各項はΣとc(j)でまとまる。
他にもあるかもしれない。
920なし:01/10/05 12:54
>>918 s乗の和と和のs乗は異なる。
921132人目の素数さん:01/10/05 13:00
>>920
言葉足らずでした。nをn^sに置き換えるという意味です。
思えば、あたりまえですね。でも、これ以上はすっきりした形にできそうにないです。
922なし:01/10/05 13:09
1・2・3・....・(n-1)・n を「nの階乗」といい、 n! と表す。ただし、0!=1.

練習問題: 階乗を使って >>914 の分子を表し、適当に展開してまとめよ。
923132人目の素数さん:01/10/05 13:36
>>922
S(n)=n((n-1)...4(3(2+1)+2!)+3!)+...+(n-2)!)+(n-1)!/n!
ですか。う〜ん、見にくいですね。
ところで、nが無限大にいくとき、Sも無限大になるんですよね。
確かに分子の方が分母よりも断然大きくなりそうです。
924なし:01/10/05 13:47
4(3(2+1)+1・2) + 1・2・3 = 4(3(2+1!)+2!) + 3!
内側から順番に展開すれば
4(3(2+1!)+2!) + 3!
= 4(3・2+3・1!+2!) + 3!
= 4・3・2 + 4・3・1! + 4・2! + 3!
= (4・3・2)・0! + (4・3)・1! + (4)・2! + 3!
= (4!/1!) 0! + (4!/2!) 1! + (4!/3!) 2! + (4!/4!) 3!

ここで n・(n-1)・...・(m+1) = n! / m! であることに注意する。
925132人目の素数さん:01/10/05 14:00
>>924
なるほど、じゃあ
S(n)=(n!/1!)/0!+・・+(n!/m!)/(m-1)!+・・+(n!/n!)/(n-1)!
ですね。なかなかきれいにいくもんです。
926なし:01/10/05 14:10
>>925 厳密な証明は数学的帰納法を使うこと。

一般に関数 f(x) について
f(1) + f(2) + f(3) + ... + f(n) = Σ_[k=1,n] f(k)
とする。これを 1 から n までのf(k)の和(k=1,…,n)といい、
Σを総和(sum)記号、あるいはシグマ(ギリシャ文字σの大文字)と読む。

「4 :132人目のさくらたん」に詳しい記号の書き方が書かれている。

練習問題2: 総和を用いてS(n)を表せ。
927132人目の素数さん:01/10/05 14:25
>>926
S(n)=納k=1,n]1/k
=納k=1,n](n!/k!)/(k-1)!
総和記号は式を短く書けるけど、慣れていないせいか、とっつきにくいです。
928なし:01/10/05 14:30
>>927
n! は k を含まない共通項だから S(n)=n!・納k=1,n] 1/(k!(k-1)!).
すぐに分かるように S(n)≧n!. >>923
929なし:01/10/05 14:32
>>928 間違い。>>925 の最初の項から S(n)≧n! は明らか。
930なし:01/10/05 14:36
>>928 これより簡単にできないかどうか、後でじっくり考えてみよう。
今日の講義はおしまい。
931132人目の素数さん:01/10/05 14:39
>>928
>n! は k を含まない共通項だから S(n)=n!・納k=1,n] 1/(k!(k-1)!).
全然気がつきませんでした。やはり、年季の違いですね。
なんかアホなやりとりしてるけど
狽P/k=煤ik−1)!/k!は簡単なものを難しくしているだけだし
狽P/k=狽氏I(k−1)!/k!も間違い。
狽P/k=狽氏I/(k−1)!k!も間違い。
狽P/k≧n!も間違い。


━━━━━━━━━━━━━━━━━━━━━━━━━━━━━━

          このスレッドは終了しました
       質問がある人は、以下の新スレでお願いします

        ◆ わからない問題はここに書いてね 13 ◆
      http://cheese.2ch.net/test/read.cgi?bbs=math&key=1001342715

━━━━━━━━━━━━━━━━━━━━━━━━━━━━━━
933山崎:01/10/07 14:30
1、2,3,4,5の5個の数字から4個とって並べ、4けたの数字を作るとき次の問にこたえよ。
同じ数字を2回まで使ってよいとき、何通りの整数が出来ますか?
この問題、簡単かもしれませんが僕にはわからんのです。
934なし:01/10/09 08:30
>>932 バレたか。
935なし:01/10/09 17:57
5P4 + 5C1 × (3+2)C2 × 4P2

ここで 5C1 はどれか1つの数字を選ぶ方法の個数。
4P2 は選んだ数以外の 4つの数字を 2 つ並べる方法の個数。
(3+2)C2 は同じ数を置く場所を決める方法の個数。
936132人目の素数さん:01/10/29 16:09
円Oの円周上に3点A,B,Pをとり、弦ABに対する円周角∠APBをとる。
また円周の内側に任意の点Qをとり、円周の外側に任意の点Rをとる。
このとき∠AQB>∠APB>∠ARBとなることを証明せよ。
937名無し:01/10/29 16:20
>>936
直線ABに関してP,Q,Rが同じ側にないと成り立たないので,
以下,そのように仮定します。
位置の一般性とかがちょっと自信ないけど。
BQの延長と円との交点をSとすると,∠AQB>∠ASB
AR,BRが円と交わらないときは,僊BRと僊BPの内角を比較。
少なくとも一方が交わるときは,その交点をTとすれば,∠ATB>∠APB
ちなみに,『ここに書いてね』の最新のは15まであるから,今度からそっちに書いてね。
938132人目の素数さん:01/10/30 23:38
0≦2sinθ+cosθ≦1のとき
sinθ,cosθ,sinθ+2cosθの値の範囲を求めて下さい
939132人目の素数さん:01/11/01 05:21
0≦2sinθ+cosθ≦1のとき
sinθ,cosθ,sinθ+2cosθの値の範囲を求めて下さい
ほえー。

(√3)/2 ≦ cosθ≦1,
-(√3)/2 ≦ cosθ≦ 1/2

1/2 ≦ sinθ≦ (√3)/2,
-1/2 ≦ sinθ≦ 0

(1/2)-√3 ≦ sinθ+2cosθ ≦ 2

だよ、たぶん。
でももう終わったスレッドでの質問だから計算過程は省略だよ☆
941ベクトル:01/11/11 20:43
これをお願いします。

三角形OABにおいて、
 OA=5 OB=4 ∠OAB=60度
とし、点Oから辺ABにおろした垂線の足をHとする。
Vector(OA)=Vector(a) Vector(OB)=Vector(b)とおくとき、
Vector(OH)をVector(a)、Vector(b)を用いてあらわせ。

頻出問題だと思うんですが、この手の問題はなぜか苦手・・・。
よろしくお願いします。
942132人目の素数さん:01/11/20 22:43
重なった部分が線対称の五角形となるように一辺がaの正方形を折る.
この時の五角形の面積Sの最大値を求めよ.がわかりません.,
943132人目の素数さん:01/11/21 00:03
>942
最大値はないんじゃないの?
あ、よく見たら旧スレじゃん!
スマソ・・・
945132人目の素数さん:01/11/21 00:27
>941
vector(OH)=(1-t) vector(OA)+t vector(OB)とおけば、
(HがABの内分点だから)
vector(OH)・(vector(OB)−vector(OA))=0
ここで、・は「内積」。高校生がんばれ。
946線形代数:01/11/25 20:33
/-1 -1 -6 3\
| 1 -2 -3 0 |
| -1 1 0 1 |
\-1 -1 -5 3/
の固有値の求め方を教えて下さい。
━━━━━━━━━━━━━━━━━━━━━━━━━━━━━━

             新しいスレッドが出来ましたので
     新たに質問をする方はこちらでして頂けると嬉しいですわ

         ◆ わからない問題はここに書いてね 16 ◆
     http://cheese.2ch.net/test/read.cgi/math/1005735838/l50

━━━━━━━━━━━━━━━━━━━━━━━━━━━━━━
948masa:01/11/26 02:22
確率微分方程式
dS(t)/S(t)=μ(S(t),t)dt+σ(S(t),t)dB(t)
について このようになる過程と理論がいまいち解かりません
教えて下さい。
>>948
諦めろ
法政程度では理解できん
何故 >>947 に書かれていることが読めないのだろうか。
951132人目の素数さん:01/11/26 02:33
>950
じゃあ新スレつくってくれ・・・
なんで「じゃあ」となるのか分からない。
953132人目の素数さん:01/12/17 01:36
age
954132人目の素数さん:01/12/17 02:25
このシリーズのスレ、いっつもこうやって1000いくまでしつこくあがってきやがる。
ちゃんと最後まで使おうYO
955132人目の素数さん:01/12/17 04:17
age
956132人目の素数さん:01/12/17 11:58
だね                 
>>953-956
勘違いがこれ以上続くと非常に迷惑なので忠告。

「最後まで使おう」とかいう新参の発想はやめろ。
なぜ敢えてこういう状況なのかちゃんと調べろ。

誘導以外のレスはこれ以降禁止。以上。
958 :01/12/17 14:21
多項式の因数分解が一意にできるのは何故ですか?

m君質問しておいたよ:-)
誰が迷惑?
>958
一意じゃなかったら零点の位置が変わってしまうから違う関数になってしまう。
961132人目の素数さん:01/12/17 22:47
もうひと息age
962132人目の素数さん:01/12/17 22:54
係数にある条件のついた多項式ね。実数係数なら問題無い。
963132人目の素数さん:01/12/18 23:15
     △--△   / ̄ ̄ ̄ ̄ ̄ ̄
    ( ・∀・ ) < ゲト〜
    /つ963つ  \______
  〜人  Y
   し'(_)
sage
965132人目の素数さん:01/12/19 15:42
いま一度age
966132人目の素数さん:01/12/19 16:07
coshxって何?
967132人目の素数さん:01/12/20 12:18
age
968132人目の素数さん:01/12/20 16:43
      犬
       犬
     犬犬犬犬
        犬
    犬犬犬 犬 犬犬犬
      犬 犬 犬
     犬  犬 犬
    犬   犬  犬
   犬    犬   犬
        犬  
               犬   犬
             犬犬犬犬犬犬犬犬犬犬
               犬   犬
               犬   犬 
            犬犬犬犬犬犬犬犬犬犬
               犬   犬
              犬    犬
             犬     犬 
今後(このスレ含む)は移行後スレストが必要かも。
970132人目の素数さん:01/12/31 01:42
あら?
971132人目の素数さん :01/12/31 04:17
10L−2/3=5/2
  (10Lマイナス3分の2乗=5分の2)
 をL=で解く。
 解答にはL=8となっているんですが、与式が載っていなくて
 わかりません。すみませんが、途中過程を書いてくれませんか?
 お願いします。
━━━━━━━━━━━━━━━━━━━━━━━━━━━━━━

             新しいスレッドが出来ましたので
     新たに質問をする方はこちらでして頂けると嬉しいですわ

         ◆ わからない問題はここに書いてね 19 ◆
     http://cheese.2ch.net/test/read.cgi/math/1009102965/l40

━━━━━━━━━━━━━━━━━━━━━━━━━━━━━━
973132人目の素数さん:01/12/31 16:17
最後まで使い切りたいage
974132人目の素数さん:01/12/31 18:24
最後まで使い切りたいage              
>>973-974 おい貴様、あげてんじゃねーよ。
お願いですからsageで書いてください。
厨房の季節だねぇ…年越しちゃったよオィ
こうやって埋める事に荷担する私のような厨房がいるから
いつまでもこのような事は繰り返されちゃうんだろうなぁ……
10011001
このスレッドは1000を超えました。
もう書けないので、新しいスレッドを立ててくださいです。。。